07年英语高考卷_21世纪教育网-二一教育 (2024)

2007年普通高等学校招生全国统一考试全国II卷 英语 
英语
本试卷分第I卷(选择题)和第II卷(非选择题)两部分。第I卷1至12页。第II卷13至14页。考试结束,将本试卷和答题卡一并交回。
第I卷
注意事项:
答题前,考生在答题卡上务必用直经0.5毫米黑色墨水签字笔将自己的姓名,准考证号填写清楚,并贴好条形码。请认真核准条形码上的准考证号、姓名和科目。
每小题选出答案后,用2 B铅笔把答题卡上对应题目的答案标号涂黑。如需改动,用橡皮擦干净后,再选涂其他答案标号,在试题卷上作答无效。
第一部分 听力(共两节,满分30分)
做题时,先将答案标在试卷上。录音内容结束后,你将有两分钟的时间将试卷上答案转涂到答题卡上。
第一节 (共5 小题;每小题 1.5 分,满分 7.5 分)
听下面5段对话。每段对话后有一个小题,从题中所给的A、B、C三个选项中选出最佳选项,并标在试卷的相应位置。听完每段对话后,你都有10秒钟的时间来回答有关小题和阅读下一小题。每段对话仅读一遍。
例:How much is the shirt?
£19.15.
£9.15.
£9.18.
答案是B。
1. Who is coming for tea?
A. John. B. Mark. C. Tracy.
2. What will the man do next?
A. Leave right away. B. Stay for dinner. C. Catch a train.
3. What does the man come for?
A. A lecture. B. A meeting. C. A party.
4. What size does the man want?
A. 9. B. 35. C. 39.
5. What are the speakers talking about?
A. Life in Southeast Asia. B. Weather condition. C. A holiday tour.
第二节 (共15小题;每小题 1.5 分,满分 22.5 分)
听下面5 段对话或独白。每段对话或独白后有几个小题,从题中所给的A、B、C三个选项中选出最佳选项,并标在试卷的相应位置。听每段对话或独白前,你将有时间阅览室读各个小题,每小题5秒钟;听完后,各小题将给出5秒钟的作答时间。每段对话或独白读两遍。
听第6段材料,回答第6、7题。
6. What is the man doing?
A. Giving a speech. B. Chairing a meeting. C. Introducing a person.
7. Why does the woman sing so well?
A. She has a great teacher. B. She teaches singing. C. She is young.
听第7段材料,回答第8、9题。
8. What is the second gift for Jimmy?
A. A car. B. A watch. C. A computer.
9. Why does Jimmy feel happy?
A. He lives with his parents. B. He’s got what he dreamt of.
C. He’s received lots of presents.
听第8段材料,回答第10至12题。
10. What is the relationship between the speakers?
A. They are friends. B. They are strangers to each other.
C. They are husband and wife.
11. Why does the woman come to talk with the man?
A. To get a job. B. To take a test. C. To see the secretary.
12. What does the man mean by saying sorry?
A. He can’t hear the woman clearly. B. He doesn’t need a designer.
C. He can’t help the woman.
听第9段材料,回答第13至16题。
13. What do we know about the woman?
A. She lives close to the office. B. She is new to the company.
C. She likes the big kitchen.
14. How does the man go to work?
A. On foot. B. By bus. C. By car.
15. Why was Susan late for work?
A. She missed the bus. B. Her train was late. C. Her car broke down.
16. What will the man do the next day?
A. Go to work by train. B. Visit Lily in her flat. C. Leave home earlier.
听第10段材料,回答第17至 20 题。
17. Where can you most probably hear this talk?
A. In a class of the English language. B. In a class of the Greek language.
C. In a class of the French language.
18. How long does the class last?
A. 11 weeks. B. 13 weeks. C. 15 weeks.
19. What is “the short-cut” to learning words according to the speaker?
A. Taking more courses. B. Reading basic words aloud.
C. Learning how words are formed.
20. Why is the class popular?
A. It is not offered each term. B. It’s taught by Professor Morris.
C. It helps to master some useful rules.
第二部分 英语知识运用(共两节,满分45分)
第一节 单项填空(共15小题;每小题1分,满分15分)
例:It is generally considered unwise to give a child ________ he or she wants.
A. however B. whatever C. whichever D. whenever
答案是B。
21. ---Have you heard the latest news? ---No, What ________?
A. is it B. is there C. are they D. are those
22. Some pre-school children go to a day care center, __________ they learn simple games and songs.
A. then B. there C. while D. where
23. The manager suggested an earlier date ____________ the meeting.
A. on B. for C. about D. with
24. ---It’s a long time since I saw my sister. ---_______________ her this weekend?
A. Why not visit B. why not to visit
C. Why not visiting D. Why don’t visit
25. ---The last one __________________ pays the meal. ---Agreed!
A. arrived B. arrives C. to arrive D. arriving
26. I won’t call you, ________ something unexpected happens.
A. unless B. whether C. because D. while
27. ---How’s your tour around the North Lake? Is it beautiful?
---It ________ be, but it is now heavily polluted.
A. will B. would C. should D. must
28. We all know that, __________, the situation will get worse.
A. not if dealt carefully with B. if not carefully dealt with
C. if dealt not carefully with D. not if carefully dealt with
29. I smell something __________ in the kitchen. Can I call you back in a minute?
A. burning B. burnt C. being burnt D. to be burnt
30. Does this meal cost $50? I __________ something far better than this!
A. prefer B. expect C. suggest D. suppose
31. Between the two generations, it is often not their age, _________ their education that causes misunderstanding.
A. like B. as C. or D. but
32. I know a little bit about Italy as my wife and I ________ there several years ago.
A. are going B. had been C. went D. have been
33. ---Can you read the sign, sir? No smoking allowed in the lift! ---__________________.
A. Never mind B. Don’t mention it
C. Sure, I don’t smoke D. Pardon me
34. “Goodbye, then,” she said, without even ___________ from her book.
A. looking down B. looking up C. looking away D. looking on
35. The flowers were so lovely that they __________ in no time.
A. sold B. had been sold C. were sold D. would sell
第二节 完型填空(共20小题;每小题 1.5 分,满分 30 分)
阅读下面短文,从短文后各题所给的四个选项(A、B、C和D)中,选出可以填入空白处的最佳选项,并在答题卡上将该项涂黑。
In the United States there was an unusual tale telling of the daughter of a mechanic (技工). One day while walking along the bank of a lake, the girl 36 to see 20 eggs laid by a wild goose. After some time the girl 37 the mother would not return to her eggs and she 38 to take them home. There she carefully 39 the eggs in the heat of a lamp. Several days 40 the eggs broke and the baby geese came into the 41 .
Geese are known to take the first living thing they see as their mother. 42 , to these young geese, the girl was their mother.
As they 43 , the girl was able to 44 her birds to run across the grass, but she could not teach them to 45 . The girl became increasingly worried about this, both when 46 and in her dreams. Later, she had an 47 : She would pilot a plane to guide them in 48 . She asked her father for a plane and he assembled(组装)a small aircraft for her.
Caring about 49 safety, the father decided to pilot the plane himself. However, the birds did not 50 or follow him, and 51 slept in the grass.
One day, the girl 52 into the plane, started it and soon left the 53 . Seeing their mother take to the air, the birds 54 flapped(拍打)their wings and 55 . She flew the plane freely in the sky, her young birds following.
36. A. managed B. attempted C. happened D. supposed
37. A. realized B. expected C. imagined D. admitted
38. A. helped B. decided C. afforded D. meant
39. A. placed B. protected C. treated D. examined
40. A. ago B. out C. later D. long
41. A. family B. lake C. home D. world
42. A. But B. Also C. Thus D. Still
43. A. increased B. improved C. rose D. grew
44. A. ask B. lead C. want D. allow
45. A. fly B. race C. swim D. sing
46. A. asleep B. away C. around D. awake
47. A. idea B. opinion C. explanation D. excuse
48. A. sky B. heaven C. flight D. plane
49. A. his B. her C. their D. its
50. A. respect B. remember C. recognize D. receive
51. A. so B. instead C. hardly D. too
52. A. climbed B. looked C. reached D. fell
53. A. house B. floor C. water D. ground
54. A. secretly B. disappointedly C. patiently D. eagerly
55. A. looked away B. set out C. went by D. turned back
第三部分 阅读理解(共20小题;每小题 2 分,满分 40分)
阅读下列短文,从每题所给的四个选项(A、B、C和D)中,选出最佳选项,并在答题卡上将该项涂黑。
A
It was a winter morning, just a couple of weeks before Christmas 2005. While most people were warming up their cars, Trevor, my husband, had to get up early to ride his bike four kilometers away from home to work. On arrival, he parked his bike outside the back door as he usually does. After putting in 10 hours of labor, he returned to find his bike gone.
The bike, a black Kona 18 speed, was our only transport. Trevor used it to get to work, putting in 60-hour weeks to support his young family. And the bike was also used to get groceries(食品杂货),saving us from having to walk long distances from where we live.
I was so sad that someone would steal our bike that I wrote to the newspaper and told them our story. Shortly after that, several people in our area offered to help. One wonderful stranger even bought a bike, then called my husband to pick it up. Once again my husband had a way to get to and from his job. It really is an honor that a complete stranger would go out of their way for someone they have never met before.
People say that a smile can be passed from one person to another, but acts of kindness from strangers are even more so. This experience has had a spreading effect in our lives because it strengthened our faith in humanity(人性)as a whole. And it has influenced(影响)us to be more mindful of ways we, too, can share with others. No matter how big or how small, an act of kindness shows that someone cares. And the results can be everlasting.
56. Why was the bike so important to the couple?
A. The man’s job was bike racing. B. It was their only possession.
C. It was a nice Kona 18 speed. D. They used it for work and daily life.
57. We can infer from the text that ____________.
A. the couple worked 60 hours a week. B. people were busy before Christmas
C. the stranger brought over the bike D. life was hard for the young family.
58. How did people get to know the couple’s problem?
A. From radio broadcasts. B. From a newspaper.
C. From TV news. D. From a stranger.
59. What do the couple learn from their experience?
A. Strangers are usually of little help. B. One should take care of their bike.
C. News reports make people famous. D. An act of kindness can mean a lot.
B
Many animals recognize their food because they see it. So do humans. When you see an apple or a piece of chocolate you know that these are things you can eat. You can also use other senses when you choose your food. You may like it because it smells good or because it tastes good. You may dislike some types of food because they do not look, smell or taste very nice. Different animals use different senses to find and choose their food. A few animals depend on only one of their senses, while most animals use more than one sense.
Although there are many different types of food, some animals spend their lives eating only one type. The giant panda(大熊猫)eats only one particular type of bamboo(竹子). Other animals eat only one type of food even when given the choice. A kind of white butterfly(蝴蝶)will stay on the leaves of a cabbage, even though there are plenty of other vegetables in the garden. However, most animals have a more varied diet(多样化饮食). The bear eats fruits and fish. The fox eats small animals, birds and fruits. The diet of these animals will be different depending on the season.
Humans have a very varied diet. We often eat food because we like it and not because it is good for us. In countries such as France and Britain, people eat foods with too much sugar. This makes them overweight, which is bad for their health. Eating too much red meat and animal products, such as butter, can also be bad for the health. Choosing the right food, therefore, has become an area of study in modern life.
60. We can infer from the text that humans and animals _________.
A. depend on one sense in choosing food B. are not satisfied with their food
C. choose food in similar ways D. eat entirely different food
61. Which of the following eats only one type of food?
A. The white butterfly. B. The small bird.
C. The bear. D. The fox.
62. Certain animals change their choice of food when ___________.
A. the season changes B. the food color changes
C. they move to different places D. they are attracted by different smells
63. We can learn from the last paragraph that __________.
A. food is chosen for a good reason B. French and British food is good
C. some people have few choices of food D. some people care little about healthy diet
C
Our “Mommy and Me” time began two years ago. My next-door neighbor and fellow mother, Christie, and I were out in our front yards, watching seven children of age 6 and under ride their bikes up and down. “I wish I could take one of my children out alone,” said Christie.
Then we worked out a plan: When Christie takes one of her children out, I’ll watch her other three. And when she watches two of mine, I’ll take someone out.
The children were extremely quick to accept the idea of “Mommy and Me” time. Christie’s daughter, McKenzie, went first. When she returned, the other children showered her with tons of questions. McKenzie was smiling broadly. Christie looked refreshed and happy. “She’s like a different child when there’s no one else around,” Christie shared with me quickly. With her mother all to herself, McKenzie didn’t have to make an effort to gain attention.
Just as Christie had noticed changes in McKenzie, I also discovered something different in each of my children during our alone times. For example, I am always surprised when my daughter, who is seldom close to me, holds my hand frequently. My stuttering(口吃的)son, Tom, doesn’t stutter once during our activities since he doesn’t have to struggle for a chance to speak. And the other son, Sam, who’s always a follower when around other children shines as a leader during our times together.
The “Mommy and Me” time allows us to be simply alone and away with each child ---talking, sharing, and laughing, which has been the biggest gain. Every child deserves(应得到)to be an only child at least once in a while.
64. What is the text mainly about?
A. The experience of the only child being with mother.
B. The advantage of spending time with one child at a time.
C. The happy life of two families.
D. The basic needs of children.
65. Right after McKenzie came back, the other children were _____________.
A. happy B. curious C. regretful D. friendly
66. What is one of the changes the author finds in her children?
A. The daughter acts like a leader. B. Sam holds her hand more often.
C. The boys become better followers. D. Tom has less difficulty in speaking.
67. The author seems to believe that ___________.
A. having brothers and sisters is fun
B. it’s tiring to look after three children
C. every child needs parents’ full attention
D. parents should watch others’ children
D
Anyone who cares about what schools and colleges teach and how their students learn will be interested in the memoir(回忆录)of Ralph W. Tyler, who is one of the most famous men in American education.
Born in Chicago in 1902, brought up and schooled in Nebraska, the 19-year-old college graduate Ralph Tyler became hooked on teaching while teaching as a science teacher in South Dakota and changed his major from medicine to education.
Graduate work at the University of Chicago found him connected with honorable educators Charles Judd and W. W. Charters, whose ideas of teaching and testing had an effect on his later work. In 1927, he became a teacher of Ohio State University where he further developed a new method of testing.
Tyler became well-known nationality in 1938, when he carried his work with the Eight-Year Study from Ohio State University to the University of Chicago at the invitation of Robert Hutchins.
Tyler was the first director of the Center for Advanced Study in the Behavioral Sciences at Stanford, a position he held for fourteen years. There, he firmly believed that researchers should be free to seek an independent(独立的)spirit in their work.
Although Tyler officially retired in 1967, he never actually retired. He served on a long list of educational organizations in the United States and abroad. Even in his 80s he traveled across the country to advise teachers and management people on how to set objectives(目标)that develop the best teaching and learning within their schools.
68. Who are most probably interested in Ralph W. Tyler’s memoir?
A. Top managers. B. Language learners.
C. Serious educators. D. Science organizations.
69. The words “hooked oh teaching” underlined in Paragraph 2 probably mean ________.
A. attracted to teaching B. tired of teaching
C. satisfied with teaching D. unhappy about teaching
70. Where did Tyler work as the leader of a research center for over 10 years?
A. The University of Chicago. B. Stanford University.
C. Ohio State University. D. Nebraska University.
71. Tyler is said to have never actually retired because ____________.
A. he developed a new method of testing B. he called for free spirit in research
C. he was still active in giving advice D. he still led the Eight-Year Study
E
Today about 70 countries use Daylight Saving Time (DST). Daylight Saving was first introduced during World War I in Australia. During the world wars, DST was used for the late summers beginning January 1917 and 1942, and the full summers beginning September 1942 and 1943.
In 1967, Tasmania experienced a drought(干旱). The State Government introduced one hour of daylight saving that summer as a way of saving power and water. Tasmanians liked the idea of daylight saving and the Tasmanian Government has declared daylight saving each summer since 1968. Persuaded by the Tasmanian Government, all states except two passed a law in 1971, for a test use of daylight saving. In 1972, New South Wales, South Australia and Victoria joined Tasmania for regular daylight saving, but Queensland did not do so until 1989.
Tasmania, Queensland and South Australia have had irregular plans, often changing their dates due to politics or festivals(节日). For example, in 1992, Tasmania extended(延长)daylight saving by an extra month while South Australia began extending daylight saving by two weeks for the Adelaide Festival. Special daylight saving plans were made during the Sydney 2000 Olympic Games.
The differences in daylight saving in Australia continue to cause serious problems in transport and many other social activities. It also reduces the number of hours in the working day that are common to all centers in the country. In particular, time differences along the east coast cause major differences, especially for the broadcasters of national radio and television.
72. Daylight Saving Time was introduced in Tasmania _______________.
A. to stop the drought in 1967 B. to support government officials
C. to pass a special law in the state D. to save water and electricity
73. According to the text, which state was the last to use DST?
A. Victoria. B. Queensland.
C. South Australia. D. New South Wales.
74. What can we learn about DST in some Australian states?
A. It doesn’t have fixed dates. B. It is not used in festivals.
C. Its plan was changed in 2000. D. It lasts for two weeks.
75. What do we know about the use of DST from the last paragraph?
A. There exist some undesirable effects. B. It helps little to save energy.
C. It brings about longer working days. D. Radio and TV programs become different.
2007年普通高等学校招生全国统一考试
英语
第II卷
注意事项:
答题前,考生先在答题卡上用直径0.5毫米黑色墨水签字笔将自己的姓名、准考证号填写清楚,然后贴好条形码,请认真核准条形码上的准考证号、姓名和科目。
第II卷共2页,请用直径0.5毫米黑色墨水签字笔在答题卡上各题的答题区域内作答,在试题卷上作答无效。
第四部分 写作(共两节,满分35分)
第一节 短文改错(共10小题;每小题1分,满分10分)
此题要求改正所给短文中的错误。对标有题号的每一行作出判断:如无错误,在该行右边横线上画一个勾(√);如有错误(每行只有一个错误),则按下列情况改正:
此行多一个词:把多余的词用斜线(╲)划掉,在该行右边横线上写出该词,并也用斜线划掉。
此行缺一个词:在缺词处加一个漏字符号(∧),在该行右边横线上写出该加的词。
此行错一个词:在错的词下划一横线,在该行右边横线上写出改正后的词。
注意:原行没有错的不要改。
I was only about six that he held his hand 76. _________________
out to me. I took it in me and we walked. Then 77. _________________
I ask him, “Grandpa, how come you have 78. _________________
so much lines on your hand?” he laughed and 79. _________________
said, “Well, that’s a big question!” He was 80. _________________
silent for a moment. So he answered slowly: 81. _________________
“Each these lines stands for a trouble in my 82. _________________
life.” I looked at his other hand. “But Grandpa, 83. _________________
what do you have more lines on that one?” 84. _________________
“Because there are more the honors and joys 85. _________________
in my life.”
第二节 书面表达(满分25分)
假定你是李华,希望通过外籍教师Peter找一个英语笔友。请写一封短信,描述一下你理想中笔友的条件,并说明为什么选这样的笔友。具体条件包括:
年龄;
性别;
爱好(旅游、运动、宠物等)。
注意:1.词数100 左右;
2. 可以适当增加细节,以使行文连贯;
3. 开头语已为你写好。
Dear Peter,
I am writing to ask whether you are able to do me a favor.
Best regards,
Li Hua
(在试题卷上作答无效)
参考答案
1---5 BABAC 6---10 CACBB 11---15 ACABB 16---20 CABCC
21---25 ADBAC 26---30 ACBAB 31---35 DCDBC
36---40 CABAC 41---45 DCDBA 46---50 DACBC 51---55 BADDB
56---60 DDBDC 61---65 AADBB 66---70 DCCAB 71---75 CDBAA
第四部分 第一节 短文改错:
76. that — when 77. me —mine 78. ask — asked 79. much — many
80. √ 81. So — Then / And 82. Each∧ these 加上 of 83. hands — hand
84. what — why 85. the 去掉
参考译文:
One possible version:
Dear Peter,
I’m writing to ask whether you are able to do me a favor.
I want to have a pen friend, hopefully a girl in her early twenties, and with interests similar to mine. In my mind, she is someone who is interested in traveling, swimming, and playing table tennis. Besides, it would be better for her to have a pet dog as I have kept one at home for some time. With such a pen friend, I think I can share with her our traveling experiences, taking care of pets, or whatever we have in common. And I believe I will improve my English by doing so and learn more about her country.
I look forward to hearing from you soon.
Best regards,
Li Hua
2007年普通高等学校招生全国统一考试试题卷
(全国卷I)
英语
注意事项:
本试题卷分第1卷(选择题) 和第II卷 (非选择题) 两部分, 总分150分, 考试时间120分钟.
答题前, 考生须将自己的姓名、准考证号、考场号、座位号填写在本试题卷指定的位置。
选择题的每小题选出后,用2B铅笔将答题卡上对应题目的答案标号涂黑,如需改动,用橡皮擦干净后,再选涂其它答案,不能答在试题卷上。
非选择题必须使用0.5毫米的黑色字迹的签字笔在答题卡上书写,字体工整,笔迹清楚。
非选择题必须按照题号顺序在答题卡上各题目的答题区域内作答。超出答题区域或在其它题的答题区域内书写的答案无效;在草稿纸、本试题卷上答题无效。
考试结束,将本试题卷和答题卡一并交回。
第一卷 (选择题)
第一部分 英语知识运用(共三节,满分50分)
语音知识(共5小题;每小题1分,满分5分)
从A、B、C、D四个选项中,找出其划线部分与所给单词的划线部分读音相同的选项,并在答题卡上将该项涂黑。
例:have
A. gaveB. saveC. hatD. made
stopA. loseB. womanC. shockD. rose
breatheA. thickB. southernC. mathematicsD. method
groundA. houseB. countryC. groupD. cough
centerA. oceanB. decideC. causeD. socialist
animalA. acheB. anythingC. advanceD. anxious
语法与词汇知识(共15小题;每小题1分,满分15分)
从A、B、C、D四个选项中,选出可以填入空白处的最佳选项,并在答题卡将该项涂黑。
例:We ____ last night, but we went to the concert instead.
A. must have studiedB. might studyC. should have studiedD. would study
答案是C。
–We have booked a room for today and tomorrow.–____, sir.A. I’m sureB. My pleasureC. It’s all rightD. I’ll check
____ felt funny watching myself on TV.A. OneB. ThisC. ItD. That
____ he had not hurt his leg, John would have won the race.A. IfB. SinceC. ThoughD. When
After two years’ research, we now have a ____ better understanding of the disease.A. veryB. farC. fairlyD. quite
Speaking of all the songs he has written, I think this is probably his ____ one.A. better-knownB. well-knownC. best-knownD. most-known
If Joe’s wife won’t go to the party, ____.A. he will eitherB. neither will heC. he neither willD. either he will
At the beginning of class, the noise of desks ____ could be heard outside the classroom.A. opened and closedB. to be opened and closedC. being opened and closedD. to open and close
I have ____ all my papers but I still can’t find my notes.A. looked throughB. looked forC. looked afterD. looked out
–I’m sorry to have kept you waiting.–____, Bill.A. You’re welcomeB. Go aheadC. Don’t mention itD. No problem
–Is there anything wrong, Bob? You look sad.–Oh, nothing much. In fact, I ____ of my friends back home.A. have just thoughtB. was just thinkingC. would just thinkD. will just be thinking
Some people choose jobs for other reasons ____ money these days.A. forB. exceptC. besidesD. with
____ matters most in learning English is enough practice.A. WhatB. WhyC. WhereD. Which
Why don’t you just ____ your own business and leave me alone?A. makeB. openC. considerD. mind
–Could you tell me the way to ____ Johnsons, please?--Sorry, we don’t have ____ Johnson here in the village.A. the; theB. the; aC. /; theD. the; /
–Tom, you didn’t come to the party last night?–I ____, but I suddenly remembered I had homework to do.A. had to B. didn’t C. was going toD. wouldn’t
完型填空(共20小题;每小题1.5分,满分30分)
阅读下面短文,从短文后各题所给的四个选项(A、B、C、D)中,选出可以填入空白处的最佳选项,并在答题卡上将该项涂黑。
We arrived in Spain for the first time a few weeks ago. I decided to 21 a car because we had sold the one we had in England before 22 home. Yesterday the sales office rang us to say the car was 23 . I had tried out a model like it before, but as I was not yet 24 driving in this city, my wife did not want me to collect it 25 so we went together to 26 it. We paid for the car and 27 the papers. They told us that there was 28 petrol(汽油)to take us to a garage, where we could fill up. The 29 garage to the office was about 100 yards away and we got there 30 . But when I turned into main road I suddenly saw a lot of cars racing 31 me. I got out of 32 as fast as I could by backing into the garage 33 and the man behind 34 me.
“It’s such a problem to 35 to drive on the right side, isn’t it?” my wife said. “Yes, if only I had had a few lessons for 36 .” I replied. “You had better go 37 on the way home,” my wife said. “You’d be sorry if you had 38 on the first day, wouldn’t you?” While we were talking, the man behind got out of his car and said in good English,” Would you mind telling me 39 you are thinking of leaving? 40 are you going to sit in your car all day?”
A. borrowB. driveC. buyD. choose
A. leavingB. makingC. returningD. getting
A. rightB. readyC. fixedD. sold
A. sure ofB. satisfied withC. interested inD. used to
A. on my ownB. right awayC. in a hurryD. on the way
A. receiveB. bringC. orderD. fetch
A. acceptedB. wroteC. signedD. copied
A. littleB. enoughC. muchD. no
A. bestB. nearestC. quickestD. cleanest
A. latelyB. directlyC. safelyD. slowly
A. afterB. withC. aroundD. towards
A. their wayB. the garageC. their sightD. the car
A. at lastB. once moreC. as usualD. as well
A. caughtB. cheeredC. shouted atD. chatted with
A. prepareB. continueC. chooseD. remember
A. discussionB. adventureC. experimentD. practice
A. carefullyB. smoothlyC. quicklyD. differently
A. an errorB. a problemC. as accidentD. a headache
A. whenB. whyC. howD. what
A. ForB. OrC. ButD. So
第二部分 阅读理解(共25小题,第一节第小题2分,第二节每小题1分;满分45分)
第一节 阅读下列短文,从每题所给的四个选项(A、B、C、D)中,选出最佳选项,并在答题卡上将该项涂黑。
A
Growing up in Philadelphia, Lieberman started cooking with his stay-at-home dad when he was seven. His food-loving family had two kitchens, and he quickly learned what was the best way to bake his cakes. Lieberman improved his kitchen skills greatly during a year abroad before college, learning from a cook in Italy and studying local specialties(地方特色菜)in Germany, Spain and France. At Yale, he was known for throwing dinner parties, single-handedly frying and baking while mixing drinks for dozens of friends. Just for fun, he and some friends decided to tape a show named Campus Cuisine about his cooking. Lieberman was a real college student showing his classmates how to do things like make drinks out of dining-hall fruit. That helped the show become very popular among the students. They would stop Lieberman after classes to ask for his advice on cooking. Tapes of the show were passed around, with which his name went beyond the school and finally to the Food Network.
Food Network producer Flay hopes the young cook will find a place on the network television. He says Lieberman’s charisma is key. “Food TV isn’t about food anymore,” says Flay. “It’s about your personality(个性)and finding a way to keep people’s eyeballs on your show.”
But Lieberman isn’t putting all his eggs in one basket. After taping the first season of the new how, Lieberman was back in his won small kitchen preparing sandwiches. An airline company(航空公司)was looking for some one to come up with a tasteful, inexpensive and easy-to-make menu to serve on its flights, Lieberman got the job.
We can learn from the text that Lieberman’s family ____.A. have relatives in EuropeB. love cooking at homeC. often hold partiesD. own a restaurant
The Food Network got to know Lieberman ____.A. at one of his partiesB. from his teachersC. through his taped showD. on a television program
What does the word “charisma” underlined in the text refer to?A. A natural ability to attract others.B. A way to show one’s achievement.C. Lieberman’s after-class interest.D. Lieberman’s fine cooking skill.
Why did the airline company give Lieberman the job?A. He could prepare meals in a small kitchen.B. He was famous for his shows on Food TV.C. He was good at using eggs to make sandwiches.D. He could cook cheap, delicious and simple meals.
What can we learn about Lieberman from the text?A. He is clever but lonely.B. he is friendly and active.C. He enjoys traveling around.D. He often changes his menus.
B
Some people have the feeling that nothing can be done about their poor reading ability(能力). They feel hopeless about it. Can you learn to read better, or must you agree that nothing can be done about it?
To be sure, people are different. You cannot to do everything as well as certain other people do. It al the students in a class tried out for basketball, some would be very good players; others would be very poor; and many would be in between. But even the very poor players can become much better players if they are guided in the right way, and with plenty of practice. It is the same with reading. Some seem to enjoy reading and to read well without any special help. Others find reading a slow and tiring job. In between, there are all degrees of reading ability.
Many experiments have shown that just about every poor reader can improve his reading ability. In these experiments, the poor readers were given tests of reading ability. After some of the causes of their poor reading were discovered, they were given special instruction and practice in reading. After a few months, another test of the same kind was given. In nearly all cases, these people had raised their reading scores.
With the example of basketball players, the author shows ____.A. why certain people are poor readersB. that there are differences in people’s abilitiesC. why some people are good basketball playersD. that good basketball players can be good readers
To improve their reading ability, people should ____.A. work long and hardB. take different forms of testsC. have special help and practiceD. try different reading materials
The experiments mentioned in the text show that ____.A. good readers seem to enjoy readingB. almost all poor readers can make progressC. causes of poor reading are difficult to find outD. tests help people improve their reading ability
C
Odland remembers like it was yesterday working in an expensive French restaurant in Denver. The ice cream he was serving fell onto the white dress of a rich and important woman.
Thirty years have passed, but Odland can’t get the memory out of his mind, nor the woman’s kind reaction(反应). She was shocked, regained calmness and, in a kind voice, told the young Odland. “It’s OK. It wasn’t your fault.” When she left the restaurant, she also left the future Fortune 500 CEO(总裁)with a life lesson: You can tell a lot about a person by the way he or she treats the waiter.
Odland isn’t the only CEO to have made this discovery. Rather, it seems to be one of those few laws of the land that every CEO learns on the way up. It’s hard to get a dozen CEO’s to agree about anything, but most agree with the Waiter Rule. They say how others treat the CEO says nothing. But how others treat the waiter is like a window into the soul.
Watch out for anyone who pulls out the power card to say something like, “I could but this place and fire you,” or “I know the owner and I could have you fired.” Those who say such things have shown more about their character(人品)than about their wealth and power.
The CEO who came up with it, or at least first wrote it down, is Raytheon CEO Bill Swanson. He wrote a best-selling book called Swanson’s Unwritten Rules of Management.
“A person who is nice to you but rude to the waiter, or to others, is not a nice person,” Swanson says. “I will never offer a job to the person who is sweet to the boss but turns rude to someone cleaning the tables.”
What happened after Odland dropped the ice cream onto the woman’s dress?A. He was fired.B. He was blamed.C. The woman comforted him.D. The woman left the restaurant at once.
Odland leaned one of his life lessons from ____.A. his experience as a waiter.B. the advice given by the CEOsC. an article in FortuneD. an interesting best-selling book
According to the text, most CEOs have the same opinion about ____.A. Fortune 500 companiesB. the Management RulesC. Swanson’s bookD. the Waiter Rule
From the text we can learn that ____.A. one should be nicer to important peopleB. CEOs often show their power before othersC. one should respect others no matter who they areD. CEOs often have meals in expensive restaurants
D
It is difficult for doctors to help a person with a damaged brain. Without enough blood, the brain lives for only three to five minutes. More often the doctors can’t fix the damage. Sometimes they are afraid to try something to help because it is dangerous to work on the brain. The doctors might make the person worse if he operates on the brain.
Dr. Robert White, a famous professor and doctor, thinks he knows a way to help. He thinks doctors should make the brain very cold. If it is very cold, the brain can live without blood for 30 minutes. This gives the doctor a longer time to do something for the brain.
Dr. White tried his idea on 13 monkeys. First he taught them to do different jobs, then he operated on them. He made the monkeys’ blood back to the monkeys’ brains. When the brain’s temperature was 10℃, Dr. White stopped the blood to the brain. After 30 minutes he turned the blood back on. He warmed the blood again. After their operations the monkeys were like they had been before. They were healthy and busy. Each one could still do the jobs the doctor had taught them.
The biggest difficulty in operating on the damaged brain is that ____.A. the time is too short for doctorsB. the patients are often too nervousC. the damage is extremely hard to fixD. the blood-cooling machine might break down
The brain operation was made possible mainly by ____.A. taking the blood out of the brainB. trying the operation on monkeys firstC. having the blood go through a machineD. lowering the brain’s temperature
With Dr. White’s new idea, the operation on the damaged brain ____.A. can last as long as 30 minutesB. can keep the brain’s blood warmC. can keep the patient’s brain healthyD. can help monkeys do different jobs
What is the right order of the steps in the operations?a. send the cooled back to the brainb. stop the blood to the brainc. have the blood cooled downd. operate on the brainA. a, b, c, dB. c, a, b, dC. c, b, d, aD. b, c, d, a
E
Most people want to know how things are made. They honestly admit, however, that they hardly know a thing when it comes to understanding how a piece of music is made. Where a composer(作曲家)begins, how he manages to keep going – in fact, how and when he learns his trade – all are covered in complete darkness. The composer, in short, is a man of mystery(神秘).
One of the first things the common man wants to know about is the part inspiration(灵感)plays in a composer’s work. He finds it difficult to believe that composers are not much interested in that question. Writing music is as natural for the composer as eating or sleeping for all. Music is something that the composer happens to have been born for.
The composer, therefore, does not say to himself: “Do I feel inspired?” He says to himself: “Do I feel like working today?” And if he feels like working, he does. It is more or less like saying to himself: “Do I feel sleepy?” if you feel sleepy, you go to sleep. If you don’t feel sleepy, you stay up. If the composer doesn’t feel like working, he doesn’t work. It’s as simple as that.
What would be the best title for the text?A. Composer: a man of mysteryB. Practice makes good musicC. Relation between sleeping and musicD. Music: product of nature
The words “covered in complete darkness” underlined in Paragraph 1 most probably mean ____.A. difficult to be madeB. without any lightC. black in colorD. not known
Most people seem to think that a composer ____.A. finds it difficult to write musicB. considers it important to have a good restC. should like to talk about inspirationD. never asks himself very simple quesiotns
The author will most probably agree that composers ____.A. are born with a gift for musicB. are people full of mysteryC. work late at night for their musicD. know a lot about eating and sleeping
第二节 根据对话内容,从对话后的选项中选出能填入空白处的最佳选项。选项中有两项为多余选项。
-Mary, do you want to see the pictures of my holiday in Italy?
- 61
-Ah, it was great! The food was great! The wine was great! But the traffic was terrible!
- 62
-Those Italians are crazy drivers! I don’t want to think about it!
-Yes, so here’s a picture of the Tower of Pisa.
-How nice!
-It was raining that day, but it was still wonderful. We climbed to the top!
- 64
-That’s a photo of the Arno River. That’s the “Ponte Vecchio”, the old bridge.
- 65
-It was very interesting. There were beautiful old buildings in the city, and lots of wonderful museums.
-That’s nice.
And what’s this?
Why was it so bad?
Yes, it was wonderful.
What was Florence like?
Let’s return to the good parts.
Sure, what was your holiday like?
Well, did you like your hotel there?
第二卷(非选择题)
第三部分 写作(共三节,满分分)
单词拼写(共小题;每小题分,满分分)
根据下列句子及所给汉语注释,在句子右边的横线上写出空缺处各单词的正确形式。(每空只写一词)
There’s a ____66____(留言)from Karen on the phone.
They football game was ____67____(播出)live on TV across Europe.
They’re going to ____68____(庆祝)their victory with music and dancing.
The ____69____(大多数)of students find it quite hard to learn German.
Very few people ____70____(成功)in losing weight these days.
The book gives a short ____71____(描述)of the city.
The doctor ____72____(表扬)our daughter for her courage this morning.
There were piles of newspapers ____73____(到处)in the house.
What is your ____74____(最喜欢的)color?
The little girl is wearing a ____75____(粉红色的)dress.
短文改错(共10小题;每小题1.5分,满分15分)
此题要求改正所给短文中的错误。对标有题号的每一行作出判断:如无错误,在该行右边横线上画一个(√);如有错误(每行只有一个错误),则按下列情况改正:
此行多一个词:把多余的词用斜线()划掉,在该行右边横线上写出该词,并也用斜线划掉。
此行缺一个词:在缺词处如一个漏字符号(∧),在该行右边横线上写出该加的词。
此行错一个词:在错的词下划一横线,在该行右边横线上写出改正后的词。
注意:原行没有错的不要改。
Dear Grandpa,
Thank you for your letter,
My school is organizing a basketball team and
there's just a chance which I can join it. I’m little of 76. ________
course, but terribly quick and bravely. While the others 77. ________
are jumping about in the air, I can run under my legs78. ________
and get the ball. It will be lots of fun for practicing out79. ________
in the playground in the afternoon with the tree80. ________
around us all red and yellow and everybody laughing81. ________
and shouting. These are the happier girls I’ve ever 82. ________
seen and I’m the happiest in all!83. ________
I meant to write long letter and tell you all the 84. ________
things I’m doing at school, but the bell was ringing,85. ________
so I just have to stop here.
Love,
Judy
书面表达(满分30分)
一家宾馆新开业,为吸引外国宾客,希望在互联网上进行宣传,请你用英语为其写一篇文字介绍。主要内容就包括:
地点:距白山入口处500米;
房间及价格:单人间(共20间);
100元/天;双人间(共15间);
150元/天;热水淋浴;
餐饮:餐厅(中、西餐),咖啡厅(茶、咖啡);
游泳池:全天免费开放;
欢迎预定。
注意:
词数100左右,开头语已为你写好;
可适当增加细节,以使行文连贯
********************************************
Welcome to Baishan Mountain Hotel
Baishan Mountain Hotel is now open for business.
2007年普通高等学校招生全国统一考试试题卷
英语参考答案
第一部分:
1~5 CBABD6~10 DCABC11~15 BCADB16~20 CADBC
21~25 CABDA26~30 DCBBC31~35 DABCD36~40 DACAB
第二部分:
41~45 BCADB46~50 BCBCA51~55 DCADA56~60 BADCA
61~65 FBEAD
第三部分:
第一节:
66. message67. broadcast/broadcasted68. celebrate
69. majority70. succeed71. description72. praised
73. everywhere74. favo(u)rite75. pink
第二节:
76. which ( that77. bravely ( brave78. my ( their79. 去掉for
80. tree ( trees81. 正确82. happier ( happiest83. in ( of
84. write ∧ long ( write (a) long 85. was ( is
写作:
Welcome to Baishan Mountain Hotel
Baishan Mountain Hotel is now open for business.
Our hotel stands 500 meters away from the entrance to Baishan Mountain. It has 20 single rooms and 15 double rooms, all with hot showers. A single room is 100 yuan and a double room 150 yuan for one night. You are advised to book in advance. The hotel serves three meals a day and there are Chinese food and western food for you to choose from. You can also enjoy yourself at the café drinking tea or coffee in the evening. We also have a swimming pool, which is open all day and free of charge.
All are welcome!
2007年普通高等学校招生全国统一考试(天津卷)
英 语
第一部分:英语知识运用(共两节,满分45分)
1.He didn’t make ________ clear when and where the meeting would be held.
A. this B. that C. it D. these
2.---- Could you turn the TV down a little bit?
---- ________. Is it disturbing you?
A. Take it easy. B. I’m sorry. C. Not a bit D. It depends
3.I wanted to catch ________early train, but couldn’t get ________ ride to the station.
A. an, the B. /, the C. an, / D. the, a
4.________ fire, all exits must be kept clear.
A. In place of B. Instead ofC. In case of D. In spite of
5.Hardly could he ________ this amount of work in such a short time.
A. get through B. get off C. get into D. get down
6.The glass doors have taken the place of the wooden ones at the entrance, ________ in the natural light during the day.
A. to let B. letting C. let D. having let
7.Lucy has ________ all of the goals she set for herself in high school and is ready for new challenges at university.
A. acquired B. finished C. concluded D. achieved
8.It is difficult for us to learn a lesson in life ________ we’ve actually had that lesson.
A. until B. after C. since D. when
9.A new ________ bus service to Tianjin Airport started to operate two months ago.
A. normal B. usual C. regular D. common
10.---- I apologize for not being able to join you for dinner.
---- ________. We’ll get together later.
A. Go ahead B. Not to worry
C. That’s right D. Don’t mention it
11.Those successful deaf dancers think that dancing is an activity ________ sight matters more than hearing.
A. when B. whose C. which D. where
12.One thousand dollars a month is not a fortune but would help cover my living ________.
A. bills B. expenses C. prices D. charges
13.If Newton lived today, he would be surprise by what ____ in science and technology.
A. had discovered B. had been discovered
C. has discovered D. has been discovered
14.The final score of the basketball match was 93-94.We were only ________ beaten.
A. nearly B. slightly C. narrowly D. lightly
15.The seaside here draws a lot of tourists every summer. Warm sunshine and soft sands make ________ it is .
A. what B. which C. how D. where
第二节:完形填空(共20小题,每小题1.5分,满分30分)
阅读下面短文,掌握其大意,然后从16-35各题所给的A.B.C.D四个选项中,选出最佳选项。
Roberta appeared on the stage. She took a deep breath and began to 16 . Now she was Portia, a strong–willed 17 in Shakespeare’s The Merchant of Venice. The theater was filled with people. She was speaking with a power she had never before experienced, the words flowing 18 form her.
19 , Roberta had never acted in her life before the audition (选拔试演). She 20 being in front of other people. She was very 21 at school. She had never thought she was good enough at anything to 22 much attention. She stayed mostly to herself, making 23 friends. She had excellent grades, 24 she always thought that something was missing.
Two weeks before the audition, Robertsa’s mother had heard about it and 25 her to join in.
“I can’t think of anyone else better suited to 26 the part. Remember all the plays you used to act our for us?”
Her mother wouldn’t let the 27 drop. “You’re just a little scared (害怕) .Everyone gets scared. You know you 28 do it. The trick is to look past the 29 to find the love of what you’re doing.’’
So Roberta had made an appointment (预约) with the head of the Drama Club. She had read the play and found herself excited by the 30 of speaking such rich words. In secret she practiced Portia’s part, 31 the lines by repeating them over and over. It wasn’t hard; she 32 every minute of it. Every time she spoke the words, she had a new 33 of the lines, as if Shakespeare had written Portia on many levels.
On the day of the audition, she 34 two of Portia’s famous speeches for the auditors. When she had finished, the head of the Drama Club announced the 35 was hers.
16.A. sing B. dance C. speak D. report
17.A. member B. actress C. player D. character
18.A. weakly B. rapidly C. smoothly D. slowly
19.A. At first B. In fact C. After all D. In all
20.A. hated B. enjoyed C. appreciated D. regretted
21.A. honest B. shy C. polite D. patient
22.A. avoid B. focus C. pay D. attract
23. A. few B. a few C. several D. many
24. A. or B. so C. for D. but
25. A. forced B. requested C. encouraged D. reminded
26. A. accept B. play C. offer D. learn
27. A. role B. matter C. interest D. grade
28. A. can B. must C. may D. should
29. A. anger B. pain C. sadness D. fear
30. A. purpose B. way C. idea D. importance
31. A. memorizing B. organizing C. checking D. improving
32. A. disliked B. loved C. expected D. bore
33. A. consideration B. description C. selection D. understanding
34. A. practiced B. planned C. performed D. delivered
35.A. part B. play C. speech D. position
第二部分:阅读理解(共20小题;每小题2分,满分40分)
阅读下列短文,从每题所给的A、B、C、D四个选项中,选出最佳选项。
A
The city of Rome has passed a new to prevent cruelty to animals. All goldfish bowls are no longer allowed and dog owners must walk their dogs.
This comes after a national law was passed to give prison sentences to people who desert cats or dogs.
“The civilization of a city can be measured by this,” said Monica Carina, the councilor (议员) behind the new law.
“It’s good to do whatever we can for our animals who in exchange for a little love fill our existence with their attention,” she told a Rome newspaper.
The newspaper reported that round bowls don’t give enough oxygen for fish and may make them go blind.
“Rome has tried to protect fish more than anywhere else in the world .It stands out for recognizing that fish are interesting animals who deserve(值得)over respect and compassion every bit as much as dogs and cats and other animals,” said Karin Robertson, a director of the People for the Ethical Treatment of Animals.
Last year a law was passed in Italy that gives people who desert pets big fines (罚款)and prison sentences. Since then local governments have added their own animal protection rules.
The northern city of Turin passed a law in April to give pet owners fines of up to $598 if they do not walk their dogs three times a day.
The new law in Rome also says that owners mustn’t leave their dogs in hot cars or cut their dogs’ tails to make them look lovelies. The law also gives legal recognition to the “cat ladies” who feed homeless cats. The cats live all over the city from ancient ruins to modern office car parks.
36. The new law passed in Rome will ________.
A. help improve fishing environment
B. guarantee better conditions for goldfish
C. stop people from catching goldfish
D. discourage keeping goldfish at home
38.The underlined word “compassion” in Paragraph 6 is the closest in meaning to ________.
A. pityB. praiseC. supportD. popularity
39.People may break the law in Turin if they ________.
A. keep their dogs or cats in cars
B. feed homeless animals in car parks
C. raise their cats near ancient ruins
D. shut their dogs home all day long
B
Charles Blackman:Alice in Wonderland
An Exhibition at the National Gallery of Victoria (NGV),Australia
June —12 August 2007
Venue (地点) The Ian Potter Centre
Admission Free entry
Charles Blackman is famous for his beautiful painting of dreams. In 1956, he heard for the first time Lewis Carroll’s extraordinary tale of Alice in Wonderland –the story of a Victorian girl who falls down a rabbit hole, meets a lot of funny characters and experience all kinds of things .At that time, Blackman’s wife was suffering form progressive blindness. The story of Alice moving through the strange situations, often disheartened by various events ,was similar to his wife’s experiences. It also reflected so much of his own life. All this contributed to the completion of the Alice in Wonderland paintings.
Illustrator Workshop
Go straight to the experts for an introductory course in book illustration. The course includes an introduction to the process of illustration and its techniques, workshop exercise and group projects.
Dates Sunday 17 June &Sunday 5Aug.10am—1pm
Venue Gas Works Arts Park
Wonderful World
Celebrate the exhibition and Children’s Book Week with special activities just for the day, including a special visit from Alice and the White Rabbit
Date Sunday 24 June ,11am—4pm
Venue Exhibition Space .Level 3
Topsy-Turvy
Visit the exhibition or discover wonderful curiosities in artworks in the NGV Collection and make a magic world in a box. Alice and the White Rabbit will be with you. Walt Disney’s Alice in Wonderland will be screened.
Dates Sunday 8,15,22,29 July, and Tuesday 24-Friday 27 July, 12noon—3pm
Venue Theatre, NGV Australia
Drawing Workshop
Distortions of scale ( 比例失真) can make artworks strange but interesting .Find out how Charles Blackman distorted scale in his paintings to create a curious world .then experiment with scale in your own drawings. More information upon booking.
Date Friday 27 July,10:30am-3pm
Venue Foryer, Level 3
40. Charles Blackman’s paintings come from ________.
A. his admiration for Lewis Carrioll
B. his dream of becoming a famous artist
C. his wish to express his own feelings
D. his eagerness to cure his wife’s illness
41.Which two activities can you participate in on the same day?
A. Illustrator Workshop and Wonderful World
B. Illustrator Workshop and Drawing Workshop.
C. Wonerful World and Topsy –Turvy.
D.Topsy-Turvy and Drawing Workshop.
42. To understand the Alice in Wonderland paintings, you should go to ________.
A. Exhibition Space .Level 3B. Gas Works Arts Park
C. Theatre ,NGV AustraliaD. Foyer, Level 3
43.Activities concerning children’s books are to be held
A. on June 24,2007 B. on July15,2007
C. on July 24,2007 D. on August 5,2007
C
I recently turned fifty, which is young for a tree, midlife for an elephant ,and ancient for a sportsman. Fifty is a nice number for the states in the US or for a national speed limit but it is not a number that I was prepared to have hung on me. Fifty is supposed to be my father’s age, but now I am stuck with this number and everything it means.
A few days ago, a friend tried to cheer me up by saying, “ Fifty is what forty used to be .”He had made an inspirational point ,Am I over the hill ?People keep telling me that the hill has been moved, and I keep telling them that he high-jump bar has dropped from the six feet I once easily cleared to the four feet that is impossible for me now.
“ Your are not getting older, you are getting better.” says Dr. Joyce Brothers. This, however, is the kind of doctor who inspires a second opinion.
And so .as I approach the day when I cannot even jump over the tennis net. I am moves to share some thoughts on aging with you. I am moved to show how aging feels to me physically and mentally. Getting older. of course, is obviously a better change than the one that brings you eulogies(悼词).In fact ,a poet named Robert Browning considered it the best change of all:
Grow old along with me!
The best is yet to me.
Whether or not Browning was right, most of my first fifty years have been golden ones, so I will settle for what is ahead being as good as what has gone by. I find myself moving toward what is ahead with a curious blend (混合) of both fighting and accepting my aging, hoping that the philosopher (哲学家) was right when he said .”Old is always fifteen years from now.”
44. The author seems to tell us in Paragraph 1 that ________.
A. time alone will tell B. time goes by quickly
C. time will show what is right D. time makes one forget the past
45. When the author turned fifty ,people around him ________.
A. tried to comfort him B. got inspiration with him
C. were friendlier with him D. found him more talkative
46. The author considers his fifty years of life
A. peaceful B. ordinary C. satisfactory D. regretful
47. We can infer from the passage that
A. the old should led a simple life
B. the old should face the fact of aging
C. the old should take more exercise
D. the old should fill themselves with curiosity
D
In meditation(冥想),people sit quietly and focus their attention on their breath .As they breathe in and out ,they attend to their feelings .As thoughts go through their minds. they let them go. Breathe. Let go. Breathe. Let go.
According to a recent study at the Insight Meditation Society in Barre, Massachusetts, three months of training in this kind of meditation causes a market change in how the brain allocates(分配) attention. It appears that the ability to let go thoughts that come into mind frees the brain to attend to more rapidly changing things and events in the outside world. Expert mediators are better than other people at catching such fast-changing stimuli( 刺激),like facial expressions.
The study provides evidence for changes in the workings of the brain with mental training. People can learn and improve abilities of all sorts with practice. everything from driving to playing the piano. The study has shown that meditation is good for the brain ,It appears to reduce pressure and promote a sense of well-being.
In an experiment ,17 volunteers with no meditation experience in the experimental group spent three months meditating 10 to 12 hours a day .A control group also with no meditation experience meditated for 20 minutes a day over the same period .Both groups were then given the tests with two numbers in a group of letters. As both group looked for the numbers ,their brain activity was recorded.
Everyone could catch the first number .But the brain recordings showed that the less experienced mediators tended to grasp the first number and hang onto it ,so they missed the second number .Those with more experience gave less attention to the first number .as if letting it go ,which led to an increased ability to grasp the second number ,This shows that attention can change with practice.
Just ask Daniel Levision, who meditated for three months as part of the study .”I am a much better listener,” he said. “I do not get lost in my own personal reaction to what people are saying.”
48.The underlined word “them” in Paragraph 1 refers to ________.
A. feelings B. minds C. people D. thoughts
49.Meditations manage their daily tasks better because they ________.
A. are given less pressure B. allocate their attention better
C. have more stimuli for life D. practice them more frequently
50. In the experiment ,volunteers doing meditation for longer hours ________.
A. were more likely to catch both of the members
B. were used to memorizing numbers in groups
C. usually ignored the first number observed
D. paid more attention to numbers than to letters
51. The study proves that ________.
A. meditation improves one’s health
B. brain activity can be recorded
C. human attention can be trained
D. mediators have a good sense of hearing
E
When we think of leadership ,we often think of strength and power .But what are these really ,and how do they operate?
Leadership today is not about forcing others to do things. If this is even possible, it is short-term, and tends to backfire. If you order someone to do something against their will, they may do it because they feel they must ,but the anger they feel will do more harm in the long-term ,They will also experience fear.
Fear causes the thinking brain to shut down, making the person unable to fun_ction at his or her best .If they associate you with this emotion of fear. They will become less functional around you ,and you will have succeeded in not only shooting yourself in the foot .but possibly making a very good employee or partner unable to perform effectively .Fear has no place in leadership.
The way we influence people in a lasting way is by our own character, and our understanding and use of emotion .We can order someone to do something ,which may be part of the work day; or we can employ them at the emotional level ,so they became fully devoted to the projects and provide some of their own motivation (积极性). Today’s work place is all about relationships
Anyone works harder in a positive environment in which they are recognized and valued as a human being as well as a worker.. Everyone produces just a bit more for someone they like. Leaders understand the way things work .They know the pay check is not the single most motivating factor (因素) in the work life of most people.
The true strength of leadership is an inner strength that comes from the confidence of emotional intelligence---knowing your own emotions , and how to handle them ,and those of others . Developing your emotional intelligence is the single best thing you can do if you want to develop your relationships with people around you, which is the key to the leadership skills.
52. An employee may have a feeling of fear in the work place when ________.
A. he is forced to do things
B. be cannot work at his best
C. he feels his brain shut down
D. he thinks of his work as too heavy
53. Which of the following is TRUE according to the passage?
A. People tend to associate leadership with fear
B. Working conditions affect people’s physical health
C. Good relationship is the key to business success
D. Smart people are more functional in the work place.
54.To positively influence employees a leader should first of all ________.
A. provide better suggestionsB. develop his own personality
C. five his employees a pay raiseD. hide his own emotion of fear
55.Good leadership is mainly seen in a leader’s ability to ________.
A .provide a variety of project for employees
B. help raise employee’s living standards
C. give employees specific instructions
D. deal wisely with employees’ emotions
第三部分:写作
第一节:短文改错(共10小题;每小题1分,满分10分)
此题要求改正所给短文中的错误没。对标有题号的每一行做出判断;如有错误,在该行右边横线上画一个勾;如有错误(每行只有一个错误),则按下列情况改正:
此行多一个词;把多余的词用斜线()划掉,在该行右边横线上写出该词,并也用斜线划掉。
此行缺一个词:在却此处加一个漏字符号(∧),在该行右边横线上写出该加的词。
此行错一个词:在错的词下画一横线,在该行右边横线上写出改正後的词。
In Grand Eight I took physics. In one test I get 56. __________
only 36 percent of the answer correct. I failed the 57. __________
next one, either. I started to think that maybe I was 58. __________
not good at it. However, I was not lucky enough to 59. __________
have a teacher which didn’t take my bad grades as a 60. __________
judgement of my abilities, but simply like an indication 61. __________
how I should study harder. He pulled me aside and told 62. __________
me that he knew I could do better. He permitted to retake 63. __________
the test, and I was pulled my grade to an A. This is what 64. __________
I discovered: just because a subject is difficulty to learn. 65. __________
It doesn’t mean you are not good at it.
第二节 书面表达(满分25分)
66.
假设你校举办看图征文活动,题目为“How to Protect Our Eyesight”。请你用英语写一篇短文,指出图中所反映的现象,并从阅读时间、光线、眼睛保健及睡眠等方面,谈谈保护视力的具体方法。
注意:
1.词数:不少于100词;
2.可适当加入细节,以便使行文连贯。
参考词汇:
近视 — short-sighted
How to Protect Our Eyesight
____________________________________________________________________________________________________________________________________________________________________________________________________________________________________________________________________________________________________________________________________________________________________________________________________________________________________________________________________________________________________________________________________________________________________________________________________________________________________________________________________________________________________________________________________________________________________________________________________________________________________________________________________
第四部分:听力
第一节(共5小题;每小题1.5分,满分7.5分)
听下面5段对话。每段对话后有一个小题,从题中所给的A、B、C三个选项中选出最佳选项,并标在试卷的相应位置。听完每段对话后,你将有10秒钟 的时间来回答有关小题和阅读下一小题。每段对话仅读一遍。
67. When does the man’s plane take off?
A. At 5:00. B. At 5:30. C: At 7:00.
68. What do we learn about the man ?A. He is sleepy.B. He hurt his eyes.C. He failed his exam.
69. How often was the magazine published in the past?
A. Once a week. B. Once a month. C. Twice a month.
70. What are the two speakers talking about?
A. Weather forecast.B. The view of a lake. C. As out door activity.
71. Why did not the woman meet the man for lunch ?
A. She was having a class.
B. She did not like the crowd.
C. She was not feeling well.
第二节 (共15小题,每小题1.5分,满分 22.5)
听下面5段材料,每段材料后有几个小题。从题中所给的ABC三个选项中选出最佳选项,并标在试卷的相应位置,听每段材料前,你将有时间阅读各个小题。每小题5秒钟。听完后,各小题将给出5秒钟的作答时间,每段材料读两遍。
听下面一段对话,回答第72和第73小题。
72. What is the conversation about?
A. Offering a lift. B. Buying a computer. C. Asking for a favor.
73. What is the man doing when the woman calls?
A. He is moving a box.
B. He is working upstairs.
C. He is waiting for the delivery.
听下面一段对话,回答第74至第76小题。
74. What happened to the woman is purse?
A. It was missing. B. It was left home. C. It was stolen.
75. What did the woman go to the bank for?
A. To pay the bill. B. To cash the cheque.C. To open an account.
76. Which is the right order of the places the speakers have been to ?
A. Coffee shop—bookshop—bank.
B. Bookshop —-bank—coffee shop.
C. Bank—coffee shop—bookshop.
听下面一段对话,回答第77至第79小题。
77. Who supported the woman financially for her studies at nurse school?
A. Her parents.B. The government.C. The woman herself.
78. What do we learn about the woman’s parents?
A. They respected her choice.
B. They treated her as a child.
C. They controlled her too much.
79. What is TRUE about the man ?
A. He was forced to study abroad.
B. He is longing for more freedom.
C. He is open to persuasion.
听下面一段对话,回答第80 至第82小题。
80.What attracts the woman most at the store?
A. Its high –quality service.
B. Its good range of products.
C. Its nice shopping environment.
81. Which of the following products are mentioned in the conversation?
A. Cupboards, beds and armchairs
B. Armchairs, dinner tables and bed sheets.
C. Bookcases, cupboards and table cloths
82. What dose not woman like about the store?
A. No sales promotion.
B. Crowds at the checkouts.
C. High prices of the products.
听下面一段独白,回答第83题至86小题。
83. How did Europeans know the exact time in the late 1700s?
A. By looking at clocks at home.
B. By looking at clocks in factories.
C. By looking at clocks in public places.
84.When did people regard watches as a sign of wealth in Europe and the US?
A. In the 1800s.B. In the 1850s.C. In the 1900s.
85. Which would be the best title for the passage?
A. The development of watches and clocks.
B. The functions of watches and clocks.
C. The importance of watches and clocks.
2007年普通高等学校招生考试全国统一考试(天津卷)
英语参考答案
1—10 CBDCABDACB 11-20 DBDCACDCBA 21-30 BDADCBBADC
31-40 ABDCABCADC 41-50 DDABACBDBA 51-55 CACBD
In Grade Eight I took physics. In one test I get 56. got
only 36 percent of the answer correct. I failed the 57. answers
next one .either. I started to think that maybe I was 58. too
not good at it. However, I was not lucky enough to 59. not(去掉)
have a teacher which did not take my bad grades as a 60. who/that
judgment of my abilities, but simply like an indication 61. as
how I should study harder. He pulled me aside and told 62. that
me that he knew I could do better. He permitted ∧ to retake 63. ∧ me
the test ,and I was pulled my grade to an A. This is what 64. was (去掉)
I discovered: just because a subject is difficulty to learn, 65. difficult
It does not mean you are not good at it.
67---76 CABCACBABC 77—86 BABBCBCACA
2007年普通高等学校招生全国统一考试英语招生试题
安 徽 卷
第Ⅰ卷
第一部分:听力(共两节。满分30分)
做题时,先将答案标在试卷上。录音内容结束后,你将有两分钟的时间将试卷上的答案转涂到答题卡上。
第一节(共5小题;每小题1.5分,满分7.5分)
听下面5段对话。每段对话后有一个小题,从题中所给的A、B、C三个选项中选出最佳选项,并标在试卷的相应位置。听完每段对话后,你都有10秒钟的时间来回答有关小题和阅读下一小题。每段对话仅读一遍。
例: How much is the shirt?
A. £9.15. B. £9.15. C. £9.18.
答案是 B。
1. Who is coming for tea?
A. John. B. Mark. C. Tracy.
2. What will the man do next?
A. Leave right away. B. Stay for dinner. C. Catch a train.
3. What does the man come for?
A. A lecture. B. A meeting. C. A party.
4. What size does the man want?
A. 9. B. 35. C. 39.
5. What are the speakers talking about?
A. Life in Southeast Asia. B. Weather conditions. C. A holiday tour.
第二节(共15小题;每小题1.5分,满分22.5分)
听下面5段对话。每段对话后有几个小题,从题中所给的A、B、C三个选项中选出最佳选项,并标在试卷的相应位置。听每段对话前,你将有时间阅读各个小题,每小题5秒钟;听完后,各小题将给出5秒钟的作答时间。每段对话读两遍。
听第6段材料,回答第6、7题。
6. What is the man doing?
A. Giving a speech. B. Chairing a meeting. C. Introducing a person.
7. Why does the woman sing so well?
A. She has a great teacher. B. She teaches singing. C. She is young.
听第7段材料,回答第8、9题。
8. What is the second gift for Jimmy?
A. A car. B. A watch. C. A computer.
9. Why does Jimmy feel happy?
A. He lives with his parents. B. He's got what he dreamt of.
C. He's received lots of presents.
听第8段材料,回答第10、12题。
10. What is the relationship between the speakers?
A. They are friends. B. They are strangers to each other. C. They are husband and wife.
11. Why does the woman come to talk with the man?
A. To get a job. B. To take a test. C. To see the secretary.
12. What does the man mean by saying sorry?
A. He can' t hear the woman clearly. B. He doesn't need a designer.
C. He can' t help the woman.
听第9段材料,回答第13、16题。
13. What do we know about the woman?
She lives close to the office. B. She is new to the company.
C. She likes the big kitchen.
14. How does the man go to work?
A. On foot. B. By bus. C. By car.
15. Why was Susan late for work?
A. She missed the bus. B. Her train was late. C. Her ear broke down.
16. What will the man do the next day?
A. Go to work by train. B. Visit Lily in her flat. C. Leave home earlier.
听第10段材料,回答第17、20题。
17. Where can you most probably hear this talk?
A. In a class of the English language. B. In a class of the Greek language.
C. In a class of the French language.
18. How long does the class last?
A. 11 weeks. B. 13 weeks. C. 15 weeks.
19. What is "the short-cut" to learning words according to the speaker?
A. Taking more courses. B. Reading basic words aloud.
C. Learning how words are formed.
20. Why is the class popular?
A. It is not offered each term. B. It' s taught by Professor Morris.
C. It helps to master some useful rules.
第二部分:英语知识运用(共两节.满分45分)
第一节:语法和词汇知识(共15小题;每小题1分,满分15分)
从A、B、C、D四个选项中,选出可以填入空白处的最佳选项,并在答题卡上将该项涂黑。
例: It is generally considered unwise to give a child ______ he or she wants.
A. however B. whatever C. whichever D. whenever
答案是 B。
21. The field research will take Joan and Paul about five months; it will be a long time _____ we meet them again.
A. after B. before C. since D. when
22. Little _______ that we were watching his every move, so he seemed to be going his own way in this business.
A. he realized B. he didn't realizeC. didn't he realize D. did he realize
23. The teacher __ have thought Johnson was worth it or she wouldn't have wasted time on him, I suppose.
A. should B. can C. would D. must
24. -- Was Martin sorry for what he' d done?
-- ___________. It was just like him!
A. Never mind B. All right C. Not really D. Not surprisingly
25. In my opinion, life in the twenty-first century is much easier than ________.
A. that used to be B. it is used toC. it was used to D. it used to be
26. They became friends again that day. Until then, they _______ to each other for nearly two years.
A. didn't speak B. hadn't spoken C. haven' t spoken D. haven' t been speaking
27. -- Didn't you have a good time at the party?
-- Of course I did. As a matter of fact, I had such fun that time seemed to _______ so quickly.
A. go by B. go away C. go out D. go over
28. The school' s music group will be giving a big show tomorrow night and two on the weekend.
A. more B. other C. else D. another
29. John received an invitation to dinner, and with his work ________, he gladly accepted it.
A. finished B. finishing C. having finished D. was finished
30. -- Robert is indeed a wise man.
-- Oh, yes. How often I have regretted ______ his advice!
A." to take B. taking C. not to take D. not taking
31. -- Bill, if it doesn't rain, we can go straight on and spend more time in Vienna.
-- I just want to hear Mozart.
A. Well done ! B. No problem. C. That' s great ! D. That' s it.
32. The practice of hanging clothes across the street is a common ______ in many parts of the city.
A. look B. sign C. sight D. appearance
33. You can only be sure of you have at present; you cannot be sure of something _____ you might get in the future.
A. that; what B. what; / C. which; that D. /; that
34. Last week, only two people came to look at the house, _______ wanted to buy it.
A. none of them B. both of them C. none of whom D. neither of whom
35. -- Did Peter fix the computer himself?
-- He ________, because he doesn't know much about computers.
A. has it fixed B. had fixed it C. had it fixed D. fixed it
第二节:完形填空(共20小题;每小题1.5分,满分30分)
阅读下面短文,从短文后所给各题的四个选项(A、B、C和D)中,选出可以填入空白处的最佳选项,并在答题卡上将该项涂黑。
It was the night of the full moon, a time which always drives Java' s young people mad with excitement.
Fireworks were lit long before the moon 36 . The big noise brought people out 37 the warm night to enjoy the interesting scene. Everywhere, there were the paper remains of 38__ fireworks lying on the ground. Little boys 39 more and covered their ears as they waited _ 40__ for the explosions.
The moon appeared above the horizon(地平线) : huge, 41 ball high above the city, and the __42 filled with people, as Java began to enjoy one of the year' s greatest _ 43 _ : ' the Night of the Full Moon' , a festival(节日) that is especially popular 44_ young people.
More and more young Javanese 45 together and walked slowly through the 46 Joking and chatting, they moved towards the mountain _ 47 the city. They continued to climb 48 they reached the old temple( 寺庙 ) at the 49 of the mountain.
After they were _ 50 the temple, they drank their water and ate their moon-cakes -- delicious home-made ones, 51 of dried fruit and nuts. Outside, on the mountain, young people 52__ cross-legged in circles, chatting and telling each other jokes. And 53 , in their hundreds, more young people continued to make their way up the mountain to 54 the brightly shining moon.
By midnight, the fireworks had stopped shooting up from the 55 city in the valley below them. But during the night, the sound continued to be heard from the distance.
36. A. let outB. gave outC. came outD. set out
37. A. intoB. atC. ofD. from
38. A. burningB. usedC. explodingD. broken
39. A. litB. boughtC. piledD. removed
40. A. patientlyB. calmlyC. worriedlyD. excitedly
41. A. silverB. newC. colorfulD. gold
42. A. mountainsB. valleysC. streetsD. shops
43. A. gamesB. meetingsC. sportsD. events
44. A. forB. toC. withD. in
45. A. dancedB. gatheredC. drankD. shouted
46. A. villageB. sceneC. nightD. ground
47. A. on the edge ofB. on the way toC. in the center ofD. in the direction of
48. A. whileB. untilC. unlessD. though
49. A. tipB. backC. topD. bottom
50. A. insideB. nearC. offD. across
51. A. fondB. littleC. fullD. free
52. A. jumpedB. satC. stoodD. bent
53. A. soB. evenC. yetD. still
54. A. followB. showC. noticeD. admire
55. A. cleanB. grayC. peacefulD. empty
第三部分:阅读理解 (共20小题;每小题2分, 满分40分)
阅读下列短文,从每题所给的四个选项(A、B、C和D)中,选出最佳选项,并在答题卡上将该项涂黑。
A
Many facts suggest that children are overweight and the situation is getting worse, according to the doctors. I feel there are a number of reasons for this.
Some people blame the fact that we are surrounded (包围) by shops selling unhealthy, fatty foods, such as fried chicken and ice cream, at low prices. This has turned out a whole generation of grown-ups who seldom cook a meal for themselves. If there were fewer of these restaurants, then probably children would buy less take-away food.
There is another argument that blames parents for allowing their children to become overweight. I agree with this, because good eating habits begin early in life, long before children start to visit fast food shops. If children are given fried chicken and chocolate rather than healthy food, or are always allowed to choose what they eat, they will go for sweet and salty foods every time, and this will carry on throughout their lives.
There is a third reason for this situation. Children these days take very little exercise. They do not walk to school. When they get home, they sit in front of the television or their computers and play computer games. Not only is this an unhealthy pastime(消遣), it also gives them time to eat more unhealthy food. What they need is to go outside and play active games or sports.
The above are the main reasons for this problem, and therefore we have to encourage young people to be more active, as well as steering them away from fast food shops and bad eating habits.
56. According to the text, what kind of children may eat more unhealthy food?
A. Those who often take exercise. B. Those who often watch television.
C. Those who often have meals at home. D. Those who often walk to school.
57. The author thinks that children are becoming overweight because
A. their parents often cook meals for themB. they are too busy to go out and play
C. they can' t choose what to eat D. there are too many fast food shops around
58. The word "steering" underlined in the last sentence most probably means _______.
A. forcing B. guiding C. driving D. moving
59. The main purpose of the text is to _________.
A. tell a story B. provide facts C. give advice D. compare opinions
B
King's College Summer School
King's College Summer School is an annual( 每年的) training program for high school students at all levels who want to improve their English. Courses are given by the teachers of King's College and other colleges in New York. Trips to museums and culture centers are also organized. This year's summer school will be from July 25 to August 15.
More information is as follows:
Application (申请) date
Students in New York should send their applications before July 18, 2007.
Students of other cities should send their applications before July 16, 2007.
Foreign students should send their applications before July 10, 2007.
Courses
English Language
Spoken English: 22 hours
Reading and Writing: 10 hours
American History: 16 hours
American Culture: 16 hours
Steps
A letter of self-introduction
A letter of recommendation(推荐)
﹡ The letters should be written in English with all the necessary information.
Cost
Daily lessons: $200
Sports and activities: $100
Travels: $200
Hotel service: $400
﹡You may choose to live with your friends or relatives in the same city.
Please write to:
Thompson, Sanders
1026 King' s Street
New York, NY 10016, USA
E-mail: KC-Summer-School@ yahoo, com
60. You can most probably read the text in ________.
A. a newspaper B. a travel guide C. a textbook D. a telephone book
61. Which of the following is true about King' s College Summer School?
A. Only top students can take part in the program.
B. King' s College Summer School is run every other year.
C. Visits to museums and culture centers are part of the program.
D. Only the teachers of King' s College give courses.
62. If you are to live with your relatives in New York, you will have to pay the school __
A. $200 B. $400 C. $500 D. $900
63. What information can you get from the text?
A. The program will last two months. B. You can write to Thompson only in English.
C. As a Chinese student, you can send your application on July 14, 2007.
D. You can get in touch with the school by e-mail or by telephone.
C
As societies develop, their members start to see things not so much according to what they need, but according to what they want. When people have enough money, these wants become demands.
Now, it' s important for the managers in a company to understand what their customers want if they are going to develop effective marketing strategies (策略). There are various ways of doing this. One way at supermarkets ( 超市), for example, is to interview(采访) customers while they're doing their shopping. They can be asked what they prefer to buy and then the results of the research can be studied. This provides information on which to base future marketing strategies. It' s also quite normal for top managers from department stores to spend a day or two each month visiting stores and mixing freely with the public, as if they were ordinary customers, to get an idea of how customers act.
Another way to get information from customers is to give them something. For example, some fast food restaurants give away tickets in magazines or on the street that permit customers to get part of their meal for nothing. As well as being a good way of attracting customers into the restaurants to spend their money, it also allows the managers to get a feel for where to attract customers and which age-groups to attract.
Another strategy used at some well-known parks such as Disneyland is for top managers to spend at least one day in their work, touting the park dressed as Mickey Mouse or something like that. This provides them with a perfect chance to examine the scene and watch the customers without being noticed.
64. The text is designed for _______
A. mangers B. salesmen C. researchers D. customers
65. Which of the following can help managers get useful information?
A. Visiting customers themselves. B. Giving customers free food on the street.
C. Visiting parks as ordinary customers. D. Asking customers questions at supermarkets.
66. What does the word "this" underlined in the last sentence refer to?
A. Visiting Disneyland. B. Wearing attractive clothes.
C. Acting Mickey Mouse. D. Dressing up and walking around.
67. The main idea of the text is _________.
A. how to do market research B. how to develop marketing strategies
C. how to find out customers' social needs
D. how to encourage customers to spend more money
D
The summer I was ten, my mother decided to bring us to the world of art. My brother and I were not very excited when we realized what my mother meant. What she meant was not that we could take drawing classes or painting classes but that we would have to spend one afternoon a week with her at the Fine Arts Museum. Before each visit to the museum, she made us read about artists and painting styles(风格). It was almost as bad as being in school. Who wants to spend the summer thinking about artists when you could be with your friends at the swimming pool?
First we had to read about ancient Egyptians(古埃及人) and their strange way of painting faces and then go to look at them at the museum. My 12-year-old brother thought this was so funny, but I was not interested. Later we had to learn about artists in the Middle Ages who painted people wearing strange long clothing. We had to look at pictures of fat babies with wings and curly (鬈曲的) hair and with no clothes on flying around the edges of paintings. I certainly couldn't see what was so great about art.
On our last visit to the museum, something happened when I saw a painting by a woman called Mary. In it, a woman was reading to a child. The colors were soft and gentle, and you could tell by the mother' s expression how happy she was just to be with the child. I couldn't stop looking at this painting ! I wanted to see every painting Mary had ever made! It was really worth looking at so many paintings to find a painter who could interest me so much.
68. The aim of the mother' s plan was to _________.
A. take them to visit the museum B. introduce them to the world of art
C. ask them to read about artists D. show them different painting styles
69. What was the writer' s experience in the museum before the last visit?
A. She came to feel her mother' s love. B. She liked many paintings.
C. She hardly enjoyed herself. D. She could understand the pictures of fat babies.
70. What made the writer go through a change that summer?
A. One of Mary's paintings. B. A strange way of painting.
C. Artists in the Middle Ages. D. Her mother' s instruction.
71. From the text, we can see _________.
A. the importance of curiosity B. the effect of art
C. the value of learning D. the power of family education
E
Meeting people from another culture can be difficult. From the beginning, people may send the wrong signal (信号). Or they may pay no attention to signals from another person who is trying to develop a relationship.
Different cultures emphasize (强调) the importance of relationship building to a greater or lesser degree. For example, business in some countries is not possible until there is a relationship of trust. Even with people at work, it is necessary to spend a lot of time in "small talk", usually over a glass of tea, before they do any job. In many European countries -- like the UK or France -- people find it easier to build up a lasting working relationship at restaurants or caf6s rather than at the office.
Talk and silence may also be different in some cultures. I once made a speech in Thailand. I had expected my speech to be a success and start a lively discussion; instead there was an uncomfortable silence. The people present just stared at me and smiled. After getting to know their ways better, I realized that they thought I was talking too much. In my own culture, we express meaning mainly through words, but people there sometimes feel too many words are unnecessary.
Even within Northern Europe, cultural differences can cause serious problems. Certainly, English and German cultures share similar values; however, Germans prefer to get down to business more quickly. We think that they are rude. In fact, this is just because one culture starts discussions and makes decisions more quickly.
People from different parts of the world have different values, and sometimes these values are quite against each other. However, if we can understand them better, a multicultural environment (多元文化环境) will offer a wonderful chance for us to learn from each other.
72. In some countries, eating together at restaurants may make it easier for people to _______.
A. develop closer relations B. share the same culture
C. get to know each other D. keep each other company
73. The author mentions his experience in Thailand to show that _________.
A. the English prefer to make long speeches B. too many words are of no use
C. people from Thailand are quiet and shy by nature
D. even talk and silence can be culturally different
74. According to the text, how can people from different cultures understand each other better?
A. By sharing different ways of life. B. By accepting different habits.
C. By recognizing different values. D. By speaking each other' s languages.
75. What would be the best title for the text?
A. Multicultural Environment. B. Cross-Cultural Differences.
C. How to Understand Each Other. D. How to Build Up a Relationship.
第Ⅱ卷
第四部分:写作(共两节。满分35分)
第一节短文改错(共10小题;每小题1分,满分10分)
此题要求改正所给短文中的错误。对标有题号的每一行作出判断:如无错误,在该行右边横筑上画一个勾(√);如有错误(每行只有一个错误),则按下列情况改正:
此行多一个词:把多余的词用斜线(\)划掉,在该行右边横线上写出该词,并也用斜线划掉。
此行缺一个词:在缺词处加一个漏字符号(∧),在该行右边横线上写出该加的词。
此行错一个词:在错的词下划一横线,在该行右边横线上写出改正后的词。
注意:原行没有错的不要改。
I' m eighteen years old and I live in small village in 76.__________
the Peak District, not far from Nottingham. I grow up 77. __________
in this village, so I know everyone here. That' s the 78. __________
problem -- the problem is that there' s nothing to do, 79. __________
and nobody of my age to do it with ! At the moments I 80. __________
travel into Nottingham every day to the shop as I work. 81. __________
There' s a bus at 7:30 in the morning or the last bus back 82. __________
in the evening is at 5: 30. From September, therefore, 83. __________
I' m going to stay in Nottingham by my aunt and uncle 84. __________
when my cousin joins in the Navy. I can use his room. 85. __________
I' m really looking forward to living in Nottingham.
第二节书面表达(满分25分)
假设你是李华。美国一所友好学校准备与你们学校共同创办一份中学生英文刊物,该校格林先生来信询问有关情况。请你给他写封回信,主要内容如下:
表示赞成。
提出你喜欢的栏目。
简要说明理由。
注意:
1.词数100左右。
2.信的开头和结尾已为你写好。
3.适当增加细节,以使行文连贯。
4.参考词汇:栏目一column
﹡ ﹡﹡﹡﹡﹡﹡﹡﹡﹡﹡﹡﹡﹡﹡﹡﹡﹡﹡﹡﹡﹡﹡﹡﹡﹡﹡﹡﹡﹡﹡﹡﹡﹡﹡﹡﹡﹡﹡﹡﹡ June 8
Dear Mr. Green,
I am very excited to learn that we are going to start a magazine together._____________________
___________________________________________________________________________________
Yours faithfully
Li Hua
2007年普通高等学校招生全国统一考试英语招生试题
安徽卷参考答案
第一部分:
1. B 2. A 3. B 4. A 5. C
6. C 7. A 8. C 9. B 10. B
11. A 12. C 13. A 14. B 15. B
16. C 17. A 18. B 19. C 20. C
第二部分:
21. B 22. D 23. D 24. C 25. D
26. B 27. A 28. A 29. A 30. D
31. C 32. C 33. B 34. D 35. C
36. C 37. A 38. B 39. A 40. D
41.A 42. C 43. D 44. C 45. B
46. C 47. A 48. B 49. C 50. A
51. C 52. B 53. D 54. D 55. B
第三部分:
56. B 57. D 58. B 59. C 60. A
61. C 62. C 63. B 64. A 65. D
66. D 67. A 68. B 69. C 70. A
71. B 72. A 73. D 74. C 75. B
第四部分:
第一节:
I' m eighteen years old and I five in ∧ small village in 76. a
the Peak District, not far from Nottingham. I grow up 77. grew
in this village, so I know everyone here. That' s ∧ the 78. not
problem -- the problem is that there' s nothing to do, 79. √
and nobody of my age to do it with! At the moments I 80. moment
travel into Nottingham every day to the shop as I work. 81. where
There' s a bus at 7:30 in the morning or the last bus back 82. and
in the evening is at 5: 30. From September, therefore, 83. however
I' m going to stay in Nottingham by my aunt and uncle 84. with
when my cousin joins in the Navy. I can use his room. 85. in
I' m really looking forward to living in Nottingham.
第二节:
One Possible Version
June 8
Dear Mr. Green,
I am very excited to learn that we are going to start a magazine together. It is indeed a wonderful idea and everyone hopes for its success.
I would like the magazine to have three columns: News, Teachers' Advice and Students' Voice. From the News column we can learn about what is going on in both of our schools. In Teachers' Advice, our teachers can give us such help as we may not be able to get in the classroom. Students' Voice will allow us to freely express our thoughts and feelings about our lives and studies.
I believe that this magazine will surely strengthen our ties, and I am looking forward to reading it.
Yours faithfully,
Li Hua
2007 年普通高等学校招生全国统一考试(山东卷)
英语试卷
本试卷分第I卷(选择题)和第II卷(非选择题)。第I卷1至14页,第II卷15至18页。满分为150分,考试时间120分钟。考试结束后,将本试卷和答题卡一并交回。
第Ⅰ卷(共105分)
考生注意事项:
1.答第 I卷前,考生务必将自己的姓名、准考证号、考试科目涂写在答题卡上。
2.每小题选出答案后,用 2B 铅笔把答题卡上对应题目的答案标号涂黑。如需改动,用橡皮擦干净后,再涂其他答案标号。不能答在试卷上。
第一部分 听力(共两节,满分 30 分)
该部分分为第一、第二两节。注意:回答听力部分时,请先将答案标在试卷上。听力部分结束前,你将有两分钟的时间将你的答案转涂到客观题答题卡上。
第一节 (共 5 小题;每小题 1.5 分,满分 7.5 分)
听下面 5 段对话。每段对话后有一个小题,从题中所给的 A、B、C 三个选项中选出最佳选项,并标在试卷的相应位置。听完每段对话后,你都有 10 秒钟的时间来回答有关小题和阅读下一小题。每段对话仅读一遍。
例:How much is the shirt?
A.£19.15.
B.£9.15.
C.£9.18.
答案是 B。
1.Who is coming for tea?
A.John.
B.Mark.
C.Tracy.
2.What will the man do next?
A.Leave right away.
B.Stay for dinner.
C.catch a train.
3.What does the man come for?
A.A lecture.
B.A meeting.
C.A party.
4.What size does the man want?
A.9.
B.35.
C.39.
5.What are the speakers talking about?
A.Life in Southeast Asia.
B.Weather conditions.
C.A holiday tour.
第二节:(共 15 小题;每小题 1.5 分,满分 22.5 分)
听下面 5 段对话或独白。每段对话或独白后有几个小题,从题中所给的 A、B、C 三个选 项中选出最佳选项,并标在试卷的相应位置。听每段对话或独白前,你将有时间阅读各个小 题,每小题 5 秒钟;听完后,各小题将给出 5 秒钟的作答时间。每段对话或独白读两遍。
听下面一段对话,回答第 6 和第 7 两个小题。
6.What is the man doing?
A.Giving a speech.
B.Chairing a meeting.
C.Introducing a person.
7.Why does the woman sing so well?
A.She has a great teacher.
B.She teaches singing.
C.She is young.
听下面一段对话,回家第 8 和第 9 两个小题。
8.What is the second gift for Jimmy?
A.A car.
B.A watch.
C.A computer.
9.Why does Jimmy feel happy?
A.He does watch has parents.
B.He’s get what be dreamt of.
C.He’s received lots of presents.
听下面一段对话,回答第 10 至第 12 三个小题。
10.What is the relationship between the speakers?
A.They are friends.
B.They are strangers to each other.
C.They are husband and wife.
11.Why does the woman come to talk with the man?
A.To get a job.
B.To take a test.
C.To see the secretary.
12.What does the man mean by saying sorry?
A.He can't hear the woman clearly.
B.He doesn't need a designer.
C.He can't help the woman.
听下面一段对话,回答第 13 至第 16 四个小题。
13.What do we know about the woman?
A.She lives close to the office.
B.She is new to the company.
C.She likes the big kitchen.
14.How does the man go to work?
A.On foot.
B.By bus.
C.By car.
15.Why was Susan late for work?
A.She missed the bus.
B.Her train was late.
C.Her car broke down.
16.What will the man do the next day?
A.Go to work by train.
B.Visit Lily in her flat.
C.Leave home earlier.
听下面一段独白,回答第 17 至第 20 四个小题。
17.Where can you most probably hear this talk?
A.In a class of the English language.
B.In a class of the Greek language.
C.In a class of the French language.
18.How long does the class last?
A.11 weeks.
B.13 weeks.
C.15 weeks.
19.What is “the short-cut” to learning words according to the speaker?
A.Taking more courses.
B.Reading basic words aloud.
C.Learning how words are formed.
20.Why is the class popular?
A.It is not offered each term.
B.It's taught by Professor Morris.
C.It helps to master some useful rules.
第二部分:英语知识运用(共两节,满分 35 分)
第一节:语法和词汇知识(共 15 小题;每小题 1 分,满分 15 分)
从 A、B、C、D 四个选项中,选出可以填入空白处的最佳选项,并在答题卡上将该项涂黑。
例:Mr. Smith owns    collection of coins than anyone else I have ever met.
A.larger B.a larger C.the larger D.large
21.     walk is expected to last all day, so bring    packed lunch.
A.A ; a   
B.The; 不填
C.The ; a
D.A ; 不填
22.Could I speak to     is in charge of International Sales, please?
anyone  
B.someone
C.whoever
D.no matter who
23.—Have you been wasting time on computer games again?
—     . I've been studying a lot and I need a break.
A.No way
B.Not really
C.I don't agree
D.I couldn't agree more
24.     worries me the way he keeps changing his mind.
A.This
B.That
C.What
D.It
25.It’s the sort of work that      a high level of concentration.
A.calls for
B.makes up
C.lies in
D.stands for
26.Please remain     until the plane has come to a complete stop.
A.to seal
B.to be seated
C.seating
D.seated
27.I can't say which wine is beat—it's a (n)      of personal taste.
A.affair    
B.event
C.matter    
D.variety
28.I have offered to point the house-_________a week's accommodation.
A.in exchange for  
B.with regard to
C.by means of  
D.in place of
29._________I really don't like art, I find his work impressive.
A.As     
B.Since
C.If     
D.While
30.___Where did you get to know her?
___It was on the farm_________we worked.
A.that    
B.there
C.which    
D.where
31.It is nt socially_________ for parents to leave children unattended at that age.
A.accessible    
B.adorable
C.adaptable    
D.acceptable
32.They_________two free tickets to Canada, otherwise they'd never have been able to afford to go.
A.had got    
B.got
C.have got    
D.get
33.The country has already sent up three unmanned spacecraft, the most recent ---------at the end of last March.
A.has been launched   
B.having been launched
C.being launched   
D.to be launched
34.In this seaside resort, you can_________all the comfort and convenience of modern tourism.
A.enjoy     
B.apply
C.receive    
D.achieve
35.The book was written in 1946, _________the education system has witnessed great charges.
A.when        
B.during which
C.since then     
D.since when
第二节:完形填空(共 20 小题;每小题 1 分,满分 20 分)
阅读下面短文、掌握其大意、然后从 36~55 各题所给的四个选项(A、B、C 和 D)中,
选出最佳选项、并在答题卡上将该项涂黑。
Life is filled with challenges. As we get older we 36 realize that those challenges to the very things than 37 us and make us who we are ,it is the same with the challenges that come with 38 .
When we are fared with a challenge, we usually have two 39 . we can try to beat it off, or we can decide that the thing 40 the challenge isn’t worth the 41 and call it quits. Although there are certainly 42 when calling it quits it the right thing to do, in most 43 all that is needed is 44 and communicable.
When we are communed to something, it means that no matter how 45 or how uncomfortable something is, we will always choose to 46 it trough instead of running away from it. Communication is making a 47 for discussion and talking about how you feel as opposed to just saying what the other person did wrong. 48 you can say to a friend, “I got my feelings hurt.” 49 “You hurt my feelings,” you are going to be able solve the problem much faster.
In dealing with many challenges that friendship will bring to you, try to see them for 50 they are: small hurdles you need to jump or 51 on your way through life. Nothing is so big that it is 52 to get over, and hurt only 53 to make us stronger. It s all part of growing up, it 54 to everyone, and some day you will 55 all of this and say, “Hard as it was, it make me who I am today. And that a good thing.”
36.A.seem to    B.come to    C.hope to     D.try to
37.A.design     B.promote    C.direct      D.shape
38.A.confidence   B.pressure   C.friendship    D.difficulty
39.A.opportunities B.expectations C.choices     D.aspects
40.A.demanding   B.deserving   C.predicting    D.presenting
41.A.comment    B.loss     C.trouble     D.expense
42.A.spans     B.times     C.dates      D.ages
43.A.cases     B.fields    C.parts      D.occasions
44.A.assessment   B.commitment  C.encouragement  D.adjustment
45.A.doubtful   B.shameful   C.harmful     D.painful
46.A.keep     B.control    C.face       D.catch
47.A.space     B.plan     C.topic      D.room
48.A.If      B.As      C.While      D.Unless
49.A.other than   B.rather than C.or rather    D.or else
50.A.what      B.who     C.where      D.which
51.A.pass by    B.come across C.get through   D.run over
52.A.unnecessary  B.necessary  C.impossible   D.possible
53.A.serves    B.means    C.aims      D.attempts
54.A.opens     B.appeals   C.goes      D.happens
55.A.lock down on B.look back on C.look forward to D.look up to
第三部分:阅读理解(共 20 小题;每小题 2 分,满分 40 分)
阅读下列短文,从每题所给的四个选项中(A、B、C和 D)中,选出最佳选项,并在答
题卡上将该项涂黑。
A
A month after Hurricace Katrina, I returned home in New Orleans. There lay my house, reduced to waist-high rains, smelly and dirty.
Before the trip, I'd had my car fixed. When the office employee of the garage was writing up the bill, she noticed my Louisiana license plate. “You from New Orleans?” she asked. I said I was, “No charge.” She said, and firmly shock her head when I reached for my wallet. The next day I went for a haircut, and the same thing happened.
As my wife was studying in Florida, we decided to move there and tried to find a rental house that we could afford while also paying off a mortgage(抵押贷款)on our ruined house. We looked at many places, but none was satisfactory. We'd began to accept that we'd have to live in extremely reduced circ*mstances for a while, when I got a very curious e-mail from a James Kennedy in California. He'd read some pieces I'd written about our sufferings for state, the online magazine and wanted to give us ("no conditions attached") a new house across the lake from New Orleans.
It sounded a good to her return, but I replied, thinking him for his exceptional generosity, then we to go back. Then the University of Florida offered to let him house to me. While he want to England on his one year, paid leave. The rent was rather reasonable. I mentioned the poet's offer to James Kennedy, and the next day he sent a check covering our entire rent for eight months.
Throughout this painful experience, the kindness of strangers back my faith in humanity. It's almost worth losing you wordy possessions to be reminded that people really when given had a channel.
56.The garage employee's attitude toward the author was that of      .
A.unconcern    
B.sympathy
C.doubt      
D.tolerance
57.What do we know about James Kennedy?
A.He was a written of an online magazine.
B.He was a poet at the University of Florida
C.He offered the author a new house free of charge.
D.He learned about the author's sufferings.
58.It can be inferred from the text that     .
A.the author's family was in financial difficulty
B.rents were comparatively reasonable despite the disaster
C.houses were difficult to find in the hurricane0stricken area
D.the mortgage on the ruined house was paid off by the bank
59.The author learned from his experience that      .
A.worldly possessions can be given up when necessary
B.generosity should be encouraged in some cases
C.people benefit from their sad stories
D.human beings are kind after all.
B
If you look for a book as a present for a child. You will be spoiled for choice even in a year there is no new Harry Patten J.K Rowling's wizard is not alone; the past decade has been a harvest for good children's books, which has set off a large quantity of films and an increased sales of classics such as The lard of the Rings.
Yet despite that, reading is increasingly unpopular among children. According to statistics in 1997 23% said they didn't like reading at all. In 2003, 35% did. And around 6% of children leave primary school each year unable to read properly.
Maybe the decline is caused by the increasing availability of computes games. Maybe the books boom has affected only the top of the educational pile. Either way, Chancellor Cordon Brown plans to change things for the bottom of the class. In his pre-budget report, he announced the national project of Reading Recovery to help the children struggling most.
Reading Recovery is wined at six-year-olds, who receive four months of individual daily half-hour classes with a specially trained teacher. An evaluation either this year reported that children on the school made 20 months' progress in just one year, whereas similarly weak readers without special help made just five months' progress, and so ended the year even further below the level expected for their age.
International research tends to find that when British children leave primacy school they read well, but read text often for fun than those elsewhere. Reading for fun matters because children who are been on reading can report lifelong pleasure and loving books is an excellent indicator of future educational success. According to the OECD, being a regular and enthusiastic reader is of great advantage.
60.Which of the following is true of Paragraph 1?
A.Marry children's books have been adapted from films.
B.Marry high-quality children's books have been published.
C.The sales of classics have led to the popularity of films.
D.The sales of presents for children have increased.
61.Statistics suggested that      .
A.the number of top students increased with the use of computers
B.a decreasing number of children showed interest in reading
C.a minority of primacy school children read properly
D.a huge percentage of children read regularly
62.What do we know about Reading Recovery?
A.An evaluation of it will be made sometime this year.
B.Weak readers on the project were the most hardworking.
C.It aims to train special teachers to help children with reading.
D.Children on the project showed noticeable progress in reading.
63.Reading for fun is important because book-loving children _________.
A.take greater advantage of the project
B.show the potential to enjoy a long life
C.are likely to succeed in their education.
D.would make excellent future researchers
64.The arm of this text would probably be _________.
A.to overcome primary school pupils reading difficulty.
B.to encourage the publication of more children's books
C.to remind children of the importance of reading for fun
D.to introduce a way to improve early children reading
C
When Andrea Peterson landed her first teaching job, she faced the daunting task of creating a music program with almost no money for equipment or supplies in a climate where standards-based learning was the focus and music just provided a break for students and teachers.
For her drive and creativity in overcoming those challenges, she’s been named national teacher of the year.
Principal Waynes Kettler said he's worked with many outstanding teachers in his 22 years as an educator, but Peterson is "just that one step above anybody I've ever worked with before."
Kettler and others at Monte Cristo Elementary School talk about the ways she has introduced the learning from other classrooms into her music program and her creativity in working around things such as the lack of money for new music.
When students were reading S.E. Hinton's novel The Outsiders in their regular classroom, Peterson helped them write a 30-minute play with scenes from the book. Then they chose three Broadway tunes that focused no race, equality and social justice, the themes of the book. Peterson composed two other songs herself after classroom discussions about the play and the book.
The honor means a lot to residents of Granite Foils. It's inspiring to know that people from small towns own even win national honors.
As national teacher of the year, Peterson will spend the more year outside the classroom, as a national and international spokeswoman for education.
Not surpassingly. She is a big believe in the white of acts education. She said it's essential for schools to offer classes such as act or music and physical education because for some kids one of those subjects is the only thing that motivates them to come back to school day after day.
65.The underlined word "daunting" in Paragraph 1 most probably means __________.
Discouraging  
B.interesting
C.creative   
D.unbearable
66.When Peterson began her teaching career, ____________.
A.music was a focus of learning in most schools
B.the environment was favorable to music teaching
C.the school backed teaching facilities for music
D.immemorial support for music programs was unavailable
67.What is the most important reason that Peterson won the award?
A.She concerned herself with current social problems.
B.She motivated students to learn music with her creativity.
C.She has aught music at the elementary school for 22 years.
D.She make great efforts to amuse students' interest in literature.
68.Which of the following is an example of Peterson's way of teaching music?
A.She wrote plays on themes of race, equality and social justice.
B.She made use of the contents of other classes in her teaching.
C.She organized classroom discussions of Broadway tunes.
D.She helped students compose songs by themselves.
69.In Peterson's opinion, ____________.
A.art music and PE classes are all important
B.more subjects should be offered to students
C.students should be motivated to attend art classes
D.arts education is more important than other subjects
70.It can be inferred from the text that ________.
A.Peterson's honor was a surprise for the local people
B.Peterson's art classes attracted students back to school
C.Peterson aroused the local residents' passion for music
D.Peterson will change her profession next year
D
Sports shoes that out whether their owner has enough exercise to warrant time in front of the television have been devised in the UK.
The shoes — named Square Eyes — contain an electronic pressure sensor and a tiny computer chip to record how many steps the wearer has taken in a day. A wireless transmitter passes the information to a receiver connected to a television, and this decides how much evening viewing time the wearer deserves, based on the day's efforts.
The design was inspired by a desire to fight against the rapidly ballooning waistlines among British teenagers, says Gillian Swan, who developed Square Eyes as a final year design project at Brunel University to London, UK. "We looked at current issues and childhood overweight really stood out, " she says. "And I wanted to tackle that with my design. "
Once a child has used up their daily allowance gained through exercise, the television automatically switches off. And further time in front of the TV can only be earned through more steps.
Swan calculated how exercise should translate to television time using the recommended daily amounts of both. Health experts suggest that a child take 12,000 steps each day and watch no more than two hours of television. So, every 100 steps recorded by the Square Eyes shoes equals precisely one minute of TV time.
Existing pedometers (计步器) normally clip onto a belt or slip into a pocket and keep count of steps by measuring sudden movement. Swan says these can be easily tricked into recording steps through shaking. But her shoe has been built to be harder for lazy teenagers to cheat. "It is possible, but it would be a lot of effort, " she says. "That was one of my main design considerations. "
71.According to Swan, the purpose of her design project is to ________.
A.keep a record of the steps of the wearer
B.deal with overweight among teenagers
C.enable children to resist the temptation of TV
D.prevent children from being tricked by TV programs
72.Which of the following is true of Square Eyes shoes?
A.They regulate a child's evening TV viewing time.
B.They determine a child's daily pocket money.
C.They have raised the hot issue of overweight.
D.They contain information of the receiver.
73.What is stressed by health experts in their suggestion?
A.The exact number of steps to be taken.
B.The precise number of hours spent on TV.
C.The proper amount of daily exercise and TV time.
D.The way of changing steps into TV watching time.
74.Compared with other similar products, the new design ________.
A.makes it difficult for lazy teenagers to cheat
B.counts the wearer’s steps through shaking
C.records the sudden movement of the wearer
D.sends teenagers' health data to the receiver
75.Which of the following would be the best title for the text?
A.Smart Shoes Decide on Television Time
B.Smart Shoes Guarantee More Exercise
C.Smart Shoes Measure Time of Exercise
D.Smart Shoes Stop Childhood Overweight
第Ⅱ卷(共 45 分)
注意事项:
1.用黑色或蓝色钢笔,圆珠笔直接答在试卷上。
2.答卷前将密封线内的项目填写清楚。
第四部分:书面表达(共两节,满分 45 分)
第一节:阅读表达(共 5 小题,每小题 3 分,满分 15 分)
阅读下面的短文,请根据短文后的要求答题(请注意问题后的字数要求)
An American friend of mine who was high up in a big corporation had worked out a way of heading a good of e-mails before most of us bad, even heard of the concept. If any information he was sent was vital enough, his back of response would ensure the sender rang him up, if the sender wasn't important enough to have his private numbers the communication couldn't be that important. My friend is now even more senior in the same company, so the strategy must work.
Almost every week now there seems to be a report suggesting that we are all being driven crazy by the bother of e-mail. If this is the case, it's only because we haven't developed an appropriate in dealing with it.
________ Firstly. You junk nothing with an exclamation mark on a string of capital letters, or from my adders, you don't recognize on feel confident about.
Secondly, e-mail don't and have to be answered. Because e-mail is so easy, there's a tendency for correspondence to carry on for ever, but it is permissible to stop an endless discussion or to accept a point of information sent by a colleague without acknowledging it.
Thirdly, a reply e-mail thoughts have to be the same length as the original. We all have e-mail pals who read long, chatty e-mail, which are nice to receive, but who then expect an equally long reply. The chart of e-mail can consist in the simple, incomplete sentence, totally regardless of the format of the bread of the letter meat by past. You are perfectly within the bounds of polyandrous in responding to a marathon e-mail with a better reply.
76.Which sentence in the passage to the following one?
The possible existence of annoyance results from our inability to sort out e-mails.
________________________________________________________________
77.The Fill on the blank in Paragraph 3 with a proper sentence.(within 10 words )
________________________________________________________________
78.What advice to given in the last paragraph? (within 10 words)
________________________________________________________________
79.For what purpose does the author mention his American friend in Paragraph 1? (within 10 words)
________________________________________________________________
80.Translate the underlined sentence in the last paragraph into Chinese.
________________________________________________________________
第二节:写作(满分 30 分)
阅读下面的文字,用英语写一篇 120—150 词的短文。
今年,教育部直属师范大学将招收一批免费师范生,学生毕业后须回生源所在省份的中小学任教十年以上,你愿意成为一名免费师范生吗?请陈述理由。
2007 年普通高等学校招生全国统一考试(山东卷)
英语试卷
参考答案
第Ⅰ卷
1.B  2.A  3.B  4.A  5.C  6.C  7.A  8.C 
9.B  10.B  11.A  12.C  13.A  14.B  15.B  16.C
17.A 18.B  19.C  20.C  21.C  22.C  23.B  24.D
25.A 26.D  27.C  28.A  29.D  30.D  31.D  32.B
33.B 34.A  35.D  36.B  37.D  38.C  39.C  40.D
41.C 42.B  43.A  44.B  45.D  46.C  47.A 48.A
49.B 50.A  51.C  52.C  53.A  54.D  55.B 56.B
57.C 58.A  59.D  60.B  61.B  62.D  63.C 64.D
65.A 66.C  67.B  68.B  69.A 70.A  71.B 72.A
73.C 74.A  75.A
第Ⅱ卷
76.If this is the case, it's only because we haven't developed an appropriate discrimination in dealing with it.
77.E-mail(s) can be handled in the following ways.
Here are the ways to deal with e-mail(s).
Handling e-mail(s) is an art.
78.You may reply to a long e-mail briefly
You don't have to write a long reply e-mail.
79.To show a way of handling e-mail(s) with an example.
To introduce the topic of the text.
80.用简短的话回复冗长的电子邮件完全不失礼节。
用简短的话回复马拉松式的电子邮件也是礼貌的。
写作题(略)
2007年普通高等学校招生全国统一考试
英 语 (江苏卷)
第 I 卷 (共115分)
第一部分:听力(共两节,满分30分)
第一节 (共5小题;每小题1.5分,满分7.5分)
  听下面5段对话。每段对话后有一个小题,从题中所给的A、B、C三个选项中选出最佳选项,并标在试卷的相应位置。听完每段对话后,你都有10秒钟的时间来回答有关小题和阅读下一小题。每段对话仅读一遍。
1. Who is coming for tea?
A. John. B. Mark. C. Tracy.
2. What will the man do next?
A. Leave right away. B. Stay for dinner. C. Catch a train.
3. What does the man come for?
A. A lecture. B. A meeting. C. A party.
4. What size does the man want?
A. 9. B. 35. C. 39.
5. What are the speakers talking about?
A. Life in Southeast Asia.B. Weather condition. C. A holiday tour.
第二节 (共15小题;每题1.5分,满分22.5分)
  听下面5段对话或独白。每段对话或独白后有几个小题,从题中所给的A、B、C三个选项中选出最佳选项,并标在试卷的相应位置。听每段对话或独白前,你将有时间阅读各个小题,每小题5秒钟;听完后,每小题将给出5秒钟的作答时间。每段对话或独白读两遍。
听第6段材料,回答第6、7题。
6. What is the man doing?
A. Giving a speech.B. Chairing a meeting.C. Introducing a person.
7. Why does the woman sing so well?
A. She has a great teacher.B. She teaches singing.C. She is young.
听第7段材料,回答第8、9题。
8. What is the second gift for Jimmy?
A. A car. B. A watch. C. A computer.
9. Why does Jimmy feel happy?
A. He lives with his parents. B. He’s got what he dreamt of.
C. He’s received lots of presents.
听第8段材料,回答第10至12题。
10. What is the relationship between the speakers?
A. They are friends. B. They are strangers to each other.
C. They are husband and wife.
11. Why does the woman come to talk with the man?
A. To get a job. B. To take a test. C. To see the secretary.
12. What does the man mean by saying sorry?
A. He can’t hear the woman clearly. B. He doesn’t need a designer.
C. He can’t help the woman.
听第9段材料,回答第13至16题。
13. What do we know about the woman?
A. She lives close to the office. B. She is new to the company.
C. She likes the big kitchen.
14. How does the man go to work?
A. On foot. B. By bus. C. By car.
15. Why was Susan late for work?
A. She missed the bus. B. Her train was late. C. Her car broke down.
16. What will the man do the next day?
A. Go to work by train. B. Visit Lily in her flat. C. Leave home earlier.
听第10段材料,回答第17至 20 题。
17. Where can you most probably hear this talk?
A. In a class of the English language. B. In a class of the Greek language.
C. In a class of the French language.
18. How long does the class last?
A. 11 weeks. B. 13 weeks. C. 15 weeks.
19. What is “the short-cut” to learning words according to the speaker?
A. Taking more courses. B. Reading basic words aloud.
C. Learning how words are formed.
20. Why is the class popular?
A. It is not offered each term. B. It’s taught by Professor Morris.
C. It helps to master some useful rules.
第二部分:英语知识运用(共两节, 满分45分)
第一节:单项填空(共15小题,每小题1分,满分15分)
21. We have every reason to believe that ______ 2008 Beijing Olympic Games will be ______ success.
A. / … aB. the … / C. the … aD. a … a
22. --- She looks very happy. She ______ have passed the exam.
--- I guess so. It’s not difficult after all.
A. shouldB. couldC. mustD. might
23. At the end of the meeting, it was announced that an agreement ______.
A. has been reachedB. had been reached
C. has reachedD. had reached
24. --- Have you ______ some new ideas?
--- Yeah. I’ll tell you later.
A. come aboutB. come intoC. come up withD. come out with
25. Choosing the right dictionary depends on ______ you want to use it for.
A. whatB. whyC. howD. whether
26. You may not have played very well today, but at least you’ve got through to the next round and ______.
A. tomorrow never comesB. tomorrow is another day
C. never put off till tomorrowD. there is no tomorrow
27. --- ______ you ______ him around the museum yet?
--- Yes. We had a great time there.
A. Have … shownB. Do … showC. Had … shownD. Did … show
28. --- Can I smoke here?
--- Sorry. We don’t allow ______ here.
A. people smokingB. people smokeC. to smokeD. smoking
29. With April 18’s railway speedup, highway and air transport will have to compete with ______ service for passengers.
A. goodB. betterC. bestD. the best
30. He is very popular among his students as he always tries to make them ______ in his lectures.
A. interestedB. interestingC. interestD. to interest
31. --- My room gets very cold at night.
--- ___________.
A. So is mineB. So mine isC. So does mineD. So mine does
32. --- “Could we put off the meeting?” she asked.
--- “_________.” He answered politely. “This is the only day everyone is available.”
A. Not likelyB. Not exactlyC. Not nearlyD. Not really
33. He was educated at the local high school, ______ he went on to Beijing University.
A. after whichB. after thatC. in whichD. in that
34. --- Do you think that housing price will keep ______ in the years to come?
--- Sorry, I have no idea.
A. lifting upB. going upC. bringing upD. growing up
35. My parents have always made me ______ about myself, even when I was twelve.
A. feeling wellB. feeling goodC. feel wellD. feel good
第二节 完型填空(共20小题;每小题1.5分,满分30分)
  Carolyn Stradley is the founder of C&S Paving Inc. (铺路公司) in Atlanta, USA. In the following account, she recalls the job that challenged her 36 and skill but left her flying high.
  “When the Atlanta Airport was under 37 in 1979, we were a new company struggling to make it. National Car Rental wanted to have 2,500 square meters of dirt paved 38 the cars could be on site 39 the airport opened, and the official opening was only ten days away! 40 other local paving company wanted to do the job, 41 it couldn’t be done in such a short time.
  “Because we were new and really needed the work, we were 42 to try harder. We gave National Car Rental our offer and 43 our best effort to get the job finished within ten days. We also 44 them that if we failed, they would be no worse off, 45 they had plenty to gain if we succeeded.
  “We got the job and immediately went into 46 . Working at night needed lights, so I rented a machine to produce electricity for the site. Our 47 challenge was to keep the rock mixture 48 enough. All the available water wagons (洒水车) were rented out for the airport construction, and we certainly couldn’t afford to buy a new one. 49 , I got a special 50 to rent fire engine hoses (消防水龙带) and connect them to nearby hydrants (消防栓); then I 51 held one of those hoses to 52 down the rock.
  “Those ten days were filled with challenges that 53 one creative idea after another. Nine days later, the night before the airport opened, National Car Rental was the 54 company that had cars on the parking lot.
  “The key to our success was having the 55 to take on any job and then being creative in our approach to getting it done.”
36. A. kindnessB. patienceC. imaginationD. experience
37. A. constructionB. repairC. controlD. development
38. A. afterB. asC. forD. so
39. A. whileB. sinceC. whereD. when
40. A. SomeB. AnyC. NoD. Every
41. A. statingB. reportingC. tellingD. warning
42. A. ableB. nervousC. afraidD. willing
43. A. supportedB. promisedC. continuedD. improved
44. A. askedB. surprisedC. remindedD. demanded
45. A. though B. butC. asD. unless
46. A. discussionB. actionC. practiceD. production
47. A. nextB. firstC. pastD. previous
48. A. coldB. wetC. looseD. clean
49. A. NaturallyB. ObviouslyC. MeanwhileD. Instead
50. A. excuseB. orderC. permitD. reason
51. A. exactlyB. personallyC. angrilyD. hardly
52. A. pullB. knockC. hitD. water
53. A. requiredB. mixedC. followedD. formed
54. A. bestB. lastC. secondD. only
55. A. courageB. interestC. hopeD. chance
第三部分:阅读理解(共20小题,每题2分,满分40分)
A
Su Hua is studying at Cambridge, UK. She has bought a bicycle and is worried about security (安全). Her friend, Kate, found this article and sent it to her.

Introduction
A lot of crime is against bicycles. About 150,000 bicycles are stolen every year and most are never found. You can prevent this happening by following a few careful steps.
Basic Security
Do not leave your bicycle in out-of-the-way places. Always lock your bicycle when you leave. Secure it to lampposts or trees. Take off smaller parts and take them with you, for example lights and saddles (车座).
Locks
Get a good lock. There are many different types in the shops. Buy one that has been tested against attack. Ask for a recommendation from a bike shop.
Marking
Security marking your bike can act a deterrent to a thief. It can also help the police find your bicycle. It should be clearly written and include your postcode and your house or flat number. This will provide a simple way to identify your bicycle.
Registration
There are a number of companies who will security mark your bicycle for you. They will then put your registration number and personal details on their computer database. Then if your bicycle is found it will be easy to contact you.
Finally
Keep a record of the bicycle yourself: its make, model and registration number. You can even take a photograph of it. This will prove the bicycle belongs to you.
56. Which part of the text gives you information on how to lock up your bicycle when you leave it?
A. Locks.B. Marking.C. Registration.D. Basic Security.
57. The underlined phrase “act as a deterrent to a thief” means ______.
A. help you recognize your bike
B. help the police find your bicycle
C. stop someone stealing your bicycle
D. stop you worrying about your bike
58. The article advises you to keep a record of your bicycle _______.
A. in the bike shop and your computer
B. in the police station and a security company
C. in a security company and your university
D. by yourself and in a security company
59. The main purpose of this article is _________.
A. to tell you what to do if your bicycle is stolen
B. to suggest ways of keeping your bicycle safe
C. to give you advice on where to buy a good lock
D. to say why you shouldn’t keep your bicycle in a quiet place
B
  Professor Barry Wellman of the University of Toronto in Canada has invented a term to describe the way many North Americans interact (互动) these days. The term is “networked individualism”. This concept is not easy to understand because the words seem to have opposite meanings. How can we be individuals (个体) and be networked at the same time? You need other people for networks.
  Here is what Professor Wellman means. Before the invention of the Internet and e-mail, our social networks included live interactions with relatives, neighbors, and friends. Some of the interaction was by phone, but it was still voice to voice, person to person, in real time.
  A recent research study by the Pew Internet and American Life Project showed that for a lot of people, electronic interaction through the computer has replaced this person-to- person interaction. However, a lot of people interviewed for the Pew study say that’s a good thing. Why?
  In the past, many people were worried that the Internet isolated (孤立) us and caused us to spend too much time in the imaginary world of the computer. But the Pew study discovered that the opposite is true. The Internet connects us with more real people than expected — helpful people who can give advice on careers, medical problems, raising children, and choosing a school or college. About 60 million Americans told Pew that the Internet plays an important role in helping them make major life decisions.
  Thanks to the computer, we are able to be alone and together with other people — at the same time!
60. The underlined phrase “networked individualism” probably means that by using computers people ________.
A. stick to their own ways no matter what other people say
B. have the rights and freedom to do things of their own interest
C. do things in their own ways and express opinions different from other people
D. are able to keep to themselves but at the same time reach out to other people
61. According to the Pew study, what do many people rely on to make major life decisions?
A. Networks.B. Friends.C. Phones.D. Parents.
62. It can be inferred from the Pew study that _______.
A. people have been separated from each other by using computers
B. the Internet makes people waste a lot of time and feel very lonely
C. the Internet has become a tool for a new kind of social communication
D. a lot of people regard the person-to-person communication as a good thing
63. Which would be the best title for this passage?
A. We’re Alone on the Internet.
B. We’re Communicating on the Internet.
C. We’re Alone Together on the Internet.
D. We’re in the Imaginary World of the Internet.
C
  
  Experience the newly opened Grand Canyon (大峡谷) West Skywalk in Colorado. Departing from Grand Canyon’s South Rim by Airplane to Grand Canyon’s West Rim, you will land and take a ground tour to the Skywalk! Walk on air for 70 feet over the edge of Grand Canyon West.
  This Skywalk has been open since March 28, 2007. Daily visitorship to the Skywalk has been over 4,000 people. Please be patient to enjoy your moment on the Skywalk.
  After you have experienced the one and only Grand Canyon Skywalk Glass Bridge, you will return to the Grand Canyon West Airport and take your Airplane for a flight back to the South Rim of the Canyon. This is a tour never to be forgotten as you will have walked on air over the Grand Canyon.
Tour Itinerary (行程)
Tour Duration
5.7 Hours
The Grand Canyon Adventure Skywalk
Flight from Grand Canyon South Rim to Grand Canyon West
1 Hour
Experience a bird’s-eye view of the Grand Canyon as you make your way to Grand Canyon West.
Light Lunch at Guano Point at Grand Canyon West
2 Hours
You’ll be taken by bus to Guano Point with breathtaking views of the western part of the Grand Canyon where the Calorado River makes its way into Lake Mood. Every table for lunch has a view.
Walk on the World Famous Skywalk
1.5 Hours
Finally you’ll board your bus to Eagle Point, home of the Grand Canyon Skywalk. Now it is time for you to walk on air for 70 feet over the Grand Canyon.
Flight Back to Grand Canyon South Rim
1.2 Hours
After time on the Skywalk, you’ll return to the Grand Canyon West Airport and return to Grand Canyon South Rim in time for dinner and sunset.
64. This advertisem*nt is for __________.
A. Grand Canyon WestB. Grand Canyon South
C. Grand CanyonD. the Skywalk
65. The package fee does NOT cover the cost of _______.

66. The Grand Canyon Skywalk Bridge is made of glass because ______.
A. it looks strangerB. it is cheaper to build
C. it looks more beautifulD. it gives you a better view
67. According to the Tour Itinerary, the route is ______.
A. South Rim→Guano Point→West Airport→Eagle Point→West Airport→South Rim
B. South Rim→West Airport→Guano Point→Eagle Point→West Airport→South Rim
C. South Rim→West Airport→Eagle Point→Guano Point→West Airport→South Rim
D. South Rim→West Airport→Eagle Point→West Airport→Guano Point→South Rim
D
  Thinking
* Some scientists claim that we humans are the only living things that are conscious (有意识的) — we alone are aware that we are thinking.
* No one knows how consciousness works — it is one of science’s last great mysteries.
* All your thoughts take place in the cerebrum (大脑皮质), which is at the top of your brain, and different kinds of thought are linked to different areas, called association areas.
* Each half of the cerebrum has four rounded ends called lobes (脑叶) — two at the front (frontal and temporal lobes) and two at the back (occipital and parietal lobes).
* The frontal lobe is linked to your personality and it is where you have your bright ideas.
* The temporal lobe is where you hear and understand what people say to you.
* The occipital lobe is where you work out what your eyes see.
* The parietal lobe is where you record touch, heat and cold, and pain.
* The left half of the brain controls the right side of the body. The right half controls the left side.
* One half of the brain is always dominant (in charge). Usually, the left brain is dominant, which is why 90% of people are right-handed.
68. Which part of your cerebrum is most active when you are making a telephone call?
A. The frontal lobe.B. The temporal lobe.
C. The occipital lobe.D. The parietal lobe.
69. Which of the following statements is true?
A. One’s personality has something to do with the frontal lobe.
B. Bright ideas come from the parietal lobe.
C. The occipital and temporal lobes are at the back of the cerebrum.
D. The occipital lobe is in charge of sound.
70. From the passage, we know the reason why around 10% of people are left-handed is that ________.
A. their frontal lobe is usually dominant
B. their temporal lobe is usually dominant
C. their right brain is usually dominant
D. their left brain is usually dominant
E
  Some people bring out the best in you in a way that you might never have fully realized on your own. My mom was one of those people.
  My father died when I was nine months old, making my mom a single mother at the age of eighteen. While I was growing up, we lived a very hard life. We had little money, but my mom gave me a lot of love. Each night, she sat me on her lap and spoke the words that would change my life, “Kemmons, you are certain to be a great man and you can do anything in life if you work hard enough to get it.”
  At fourteen, I was hit by a car and the doctors said I would never walk again. Every day, my mother spoke to me in her gentle, loving voice, telling me that no matter what those doctors said, I could walk again if I wanted to badly enough. She drove that message so deep into my heart that I finally believed her. A year later, I returned to school — walking on my own!
  When the Great Depression (大萧条) hit, my mom lost her job. Then I left school to support the both of us. At that moment, I was determined never to be poor again.
  Over the years, I experienced various levels of business success. But the real turning point occurred on a vacation I took with my wife and five kids in 1951. I was dissatisfied with the second-class hotels available for families and was angry that they charged an extra $2 for each child. That was too expensive for the average American family. I told my wife that I was going to open a motel (汽车旅馆) for families that would never charge extra for children. There were plenty of doubters at that time.
  Not surprisingly, mom was one of my strongest supporters. She worked behind the desk and even designed the room style. As in any business, we experienced a lot of challenges. But with my mother’s words deeply rooted in my soul, I never doubted we would succeed. Fifteen years later, we had the largest hotel system in the world — Holiday Inn. In 1979 my company had 1,759 inns in more than fifty countries with an income of $ 1 billion a year.
  You may not have started out life in the best situation. But if you can find a task in life worth working for and believe in yourself, nothing can stop you.
71. What Kemmons’ mom often told him during his childhood was ______.
A. caringB. movingC. encouragingD. interesting
72. According to the author, who played the most important role in making him walk back to school again?
A. Doctors.B. Nurses.C. Friends.D. Mom.
73. What caused Kemmons to start a motel by himself?
A. His terrible experience in the hotel.
B. His previous business success of various levels.
C. His mom’s support.
D. His wife’s suggestion.
74. Which of the following best describes Kemmons’ mother?
A. Modest, helpful, and hard-working.
B. Loving, supportive and strong-willed.
C. Careful, helpful and beautiful.
D. Strict, sensitive and supportive.
75. Which of the following led to Kemmons’ success according to the passage?
A. Self-confidence, hard work, higher education and a poor family.
B. Mom’s encouragement, clear goals, self-confidence and hard work.
C. Clear goals, mom’s encouragement, a poor family and higher education.
D. Mom’s encouragement, a poor family, higher education and opportunities.
第 II 卷 (计35分)
第四部分:写作 (共两节,满分35分)
第一节:对话填空 (本节共10小题,每小题1分,满分10分)
 请认真阅读下列对话,并根据各题所给的首字母的提示,在答题卡右栏中标有题目的横线上,写出一个英语单词的完整、正确的形式,使对话通顺。
 D = David P = Peter
D: Peter, ten years ago yo uwanted to be a doctor ...
P: Yeah, but unfortunately I didn’t (76) m the
Requirements.
D: So what did you do then?
P: Well, I was very (77) d , but I just had to move
On and do something else with my life. So I took a
year off and worked as a volunteer on an aid program.
That made me realize (78) h important it is to
raise money to help people (79) l a poor life.
So that’s what I do.
D: Can you explain a bit more about what you (80) a
do?
P: I go to see people, put (81) f our suggestion
And organize activities to collect money.
D: So have you (82) a any of your goals?
P: Well, I guess so. I always said I wanted to help
(83) o people.
D: And what about your plan for the (84) f ?
P: My girlfriend and I intend to get married next year.
D: (85) C !

76. _______________
77. _______________
78. _______________
79. _______________
80. _______________
81. _______________
82. _______________
83. _______________
84. _______________
85. ________________
第二节 书面表达(满分25分)
  2007年4月29日上午10点到11点,全国亿万学生阳光体育活动在各地大中小学校同时进行,江苏省1,100万学生积极参与了这项活动。
  教育部倡导学生:
* 每天锻炼一小时
* 健康工作五十年
* 幸福生活一辈子
  请你根据以上提示,用英语准备一份发言稿,向同学们讲述一下阳光体育活动的有关情况,并就高三学生是否需要每天花一小时锻炼,谈谈你的看法及理由。
注意:
1、 发言稿应包括以上所有信息,要有适当发挥。
2、 词数:120左右。发言稿的开头和结尾已经写好,不计入总词数。
3、 参考词汇:阳光体育活动——a national student sports program;
教育部——the Ministry of Education
Good afternoon, everyone,
____________________________________________________________________
Thank you!
参考答案:
1-20: BABAC CACBB ACABB CABCC
21-35: CCBCA BADBA CDABD
36-55: CADDC ADBCB BABDC BDADA
56-75: DCDBD ACCDA DBBAC CDABB
对话填词:
76. meet 77. disappointed / discouraged / depressed
78. how 79. living / leading 80. actually
81. forward / forth 82. achieved 83. other
84. future 85. Congratulations
作文:
Good afternoon, everyone.
  On the morning of April 29, 2007, from 10:00 to 11:00, hundreds of millions of students from primary, middle schools and universities all over China joined in a national student sports program. Eleven million students in Jiangsu took an active part in this program.
  The Ministry of Education calls on students to exercise for an hour every day, in the hope that they will have good health to work fifty years and enjoy the whole life.
  Personally, I think it a good idea for us senior 3 students to have daily exercise. Although we take an hour a day for exercise, it is well worth it. Taking exercise helps us build up our body and keep a clear mind. Therefore, we can work more efficiently.
  Thank you!
  (125词)
英  语  试  题
第一卷(选择题 共115分)
第一部分 听力(共两节,满分30分)
做题时,先将答案标在试卷上。录音内容结束后,你将有两分钟的时间将试卷上的答案转涂到答题卡上。
第一节(共5小题;每小题1.5分,满分7.5分)
听下面5段对话。每段对话后有一个小题,从题中所给的A、B、C三个选项中选出最佳选项,并标在试卷的相应位置。听完每段对话后,你都有10秒钟的时间来回答有关小题和阅读下一小题。每段对话仅读一遍。
例:How much is the shirt?
A.$ 19.15. B.$ 9.15. C.$ 9.18.
答案是B。
1.Who is coming for tea?
A. John. B.Mark. C.Tracy.
2.What will the man do next?
A.Leave right esay B.Stay for dinner. C.Catch a train.
3.What does the man come for?
A.A lecture. B.A meeting. C.A party.
4.What size does the man want?
A.9. B.35. C.39.
5.What are the speakers talking about?
A.Life in Soutbeast Asia. B.Weather conditions. C.A boliday tour.
第二节(共15小题;每小题1.5分,满分22.5分)
听下面5段对话或独白。每段对话或独白后有几个小题,从题中所给的A、B、C三个选项中选出最佳选项,并标在试卷的相应位置。听每段对话或独白前,你将有时间阅读各个小题,每小题5秒钟;听完后,各小题将给出5秒钟的作答时间。每段对话或独白读两遍。
听第6段材料,回答第6、7题。
6.What is the man doing?
A.Giving a speech. B.Chairing a meeting. C.Introducing a person.
7.Why does the woman sing so well?
A.She has a great teacher. B.She teaches singing. C.She is yourne
听第7段材料,回答第8、9题。
8.What is the second gift for Jimmy?
A.A car. B.A watch. C.A computer.
9.Why does Jimmy feel happy?
A.He lives with his garens.
B.He’s got what he dreant of.
C.He’s received lots of presents.
听第8段材料,回答第10至12题。
10.What is the relationship between the speakers?
A.They are friends.
B.They are strangers to each other.
C.They are harsband and wife.
11.Why does the woman come to talk with the man?
A.To get a job.
B.To take a test.
C.To see the secretary.
12.What does the man mean by saying sorry?
A.He can’t hear the woman clearly.
B.He doesn’t need a designer.
C.He can’t help the woman.
听第9段材料,回答第13至16题。
13.What do we know about the woman?
A.She lives close to the office.
B.She is new to the company.
C.She likes the big kitchen.
14.How does the man go to work?
A.On foot. B.By bus. C.By car.
15.Why was Susan late for work?
A.She missed the bus. B.Her train was late. C.Her car brokle down.
16.What will the man do the next day?
A.Go to work by train. B.Visit Lily in her flat. C.Leave home earlier.
听第10段材料,回答第17至20题。
17.Where can you most probably hear this talk?
A.In a class of the English language.
B.In a class of the Greek language.
C.In a class of the French language.
18.How long does the class last?
A.11 weeks. B.13 weeks. C.15 weeks.
19.What is “the shourt-cut” to learning words according to the speaker?
A.Taking more courses.
B.Reading basic words aloud.
C.Learning how words are formed.
20.Why is the class popular?
A.It is not offered each term.
B.It’s taught by Professor Morris.
C.It helps to master some useful rules.
第二部分:英语知识运用(共两节,满分45分)
第一节:单项填空(共15小题;每小题1分,满分15分)
从A、B、C、D四个选项中,选出可以填入空白处的最佳选项,并在答题卡上将该项涂黑。
例:It is generally considered unwise to give a child he or she wants
A.however B.whatever C.whichever D.whenever
答案是B。
21.—Will you join us in the game?
—Thank you,
A.but why not?B.but I’d rather not.
C.and I won’tD.and I’ll join.
22.Don’t take too much of the medicine;it does you more harm than good if you .
A.do B.take C.like D.have
23.Scientists are convinced the positive effect of laughter physical and mental health.
A.of; at B.by; in C.of; on D.on;at
24.When asked why he went there, he said he was sent there for a space flight.
A.training B.being trained C.to have trained D.to be trained
25.A survey of the opinions of experts that three hours of outdoor exercise a week good for one’s health.
A.show; are B.shows; is C.show;is D.shows;are
26.Many people have come to realize that they ahould go on balanced diet and make room in their day for exercise.
A.a;/ B.the; a C.the;the D./;a
27.—Where is my dictionary? I remember I put it here yesterday.
—You it in the wrong place.
A.must put B.should have put C.might put D.might have put
28.—What do you think of the performance today?
—Great! But a musical genius could perform so successfully.
A.All B.None C.Anybody D.Everybody
29.He was told that it would be at least three more months he could receover and retum to work.
A.when B.before C.since D.that
30.The melon the Smiths served at dinner would have tasted if it had been put in the fridge for a little while.
A.good B.better C.best D.well
31.—I have got a beadache.
—No wonder. You in front of that computer too long.
A.work B.are working C.have been working D.worked
32.After gradutation she reached a point in her careet she heeded to decion what to do.
A.that B.what C.which D.where
33.Experts have been warning of the bealth risks caused by passive smoking
A.at a time B.at one time C.for some time D.for the time
34.I don’t mind her criticizing me,but is how she does it that I object to.
A.it B.that C.this D.which
35.—Ouch!You hurt me!
—I am sorry. But I any harm. I to drive a rat out.
A.didn’t mean; tried B.don’t mean; am trying
C.haven’t meant; tried D.didn’t mean; wes trying
第二节:完形填空(共20小题;每小题1.5分,满分30分)
阅读下面短文,掌握其大意,然后从36-55各题所给的四个选项(A、B、C和D)中,选出最佳选项,并在答题卡上将该项涂黑。
Until 1954 it was thought that no man could run one mile in less than four minutes. As years
36 ,the record came closer and closer to four minutes and Roger Bannister, a young English 37 ,began to believe be might 38 this almost magic barrier.
It was a cold afternoon on May 6th,1954,when Bannister knew be had a 39 chance. Bannister had been 40 hard and was very fit, but the weather conditions were a real 41 to him. Describring the 42 later,Bannister said,“On the way to the track the wind blew strongly. As I 43 for the start I glanced at the flag. It moved 44 now. This was the moment when I made my decision.”
“The gun fired. My legs 45 to meet no resistance, as if I was 46 forward by some unknown force. The noise from the faithful 47 gave me greater strength. I felt the 48 of a lifetimes had come.”
“I was driven on by a 49 of fear and pride. My body had long since used up all its energy 50 it went on running just the same. This was the critical moment when my legs were strong enough to carry me over the last few yards as they 51 could have done in previous years. When I leapt at(冲向)the 52 tape, I fell, almost 53 .
“I knew I had done it, even before I 54 the time. The announcement came. ‘Result of the one mile…Time, three minutes…’the test was 55 in the noise of excitement.”
36.A.passed along B.passed down C.went by D.went ove
37.A.coach B.athlete C.captain D.judge
38.A.defeat B.move C.beat D.break
39.A.real B.lucky C.serious D.false
40.A.competing B.training C.figthting D.attending
41.A.eagerness B.pleasure C.relief D.worry
42.A.accident B.event C.issue D.topie
43.A.did up B.made up C.put up D.lined up
44.A.safely B.heavily C.thinly D.gently
45.A.seemed B.used C.happened D.had
46.A.dragged B.drawn C.pulled D.pushed
47.A.mass B.residents C.crowd D.team
48.A.moment B.period C.while D.date
49.A.concentration B.collection C.combination D.cleassification
50.A.so B.but C.or D.as
51.A.never B.ever C.even D.still
52.A.starting B.lasting C.finishing D.running
53.A.unconcerned B.unconscious C.unkinown D.unnoticed
54.A.offered B.told C.announced D.beard
55.A.stuck B.involved C.lost D.spread
第三部分:阅读理解(共20小题;每小题2分,满分40分)
阅读下列短文,从每题所给的四个选项(A、B、C和D)中,选出最佳选项,并在答题卡上将该项涂黑。
(A)
I first went to Harrow in the summer term. The school had the biggest swimming pool I had ever seen. It was a good joke to come up behind a naked boy, and push him into the pool. I made quite a habit of this with boys of my own size or less.
One day I saw a boy wrapped in a towel on the side of the pool. He was no bigger than I was, so I thought him a fair game. Coming secretly behind,I pushed him in, holding on to his towel so that it would not get wet, I was surprised to see an angry face come out from the water, and a being of great strength masking its way by face strokes (猛力地划)to the shore. I fled, but in vain. He overtok me, seized me violently, and threw me into the deepest part of ther pool. I soon climbed out on the other side, and found myself surrounded by a crowd of younger boys.“Do you know what you have done?”they said,“It’s Amery; he is in Grade Six. He is champion at gym,he has got his football honor.”
I was frightened and felt ashamed. How could I tell his position when be was wrapped in a bath towel and so small.”He didn’t seem pleased at all, so I added in a most brilliant word,“My father, who is a great man, is also small.”At this be laughed, and after some general words about my rude behavior and how I had better be careful in the future, signified the incident was closed.
56.The writer thought Amery“a fair game”because the boy .
A.looked like an animal B.was fond of games
C.was of similar size D.was good at sports
57.The writer felt“ashamed” because .
A.he was laughted at by other boys B.Amery turned out to be in the same grade
C.he pushed Amery hard and hurt him D.he played a joke on an outstanding athlete
58.By saying “My father, who is a great man, is also small”, the write .
A.tried to please Amery B.challenged Amery
C.threatened Amery D.admired his father
59.Which of the following is TRUE?
A.The writer could run faster than Amery.
B.The writer liked playing on boys of all sizes.
C.Amery was a student in Grade Four.
D.Amery forgave the writer for his rude behavior.
(B)
Read the advertisem*nts carefully. Then answer the questions that follow.
LNTERESTED IN CHILDCARE?
Qualified person or preschool teacher needed for busy childcare center, south of the river. Full-time work guaranteed. Immediate start necessary. The candidate must be able to work as a co-operative team member. Phone 6345 2345 for an interview and fax resume to 6345 2345.
WE NEED OFFICE CLRANERS!
Three people are required for professional cleaning in the CBD area. Working hours from 5:00 pm on Mondays, Wednesdays & Fridays. Appcoximately five hours per shift. A good record is necessary. Experience preferred. Phone 6345 7843 now.
WANT TO WORK AS A DENTAL NURSE?
This is an exciting opportunity for a qualified dental nurse with a confident and cheerful personality to work in the School Dental Health Schemae. You must be able to get along well with children because wrok involves talking to groupe about dental health practices. For furtget information, visit our website a:schooledntalservices@ gov. sg. And fax your resume to 6234 4567.
FLORLST WANTED!
Are you a creative and trained florist with at least two years of experience? Then this may be just what you are seeking. The city’s leading Florist Artist Studio needs another part-time member on their wedding team. Please phone 6098 7888 now!
60.You may find the above advertisem*nts .
A.in a store window B.in a school magazine
C.in a newspaper D.on a company notice-board
61.The position of a(n) is a part-time job.
A.florist B.artist
C.preschool teacher D.dental nurse
62. Which of the following is the most important for the childcare job?
A.Cooking skills. B.Childcare experience.
C.The ability to start work in two weeks.D. D.The ability to work well with other people.
63.The office eleaners will be required to work about hours a week.
A.5 B.10 C.15 D.20
(C)
As a professor at a large American university, there is a phrase that I hear often from students:“I’m only a 1050.”The unlucky students are speaking of the score on the Seholastic Aptitude Test (SAT), which is used to determine whether they will be admitted to the college or university of their choice, or even have a chance to get a higher education at all. The SAT score, whether it is 800, 1 100 or 1550, has becomes the focus at this time of their life.
It is obvious that if students value highly their test scores, then a great amount of their self-respect is put in the number. Students who perform poorly on the exam are left feeling that it is all over. The low test score, they think, will make it impossible for them to get into a good college. And without a degree from a prestigious university, they fear that many of life’s doors will remain forever closed.
According to a study done in the 1990s, the SAT is only a reliable indicator of a student’s future performance in most cases. Interestingly, it becomes much more accurate when it is set together with other indicatorn——like a student’s high school grades. Even if standardieed tests like the SAT could show a steudnt’s academic profieiency(学业水平),they will never be able to test things like confidence, efforts and willpower, and are unable to give us the full picture of a student’s potentialities(潜力).This is not to suggest that we should stop using SAT scores in our college admission process. The SAT is an excellent test in many ways, and the score is still a useful means of testing students. However, it should be only one of many methods used.
64.The purpose of the SAT is to test students’ .
A.strong will B.academic ability
C.full potentialities D.confidence in school work
65.Students’ self-respect is influeneed by their .
A.seores in the SAT B.achievements in mathematic
C.job opportunities D.money spent on education
66.“A prestigious university”is most probably
A.a famous university B.a technical universtiy
C.a traditional university D.an expensive university
67.This passage is mainly about .
A.how to prepare for the SAT B.stress caused by the SAT
C.American higher education D.the SAT and its effects
(D)
Brian Walker chews pens. He bites them so hard that his boss has warned him to stop or buy his own. Kate’s weakness is more acceptable-she is unalbe to walk past a cake shop without overeating Sophin Cartier finds her cigarette habit a headache, while Alice’s thumb-sucking drives her boy friend crazy. Four people with very different habits, but they all share a common problem anxiety disorder or, in serious cases. Obessive Compulisive Disorder (OCD).
From nail-biting to too much hand-washing, overeating and internet addiction(上瘾),OCD is widespread in almost every workplace and countless home. “It is a relatively common form of nxiety,”says Dr. Mootee. “Tbe main feature of OCD is the repetitin of unwanted bosersive(过度的) thoughts such as worries that doors are left unlocked , gas or electrical appliances are left on.” In order to fight against the an esasy chedcking door locks and gas or electrical appliandces.
Dr. Mootee says that repetitive washing, particularlu of the hands, is the most common type of OCD. She hastreated many patients who wash their hands up to 30 times a day. The technique Mootee uses to treat people with OCD is called cognitive-behavioral therapy(认知行为疗法).“It is based on the general idea that people have the ability to change the way they think and behave,”says Mootee.
But when does a haibt become a problem?“It’s personal,”says Mootee.“Everyone has something unusual,but if you can’t put up with it, then it’s a problem and you need to do something to change it.”Mootee says many people reaist for treatmet because they fear they are“crazy”. But as people become more knoledgeable about these problems they will go and get help. The only way to coure is to coqnuer.
68.If a person suffers from OCD, be is likely to keep doing any of the following EXCEPT .
A.chewing pens B.hurting himself
C.sucking fingers D.biting nails
69.According to the passage, a person suffering from OCD .
A.reduces his/her anxiety by taking drugs
B.gets into unwanted hagbits to relieve stress
C.has unwanted thoughts about habits
D.has unwanted thoughts because of illness
70.Dr. Mootee’s treating lechnique is based ont eh idea that .
A.everyong has something different
B.people can put up with their problems
C.people can chag their way of thinking and action
D.people tend to repeat their obsessive actions
71.By saying “The only way to cure is to conquer.”Dr. Mootee suggests that an OCD sufferer .
A.has to be an extraordinary person
B.must cure his illness by himself
C.must overcome many physical illnesses
D.should have a right attitude towards the problem
(E)
The literal meaning of philosophy is “love of wisdom”. But this meaning does not tell us very much . Unlike the other discilines(学科),philosopjy cannot e defined by what you study ,because it si actually unlimited. Anything can be the subject matter of philosophy:are, history, law, language, literature, mathematics, and in fact, the other academic disciplines are directly related to philosopkhy. For this reason you get a Doctorate(博士学位)of Philosoophy (Ph. D.) in biochemisty, or computer science, or psychology.
Two broad sub-fields of philosophy are logic and the history of philosophy. Logic is the science of argument and eritical thinkging . It provides sound methods for distinguishing good from bad reasonign .The history of philosopjy involives the study of major philosophers and perrsiad in the development of philosophy.
Of what use is philosophy? First it is useful in educational advancemet . It is necessary for undesrtanding other disciplines. Only philosopjy questiong the nature of the concepts used in a discipline, and its relatin to other disco[;omes. And throught the stdy of philosophy, one develops sound methods of research and analysis that can be applied to any field.
There are a number of gengral uses of philosophy. It strengthens one’s ability to solve problesm, to communieate, to organize ideas and issues, to presuade, and to takewhat is the most important form a large quamity of data. These general uses are of great benefit in the career firld, not necessarily ofr boutaining onej’s first job after graduation ,but for preparing for posttions of responsibility, managemetn and leadershin later on. It is very shor sihte after all, to take a course of studies only for the purpose of getting one’s first job. The useful skills developed throught the study of philosophy hav significang long-term benefits in canreer advancement. No other discipline systematically follows the ideals of wisdom, leadership, and capacity to reasolve human conflict.
72.Accprdomg to Paragraph 1. philosophy can best be described as the study of .
A.social sciences B.natural sciences
C.both social and natural sciences D.the subject matter of politics
73.With the study of philosophy, you can .
A.become a great leader
B.succeed in everything
C.find a good job soon after graduation
D.make progress in your career development
74.According to the passage, which of the follwing statements is TRUE?
A.Philosophy is an independent discipline.
B.Logic helps you to become a better thinker.
C.The study of philosophy brings you immediate benefits.
D.The meaning of philosophy is too limited to define.
75.From the passage, we can conclude
A.not all the subjects have to do with philosophy
B.a person will get a Ph. D. if he/she studies philosophy
C.philosophy can be helpful for the study of any other subjects
D.philosophy is the only solution to all the probleras in the world
第二卷(非选择题 35分)
第四部分:写作(共两节,满分35分)
第一节:对话填空(共10小题;每小题1分,满分10分)
   阅读下面对话,掌握其大意,并根据所给字母的提示,在标有题号的右边横线上写出一个英语单词的完整、正确形式,使对话通顺。
Travel Agent——T; Paul——P
T:Good afternoon, how can I help you?
P:Hi, I want to (76) b a room and some tickets for the summer holiday.
T:Where are you going?
P:Sydney.
T:And how many people will be going?
P:Four. Two(77)a and two children.
T:(78)W would you like to go?
P:Anytime after 12th July,but(79)b 20th July. Can you tell me the(80)p of the cheapest flight?
T:Sure, can I ask if your children are over two years old?
P:Yes, they are.
T:In that(81)c ,for the flight they will be charged at children’s rates and for the hotel there won’t be any charge. It(82) l on 15th July and returns on 29th July on Virgin Atlantic. This is the cheapest flight(83)a .
P:Fine. Do you have a (84)c of the journey plan that I can take away with me?
T:Sure. I hope to hear from you soon.
P:Thanks. I’ll think about it tonight and ring you in the (85)m tomorrow.
T:Thanks. I’m looking forward to your reply.
第二节 书面表达(满分25分)
你所在的班级将要举办一次主题为“What Can We Do for Our School?”的英语演讲比赛。请写一篇英语演讲稿,要求从以下四方面中任选一至二个进行阐述,并举例加以说明。
1.关心他人;
2.美化校园;
3.受护学校设施;
4.营造良好的学习氛围。
注意:
1.不要在书面表达中出现所在学校的校名和本人姓名,否则本节判为零分;
2.词数:100左右;
3.参考词汇:
爱心——love and care;校园——campus; 设施——facilities; 氛围——environment
4.开头和结尾已经写好,且不记入词数。
Hello, everyone. It’s nice to speak about what we can do for our school, and I think each of us can do something.
Thank you for listening!
英语试题参考答案
第一卷(选择题 共115分)
第一部分:听力理解
1.B 2.A 3.B 4.A 5.C 6.C 7.A 8.C 9.B 10.B 11.A 12.C 13.A 14.B 15.B 16.C
17.A 18.B 19.C 20.C
第二部分:英语知识运用(共两节,满分45分)
第一节:单项填空(共15小题;每小题1分,满分15分)
21.B 22.A 23.C 24.D 25.B 26.A 27.D 28.B 29.B 30.B 31.C 32.D 33.C
34.A 35.D
第二节:完形填空(共20小题;每小题1.5分,满分30分)
36.C 37.B 38.D 39.A 40.B 41.D 42.B   43.D  44.D  45.A 
46.D  47.C  48.A  49.C  50.B  51.A  52.C 53.B 54.D 55.C
第三部分:阅读理解(共20小题;每小题2分,满分40分)
56.C 57.D 58.A 59.D 60.C 61.A 62.D 63.C 64.B
65.A 66.A 67.D 68.B 69.B 70.C 71.D 72.C 73.D 74.B 75.C
第二卷(非选择题 共35分)
第四部分:写作(共两节,满分35分)
第一节:对话填空(共10不题;每小题1分,满分10分)
76.book 77. adults 78.When 79.before 80.price 81.case
82.leaves 83.available 84.copy 85.morning
第二节 书面表达(满分25分)
One Possible Version
Hello, everyone. It’s nice to speak about what we can do for our school, and I think each of us
can do somethng.
The first thing we can do is to make our campus more beautiful. Every one of us may plant a
tree in the school, or organize a thorough cleaning on the campus. We can also form a good habit
of putting the rubbish in the trashcan.
The second thing we can do is to protect the facilities in our school. When we leave the
classroom, we should never forget to turn off the light or close the door. We should not leave the
tap water running or waste any materials in the laboratory class.
If everyone does one thing for our school, I believe we will turn our school into a better place to
Study and live in.
Thank you for listening!
2007年高考英语试题及答案浙江卷 
英语试题
第Ⅰ卷(共100分)
第一部分:英语知识运用(共两节,满分50分)
第一节:单项填空(共20小题;每小题1分,满分20分)
从A、B、C、D四个选项中,选出可以填入空白处的最佳选项,并在答题纸上将该选项标号涂黑。
1. --- Jack bought a new mobile phone the other day.
--- ________? That’s his third one in just one month.
A. Had he B. Did he C. Does he D. Has he
2. It is reported that the floods have left about _________ people homeless.
A. two thousand B. two-thousands C. two thousands D. two thousands of
3. --- You should apologize to her, Barry.
--- ______, but it’s not going to be easy.
A. I suppose so B. I feel so C. I prefer to D. I like to
4. I like _____ color of your skirt. It is _____ good match for your blouse.
A. a; the B. a; a C. the; a D. the; the
5. Why not try your lick downtown, Bob? That’s ______ the best jobs are.
A. where B. what C. when D. why
6. Work gets done ______ when people do it together, and the rewards are higher too.
A. easily B. very easy C. more easily D. easier
7. --- Look! He’s running so fast!
--- Hard to _______ his legs were once broken.
A. know B. imagine C. realize D. find
8. Many of them turned a deaf ear to his advice, _______ they knew it to be valuable.
A. as if B. now that C. even though D. so that
9. The children talked so loudly at dinner table that I had to struggle ______.
A. to be heard B. to have heard C. hearing D. being heard
10. It _____ we had stayed together for a couple of weeks ____ I found we had a lot in common.
A. was until; when B. was until; that C. wasn’t until; when D. wasn’t until; that
11. We firmly believe that war never settles anything. It only _____ violence.
A. runs into B. comes from C. leads to D. begins with
12. Of the seven days in the a week, Saturday is said to be the most popular ____ for a wedding in some countries.
A. way B. situation C. event D. choice
13. Would you please ______ this form for me to see if I’ve filled it in right?
A. take off B. look after C. give up D. go over
14. Chan’s restaurant on Baker Street, ______ used to be poorly run, is now a successful business.
A. that B. which C. who D. where
15. --- My cat’s really fat.
--- You ______ have given her so much food.
A. wouldn’t B. couldn’t C. shouldn’t D. mustn’t
16. --- He got his first book published. It turned out to be a bestseller.
--- When was _____?
--- _____ was in 2000 when he was still in college.
A. that; This B. this; It C. it; This D. that; It
17. Professor James will give us a lecture on the Western culture, but when and where ____ yet.
A. hasn’t been decided B. haven’t decided C. isn’t being decided D. aren’t decided
18. The play had already been on for quite some time when we ____ at the New Theatre.
A. have arrived B. arrived C. had arrived D. arrive
19. The open-air celebration has been put off _______ the bad weather.
A. in case of B. in spite of C. instead of D. because of
20. ______ by a greater demand of vegetables, farmers have built more green houses.
A. Driven B. Being driven C. To drive D. Having driven
第二节:完形填空(共20小题;每小题1.5分,满分30分)
阅读下面短文,掌握其大意,然后从21–40各题所给的四个选项(A、B、C和D)中,选出最佳选项,并在答题纸上将该选项标号涂黑。
The books in David’s schoolbag felt like bricks as he ran down the street. What he wanted to do was to play basketball with Eric, 21 his mother told him he would have to return his sister’s books to the library first.
He had 22 se foot in a library and he wasn’t about to do so today. He would just 23 the books in the outside return box. But there was a 24 ; it was locked.
He went into the building, only a few minutes 25 closing time. He put the books into the return box. And after a brief 26 in the toliet, he would be on his way to the playground to 27 Eric.
David stepped out of the toliet and stopped in 28 — the library lights were off. The place was 29 . The doors had been shut. They 30 be opened from the inside, he was trapped(被困) — in a library!
He tried to 31 a telephone call, but was unable to 32 . What’s more, the pay phones were on the outside of the building. 33 the sun began to set, he searched for a light and found it.
34 he could see. David wrote on a piece of paper: “ 35 ! I’m TRAPPED inside!” and stuck it to the glass door. 36 , someone passing by would see it.
He was surprised to discover that this place was not so unpleasant, 37 . Rows and rows of shelves held books, videos and music. He saw a book about Michael Jordan and took it off the shelf. He settled into a chair and started to 38 .
He knew he had to 39 , but now, that didn’t seem to be such a 40 thing.
21. A. but B. because C. or D. since
22. A. ever B. nearly C. never D. often
23. A. pass B. drop C. carry D. take
24. A. problem B. mistake C. case D. question
25. A. during B. after C. over D. before
26. A. rest B. break C. walk D. stop
27. A. visit B. meet C. catch D. greet
28. A. delight B. anger C. surprise D. eagerness
29. A. lonely B. empty C. noisy D. crowded
30. A. wouldn’t B. shouldn’t C. couldn’t D. needn’t
31. A. make B. fix C. use D. pick
32. A. get on B. get up C. get through D. get in
33. A. If B. As C. Though D. Until
34. A. On time B. Now and then C. By the way D. At last
35. A. Come B. Help C. Hello D. Sorry
36. A. Surely B. Thankfully C. Truly D. Gradually
37. A. at most B. after all C. in short D. as usual
38. A. watch B. paly C. read D. write
39. A. wait B. stand C. sleep D. work
40. A. bad B. cool C. strange D. nice
第二部分:阅读理解(第一节20小题;第二节5小题;每小题2分,满分50分)
第一节:阅读下列短文,从每题所给的四个选项(A、B、C和D)中,选出最佳选项,并在答题纸上将该选项标号涂黑。
A
If you were to walk up to Arthur Bonner and say, “ Hey, Butterfly Man,” his face would break into a smile. The title suits him. And he loves it.
Arthur Bonner works with the Palos Verdes blue butterfly(蝴蝶), once thought to have died out. Today the butterfly is coming back — thanks to him. But years ago if you’d told him this was what he’d be doing someday, he would have laughed, “ You’re crazy.” As a boy, he used to be “ a little tough guy on the streets”. At age thirteen, he was caught by police stealing. At eighteen, he landed in prison for shooting a man.
“ I knew it had hury my mom,” Bonner said after he got out of prison. “ So I told myself I would not put my mom through that pain again.”
One day he met Professor Mattoni, who was working to rebuild the habitat(栖息地) for an endangered butterfly called El Segundo blue.
“ I saw the sign ‘ Butterfly Habitat’ and asked, ‘ How can you have a habitat when the butterflies can just fly away?’” Bonner recalls. “ Dr. Mattoni laughed and handed me a magnifying glass(放大镜) , ‘Look at the leaves.’ I could see all these caterpillars(蝴蝶的幼虫) on the plant. Dr Mattoni explained, ‘ Without the plant, there are no butterflies.’”
Weeks later, Bonner received a call from Dr. Mattoni, who told him there was a butterfly needed help. That was how he met the Palos Verdes blue. Since then he’s been working for four years to help bring the butterfly back. He grows astragalus, the only plant the butterfly eats. He collects butterflies and brings them into a lab to lay eggs. Then he puts new butterflies into the habitat.
The butterfly’s population, once almost zero, is now up to 900. For their work, Bonner and Dr. Mattoni receiced lots of awards. But for Bonner, he earned something more: he turned his life around.
For six years now Bonner has kept his promise to stay out of prison. While he’s bringing back the Palos Verdes blue, the butterfly has helped bring him back, too.
41. When he was young, Arthur Bonner _______.
A. broke the law and ended up in prison
B. was fond of shooting and hurt his mom
C. often laughed at people on the streeets
D. often caught butterflies and took them home
42. Bonner came to know the Palos Verdes blue after he _______.
A. found the butterfly had died out B. won many prizes from his professor
C. met Dr. Mattoni, a professor of biology D. collected butterflies and put them into a lab
43. From the last sentence of the text, we learn that raising butterflies has _________.
A. made Bonner famous B. changed Bonner’s life
C. brought Bonner wealth D. enriched Bonner’s knowledge
44. Which of the following would be the best title for the text?
A. A Promise to Mom B. A Man Saved by Butterflies
C. A Story of Butterflies D. A Job Offered by Dr. Mattoni
B
People who have lost the ability to understand or use words due to brain damage are called aphasics(失语症患者). Such patients can be extremely good at something else. From the changing expressions on speakers’ faces and the tones of their voices, they can tell lies from truths.
Doctors studying the human brain have given a number of examples of this amazing power of aphasics. Some have even compared this power to that of a dog with an ability to find out the drugs hidden in the baggage.
Recently, scientists carried out tests to see if all that was said about aphasics was true. They studied a mixed group of people. Some were normal; others were aphasics. It was proved that the aphasics were far ahead of the normal people in recognizing false speeches — in most cases, the normal people were fooled by words, but the aphasics were not.
Some years ago, Dr. Oliver Sacks wrote in his book about his experiences with aphasics. He mentioned a particular case in a hospital. Some aphasics were watching the president giving a speech on TV. Since the president had been an actor earlier, making a good speech was no problem for him. He was trying to put his feelings into every word of his speech.
But his way of speaking had the opposite effect on the patients. They didn’t seem to believe him. Instead, they burst into laughter. The aphasics knew that the president did not mean a word of what he was saying. He was lying!
Many doctors see aphasics as people who are not completely normal because they lack the ability to understand words. However, according to Dr. Sacks, they are more gifted than normal people. Normal people may get carried away by words. Aphasics seem to understand human expressions better, though they cannot understand words.
45. What is so surprsing about aphasics?
A. They can fool other people. B. They can find out the hidden drugs.
C. They can understand language better. D. They can tell whether people are lying.
46. How did the scientists study aphasics?
A. By asking them to watch TV together. B. By organizing them into acting groups.
C. By comparing them with normal people. D. By giving them chances to speak on TV.
47. What do we learn from this text?
A. What ones says reflects how one feels.
B. Aphasics have richer feelings than others.
C. Normal people often tell lies in their speeches.
D. People poor at one thing can be good at another.
C
Below is a web page from Google.
Olympic—Modern Olympic Games
The completer results archive of summer and winter Olympic games, with winners lists, statistics, national anthems and flags of all contries since 1896.
www. olympic. it/english/home-16k
Environment
Beijing steel plants to run at the lowest level of cost during 2008 Olympics [2007-03-11] Beijing able to treat 90 pct of waste water [2007-02-05] ·Air quality in Beijing has improved over the past six years [2007-02-01]·Green Olympics dream coming true …
en. Beijing2008.com/80/67/column211716780.shtml-52k
Beijing announces planned roule of Olympic torch relay
Beijing announced the 2008 Olympic Games torch relay route and set off the Olympic Games torch on Thursday. … Green Olympics is one of the three concepts of the Beijing Games. …
english.china.com/zh_cn/news/sports/110592227/14069663.html-26k
Environmental Symbol of Beijing Olympics —‘Green Olympics’
Environmental Symbol of Beijing Olympics —‘Green Olympics’, officially announced on Saturday, 24 September 2005. The symbol, created using a calligraphic art form, is composed of human and tree-like shapes, …
my.opera.com/green_head/blog/show.dml/92155-18k
Green Olympics Forum In Beijing 2004
On behalf of BOCOC, Wang Wei, executive vice president of BOCOC, gave a presentation about Green Olympics, introduced the environment protection work of BMC and BOCOC, and answered several questions such as protection of cultural relics …
en.beijing2008.com/84/91/article211929184.shtml-36k
Olympic Games Quizzes and Olmpic Games Trivia
Who was the founder of the Modern Olympics? In which cities were the modern Olympic Games due to be held/ scheduled during the ‘war years’, i.e., 1916, 1940, and 1944? … In which three years of the Modern Olympic Games were … More questions …
www.funtrivia.com/quizzes/sports/ olympic_games.html-12k
OFFICIAL WEBSITE OF THE OLYMPIC MOVEMENT
… aimed at young researchers engaged in scholarly research on the Olympic Movement, its history and values, and the impact of the …[ Full story] SPORT ACCORD[2005-01-23]…
www.olympic.org/
Ancient Olympic Hertiage and Modern Olympic Games in Athens 2004
As put forward in the various official texts, the Olympic symbols of ancient Olympia, the Olympic flame and the Marathon race are bridges between the ancient and the modern Olympic Games, …
www.c2008.org/rendanews/knowledge_detail.asp?id=911-106k
48. When was the environmental symbol of Beijing Olympics made public?
A. On January 23, 2005. B. On September 24, 2005.
C. On February 5, 2007. D. On March 11, 2007.
49. Which of the following websites is designed for young researchers interested in the Olympics?
A. Environment
B. Green Olympics Forum In Beijing 2004
C. Olympic Games Quizzes and Olmpic Games Trivia
D. OFFICIAL WEBSITE OF THE OLYMPIC MOVEMENT
50. Which website provides a complete list of the winners in the Olympic Games?
A. Olympic—Modern Olympic Games
B. Green Olympics Forum In Beijing 2004
C. Environmental Symbol of Beijing Olympics —‘Green Olympics’
D. Ancient Olympic Hertiage and Modern Olympic Games in Athens 2004
51. What do we learn from the above web page?
A. Beijing has made efforts to improve its air quality.
B. The Olympic torch relay route has not been planned.
C. The Olympic flag will be on display in Athens.
D. A quiz on the Olympics will be held in Beijing.
D
Tell a story and tell it well, and you may open wide the eyes of a child, open up lines of communication in a business, or even open people’s mind to another culture or race.
People in many places are digging up the old folk stories and the messages in them. For example, most American storytellers get their tales from a wide variety of sources, cultures, and times. They regard storytelling not only as a useful tool in child education, but also as a meaningful activity that helps adults understand themselves as well as those whose culture may be very different from their own.
“ Most local stories are based on a larger theme,” American storyteller Opalanga Pugh says, “ Cinderella(灰姑娘), or the central idea of a good child protected by her goodness, appears in various forms in almost every culture of the world.”
Working with students in schools, Pugh helps them understand their own cultures and the general messages of the stories. She works with prisoner too, helping them knowing who they are by telling stories that her listeners can write, direct, and act in their own lives. If they don’t like the story they are living, they can rewrite the story. Pugh also works to help open up lines of communication between managers and workers. “For every advance in business,” she says, “ there is a greater need for communication.” Storytelling can have a great effect on either side of the manager-worker relationship, she says.
Pugh spent several years in Nigeria, where she learned how closely storytelling was linked to the everyday life of the people there. The benefits of storytelling are found everywhere, she says.
“I learned how people used stories to spread their culture,” she says, “ What I do is to focus on the value of the stories that people can translate into their own daily world of affairs. We are all storytellers. We all have a story to tell. We tell everybody’s story.”
52. What do we learn about American storyteller from Paragraph 2?
A. They share the same way of storytelling.
B. They prefer to tell the stories from other cultures.
C. They learn their stories from the American natives.
D. They find storytelling useful for both children and adults.
53. The underlined sentence (Paragraph 4) suggests that prisoners can _____.
A. start a new life B. settle down in another place
C. direct films D. become good actors
54. Pugh has practised storytelling with _____ groups of people.
A. 2 B. 3 C. 4 D. 5
55. What is the main idea of the text?
A. Storytelling can influence the way people think.
B. Storytelling is vital to the growth of business.
C. Storytelling is the best way to educate children in school.
D. Storytelling helps people understand themselves and others.
E
I began working in journalism(新闻工作)when I was eight. It was my mother’s idea. She wanted me to “make something” of myself, and decided I had better start young if I was to have any chance of keeping up with the competition.
With my load of magazines I headed toward Belleville Avenue. The crowds were there. There were two gas stations on the corner of Belleville and Union. For several hours I made myself highly visible, making sure everyone could see me and the heavy black letters on the bag that said THE SATURDAY EVENING POST. When it was suppertime, I walked back home.
“ How many did you sell, my boy?” my mother asked.
“ None.”
“ Where did you go?”
“ The corner of Belleville and Union Avenues.”
“ What did you do?”
“ Stood on the corner waiting for somebody to buy a Saturday Evening Post.”
“ You just stood there?”
“ Didn’t sell a single one.”
“ My God, Russell!”
Uncle Allen put in, “ Well, I’ve decided to take the Post.” I handed him a copy and he paid me a nickle(五分镍币). It was the first nickle I earned.
Afterwards my mother taught me how to be a salesman. I would have to ring doorbells, address adults with self-confidence(自信), and persuade them by saying that no one, no matter how poor, could afford to be without the Saturday Evening Post in the home.
One day, I told my mother I’d changed my mind. I didn’t want to make a success in the magazine business.
“ If you think you can change your mind like this,” she replied, “ you’ll become a good-for-nothing.” She insisted that, as soon as school was over, I should start ringing doorbells, selling magazines. Whenever I said no, she would scold me.
My mother and I had fought this battle almost as long as I could remember. My mother, dissatisfied with my father’s plain workman’s life, determined that I would not grow up like him and his people. But never did she expect that, forty years later, such a successful journalist as me would go back to her husband’s people for true life and love.
56. Why did the boy start his job young?
A. He wanted to be famous in the future.
B. The job was quite easy for him.
C. His mother had high hopes for him.
D. The competiton for the job was fierce.
57. From the dialogue between the boy and his mother, we learn that the mother was _______.
A. excited B. interested C. ashamed D. disappointed
58. What did the mother do when the boy wanted to give up?
A. She forced him to continue. B. She punished him.
C. She gave him some money. D. She changed her plan.
59. What does the underlined phrase “this battle”(last paragraph) refer to?
A. The war between the boy’s parents.
B. The arguing between the boy and his mother.
C. The quarrel between the boy and his customers.
D. The fight between the boy and his father.
60. What is the text mainly about?
A. The early life of a journalist.
B. The early success of a journalist.
C. The happy childhood of the writer.
D. The important role of the writer in his family.
第二节:Cathy、Jason、Kate、Wayne和Frank各自打算订一份杂志。第61至65题是他们的个人情况介绍。
阅读下面六种杂志的简介(A、B、C、D、E和F),选出符合各人需要的最佳选项,并在答题纸上将该选项标号涂黑。选项中有一项是多余选项。
61. A young mother raising her first child, Cathy tries to spend as much time as she can palying with her baby girl. She is eager to get more knowledge of child development and to better understand how young children look at the world.
62. Jason is a successful young businessman. He travels widely and enjoys adventures and challenges. Having been to many different countries, Jasin is still looking for new places to visit. His biggest dream is to see every corner of the world before he retires.
63. Smart and pretty, Kate is very popular among her friends. She takes great care of how she looks and has a great interest in fashion and the lifestyle of famous people. Besides, she also enjoys reading articles written especially for young girls.
64. Wayne teaches science in a middle school. He not only wants his students to know more about the modern science, but also tries hard to develop a curiosity about nature among his students. He is gathering materials for his students to read.
65. Frank is the father of a 10-year-old boy. He believes that in this fast-developing world parents should prepare their children fro lifelong learning at an early age. Therefore, in his opinion, the most important skill for children to learn is to discover things on their own. He is looking for a magazine for his son.
A
Seventeen
Seventeen provides advice and encouragement for masses of young girls. Although its primary focus is fashion and famous people, this teen magazine offers more. Mixed among the latest styles, you’ll find short but interesting articles. Topics range from beauty to the risks of drinking.
Price: $22.95 ($3.83/issue)
B
Muse
Muse seeks to stimulate, delight, and challenge every curious child between the ages of 13 and 16. Produced by the editors and publishers of Cricket, Muse features articles on space, genetics, lasers, rain forests, computers, physics, math, earth sciences, and almost everything else in the universe.
Price: $32.97 ($3.66/issue)
C
WONDERTIME
WONERTIME is a new magazine from the editors of FamilyFun, devoted to helping parents develop children’s love of learning. A blend of how and why, WONDERTIME inspires parents to see the world through the eyes of their children. WONDERTIME mixes playful activities with scientific knowledge about how children develop physically, socially, intellectually, creatively, and emotionally.
Price: $10.00 ($1.00/issue)
D
National Geographic Adventurer
A new magazine from National Geographic, National Geographic Adventurer is intended for a generation of active men and women who seek new & challenging ways to explore and experience the world. The magazine offers an exciting mix of great photography and interesting articles. Features focus on best adventure stories, travel destinations, sports and more.
Price: $14.95 ($1.50/issue)
E
Familyfun
What does family fun mean to you? Crafts? Recipes? Party ideas? Familyfun magazine dishes up these and more interesting activities in over 180 colorful pages. Aimed at parents with young children, this energetic magazine promises to enrich the lives of families. Reviews of computer games, books, videos, and DVDs, written by experts, help parents feel up to the minute.
Price: $9.95 ($0.99/issue)
F
Kids Discover
To help children aged 6 to 12 become lifelong learners, each issue of Kids Discover looks into a single topic in science or social studies which children of this age fropu have a great interest in. The combination of exciting color photography, lovely pictures and informative kif-friendly text engages readers with attractive content.
Price: $26.95 ($2.25/issue)
第Ⅱ卷(共50分)
第三部分:写作(共三节,满分50分)
第一节:单词拼写(共10小题;每小题1分,满分10分)
根据下列句子及所给单词的首字母,在答题纸上按题号写出各单词的正确的完全形式(每空限写一词)。
66. People with a poor memory often f________ things.
67. Spring is a s_________ when trees turn green and flowers open.
68. Don’t always d__________ on others. You should learn to rely on yourself.
69. George was born on June 1, so Children’s Day is also his b_____________.
70. With no adults around, it’s d__________ for children to swim in the river.
71. He caught a high f__________. His body temperature was as high as 40℃.
72. There are only five minutes left. We’ll have to walk q__________ to get there on time.
73. He is always happy and never w_________ about anything.
74. They offered me a ticket for the concert, and I a__________ it with delight.
75. The photo will r_______ me of the days when we were together.
第二节:短文改错(共10小题;每小题1分,满分10分)
此题要求改正所给短文中的错误。对标有题号的每一行作出判断:如无错误,在该行右边横线上画一个钩(√);如有错误(每行只有一个错误),则按下列情况改正:
此行多一个词:把多余的词用斜线(\)划掉,在该行右边横线上写出该词,并也用斜线划掉。
此行缺一个词:在缺词处加一个漏词符号(∧),在该行右边横线上写出该加的词。
此行错一个词:在错的词下划一横线,在该行右边横线上写出改正后的词。
原行没有错的不要改。
I had a interesting dream last night. I dreamed 76. _______________
that I took part in a race. At first, I could not to run very 77. _______________
fast and fell behind. So I didn’t lose heart and kept 78. _______________
running. All the students on the playground cheer me on, 79. _______________
? Come on!? I was so encouraged that I ran faster and fast 80. _______________
till I caught up all the other runners. I felt as if flying like 81. _______________
a superman. In the end, I got to the finishing line first. 82. _______________
I won the race. I felt very proudly of myself. Many of my 83. _______________
classmate threw me up into the air. Just at that time 84. _______________
I woke up and found me still in bed! 85. ________________
第三节:书面表达(满分30分)
在英语课堂上,你喜欢你的老师授课时只用英语,还是英语、汉语兼用?某英语杂志社就此话题邀请中学生发表看法。请你围绕“How do you prefer your English classes to be taught? In English only, or in bothe English and Chinese?”这个问题,参考所给要点,选择一种授课形式,写一篇英语短文。
授课形式一:只用英语
授课形式二:英语、汉语兼用
优点:有助于提高听说能力等……
缺憾:不易听懂……
结论:……
优点:易于理解等……
缺憾:英语氛围不浓等……
结论:……
注意:1. 词数100~120,短文开头已给出(不计词数)
2. 参考词汇:atmosphere 氛围
I prefer my English classes to be taught in …
英语试题参考答案
第一部分:
第一节(20分,每小题1分)
1-5: B A A C A 6-10: C B C A D 11-15: C D D B C 16-20: D A B D A
第二节(30分,每小题1.5分)
21-25: A C B A D 25-30: D B C B C 31-35: A C B D B 36-40: A B C A A
第二部分(50分,每小题2分)
41-45:A C B B D 46-50: C D B D A 51-55: A D A B D 56-60: C D A B A
61-65: C D A B F
第三部分
第一节(10分,每小题1分)
66. forget 67. season 68. depend 69. birthday 70. dangerous
71. fever 72. quickly 73. worries 74. accepted 75. remind
第二节(10分,每小题1分)
I had a interesting dream last night. I dreamed 76. an
that I took part in a race. At first, I could not to run very 77. to
fast and fell behind. So I didn’t lose heart and kept 78. But
running. All the students on the playground cheer me on, 79. cheered
“ Come on!” I was so encouraged that I ran faster and fast 80. faster
till I caught up∧ all the other runners. I felt as if flying like 81. with
a superman. In the end, I got to the finishing line first. 82. √
I won the race. I felt very proudly of myself. Many of my 83. proud
classmate threw me up into the air. Just at that time 84. classmates
I woke up and found me still in bed! 85. myself
第三节、参考样例

I prefer my English classes to be taught in English only. As we all know, a good learning environment is vital if we want to study Englishe well. Classes taught in English provide students with such an environment. When English is the only language used in the classroom, students will have more opportunities to practise listening and speaking. Therefore, they will be able to learn more quickly.
However, there are also some problems with this teaching method. As we haven?t mastered enough English words, sometimes we may find it difficult to follow the teacher. Slow learner may even lose interest in English.
Practice makes perfect. To learn English well requires a lot of practice. Therefore, I like my English classes to be taught completely in English.

I prefer my English classes to be taught in both English and Chinese. In my opinion, learning English is a step-by-step process. As middle school students, we only have a small vocabulary. If the teacher can use some Chinese in an English class, we will understand the lesson better.
Of course, an English class taught in two languages has its shortcomings. Such a class cannot create a good atmosphere for English learning. Some students will speak Chinese all the time and cannot learn English well.
If you want to learn a lesson well, you have to understand it first. To help students understand the lesson better, the teacher should use Chinese from time to time. Therefore, I like my teacher to use both English and Chinese in teaching.
绝密★启用前
2007年普通高等学校招生全国统一考试(湖北卷)
英 语
本试卷共16页,全卷满分150分,考试用120分钟。
★祝考试题顺利★
注意事项:
1.答题前,考生务必将自己的姓名、准考证号填写在试题卷和答题卡上,并将准考证号条形码粘贴在答题卡上的指定位置。
2.选择题在每小题选出答案后,用2B铅笔把答题卡上对应题目的答案标号涂黑;完成句子和书面表达题用0.5毫米黑色签字笔直接答在答题卡上相对应的区域内,答在试题卷上无效。
3.考试结束,请将本试题卷和答题卡一并上交。
第一部分:听力(共两节,满分30分)
做题时,先将答案划在试卷上,录音内容结束后,你将有两分钟的时间将试卷上的答案转涂到答题卡上。
第一节(共5小题;每小题1.5分,满分7.5分)
听下面5段对话。每段对话后有一个小题,从题中所给的A、B、C三个选项中选出最佳选项,并标在试卷的相应位置,听完每段对话后,你都有10秒钟的时间来回答有关小题和阅读了一小题,每段对话仅读一遍。
例:How much is the shirt?
A.$ 19.15. B.$9.15 C.$9.18
答案是B.
1.Why does the woman want to go to America?
A.To take language courses.
B.To atrend a conference.
C.To visit some friends.
2.What do know about Peter Smith?
A.He is hoving lunch a home.
B.He is out at the moment.
C.He is talking with Mary;
3.What is said about the woman?
A.She spends more than rheearns.
B.she spends more than rheearns.
C.She has a lght bidgt
4.What do we know about the speakers?
A.They are know traveling in Mexico.
B.They have been to a festivalin Mexico.
C.They painted some pictures in Mexico.
5.What is the woman doing?
A.Helping children with AIDS.
B.Raising money for African children.
C.Collecting information on African children.
第二节(共15小题;每小题4.5分,满分22.5分)
听下面5段对话或独白。每段对话或独白后有几个小题。从题中所给的A、B、C三个选项中选出最佳选项,并标在试卷的相应位置。听每段对话或独白前,你将有时间阅读各个小题,每小题5秒钟;听完后,各小题将给出5秒钟的作答时间,每段对话或独白读两遍。
听第6段材料,回答第6、7题。
6.What was the man doing during the earthquake?
A.Shouting.
B.Running.
C.Sleeping.
7.How was the man when the earthquake took place?
A.He was ill.
B.He was helpless.
C.He was frightened.
听第7段材料,回答第8、9题。
8.Why does the boy love Sunday?
A.He can play basketball.
B.He has no classes at school.
C.He can watch sports on TV.
9.What is the boy expected to do?
A.To help with housework.
B.To watch a game on TV.
C.To have a good rest.
听第8段材料,回答第10至12题。
10.What is said about medical development in the furure?
A.Health care will be free.
B.Cancer may-be cured.
C.AIDS may disappear.
11.What will make distant places more popular for holiday?
A.Better air service.
B.Faster air transport.
C.Lower cost for air travel.
12.What is the topic of this interview?
A.Future ways of travelling.
B.Medical progress in the future.
C.Changes of life in the next dueade.
听第9段材料,回答13至16题。
13.Where is the woman going now?
A.To an art museum.
B.To a Chinese restaurant.
C.To an underground station.
14.Why does the woman come to the city?
A.For business.
B.For traveling.
C.For shopping.
15.Why does the man recommend the resthurant to the woman?
A.The service there is good.
B.The food there is tasty.
C.The price there is low.
16.According to the man, which is the best means of transport to the restaurant?
A.The bus.
B.The taxi.
C.The underground.
听第10段材料,回答第17至20题。
17.What kind of passage have you just heard?
A.Apublic lecture.
B.Aradio announcement.
C.Agovernment statement.
18.What is the main purpose of the event?
A.To increase people’s sense of environment protection.
B.To invite people to join an environmental organization.
C.To persuade families to have an outing in the mountains.
19.How many trees are going to be planted today?
A.700B.2000C.4000
20.What does the speaker advise volunteers to do?
A.To learn some tree-planting skills.
B.To come along any time they like.
C.To bring along their gloves.
第二部分:英语语言知识运用(共三节,满分55分)
第一节:多项选择(共10小题;第小题1分,满分10分)
从A、B、C、D四个选项中,选出可以填入空白处的最佳选项,并在答题卡上将该项涂黑。
例:To make members of a team perform better, the trainer first of all has to know their
A.strengths B.benefits C.techniques D.values
答案是A。
21.This magazine is very with young people, who like its content and style.
A.familiarB.popularC.similarD.particular
22.Emergency line opertors must always calm and make sure that they get all the information they need to send help.
A.growB.appearC.becomeD.stay
23.Despite such a big difference in towards what one eats, there is no doubt that people in the west regard the Chinese food as something special.
A.pointB.ideaC.attitudeD.sight
24.If the firms failed to make enough money, they would .
A.close downB.call offC.turn downD.set off
25.She devoted herself to her research and it earned her a good reputation in her field.
A.stronglyB.extremelyC.entirelyD.freely
26.People try to avoid public transportation delays by using their own cars, and this creates further problems.
A.in short B.in caseC.in doubtD.in turn
27.If we can our present difficulties, then everything should be all right.
A.come acrossB.get overC.come overD.get off
28.He began to take political science only when he left school.
A.strictlyB.trulyC.carefullyD.seriously
29.Surely it doesn’matter where the student associations get their money from; what is what they do with it.
A.countsB.appliesC.stressesD.functions
30.Water can absorb and give off a lot of heat without big changes in temperature, thus creating a
environment.
A.peacefulB.sensitiveC.commonD.stable
第二节:完成句子(共10小题;每小题1.5分,满分15分)
阅读下列各小题,根据括号内的汉语提示,用句末括号内的英语单词完成句子,并将答案写在答题卡上的相应题号后。
例:We (起床) before dawn. It was still dark outside.(get)
答案:got up
31.They sat together around the table, with (门关着),(shut)
32.I haven’t the slightest idea (他正在说什么).(talk)
34.Last night, John was answering the letters that (寄给他的) during the past two weeks.(arrive)
35.He believes that children (应允许……学习) at their own pace.(allow)
36.She has an excellent (对名字的记忆力), which helps her quite a lot in her work.( memory)
37. (他是否出过国)doesn’t make much difference.(he, abroad)
38.The factory’s output of cars this year is (大约是去年的三倍).(as, great)
39.Not only(要帮助)the disabled to find jobs, but also medical treatment will be provided for those who need it.(give)
40.It is possible that the King of Stonehenge was linked to the stones: he (可能参与) in planning the monument, or in helping transport and pull up the stones.(hand)
第三节:完形填空(共20小题;每小题1.5分,满分30分)
阅读下面短文,从短文后所给各题的四个选项(A、B、C和D)中,选出可以填入空白处的最佳选项,并在答题卡上将该项涂黑。
Many years ago,I owned a service station and roadhouse on the main road between Melbourne and Adelaide.
One very cold,wet night at about 3:30 a.m.,there was a 41 on the front door of our house.A young man,wet from 42 to toe,explained that he had 43 out of petrol about 30 km up the road.He had left his pregnant(怀孕的) wife and his two children 44 at the car and siad that he would hitchhike(搭便车) back.
Once I had 45 a can with petrol,I took him back to his car where his two-year-old and four-year-old children were both 46 ,saying that they were cold.Once the car had started,I suggested that he 47 me back.
Before leaving,I had turned the heater 48 in the roadhouse,so that when we went in,it was nice and 49 .While the little ones played and ran 50 ,I prepared bread and butter for the children,and hot chocolate for the 51 .
It was about 5 a.m. before they 52 .The young fellow asked me how much he 53 me and I told him that the petrol pump(加油泵) had 54 $15.He offered to pay “call-out fee”,but I wouldn’t accept it.
About a month later,I received a 55 from Interstate,a large bus company that we had been trying to 56 to stop off at our roadhouse for a long time.It 57 out that the young fellow I had helped was its general manager,the most 58 person in the company.
In his letter,he thanked me again and 59 me that,from then on,all their buses would stop at my service station.In this 60 ,a little bit of kindness was rewarded with a huge amount of benefits.
41.A.kickB.hitC.beatD.knock
42.A.fingerB.shoulderC.headD.hand
43.A.drivenB.usedC.comeD.run
44.A.awayB.behindC.overD.out
45.A.suppliedB.pouredC.equippedD.filled
46.A.sleepingB.cryingC.quarrellingD.fighting
47.A.allowB.ringC.leadD.follow
48.A.onB.offC.inD.over
49.A.neatB.hotC.warmD.attractive
50.A.aroundB.insideC.nearbyD.along
51.A.driversB.guestsC.customersD.adults
52.A.leftB.arrivedC.ateD.disappeared
53.A.gaveB.paidC.owedD.offered
54.A.appearedB.exhibitedC.calculatedD.shown
55.A.callB.letterC.checkD.notice
56.A.getB.forceC.requiteD.hope
57.A.pointedB.turnedC.workedD.found
58.A.generousB.successfulC.seriousD.powerful
59.A.praisedB.persuadedC.informedD.convinced
60.A.lessonB.businessC.aspectD.case
第三部分:阅读理解(共20小题;每小题2分,满分40分)
阅读下列短文,从每篇短文后所给各题的四个选项(A、B、C和D)中,选出最佳选项,并在答题卡上将该项涂黑。
A
Karen,grown up in a very traditional family in the western United States,maintained high moral(道德的) standars throughout her youth..In 1984,at the age of 23,she married Bill.They were blessed with two children,a boy and a girl.
By 1991 their love had deepened,and they were happy.Later that year,Bill developed a white spot on his tongue.He visited a doctor.
One day shortly after that,Bill called Karen to sit beside him.He said with tears in his eyes that he loved her and wanted to live forever with her.The doctor suspected that he had been infected with HIV,the virus that leads to AIDS.
The family was tested.Bill and Karen’s results were positive.Bill had become infected before he met Katen;then he passed the virns on to Karen.The children’s results were negative.Within three years,Bill was dead.“I don’t know how to express what it is like to watch the once handsome man you love and intend to live with forever dying slowly.I cried many nights.He died three months short of ten years of our marriage,”says Karen.Though a doctor told Karen that she would soon follow her husband into death,she is still alive.The infection has progressed to the early stages of AIDS.
Karen is but one of about 30 million people now living with HIV/AIDS,a figure larger than the combined populations of Australia.Ireland and Paraguay.According to one UN report,Africa has 21 million of these victims.By the turn of the century that number could reach 40 million and the disease will bring on the greatest disaster in human history.Of the wold’s sexually active adults aged 15 to 49,1 in 100 has already been infected with HIV.Of these,only 1 in 10 realizes that he or she is infected.In some parts of Africa,25 percent of the adults are infected.
Since the beginning of the spread of AIDS in 1981,about 11.7 million people have died of it.It is roughly calculated that in 1997 alone,about 2,3 million people died of it.Nevertheless,there are fresh reasons for optimism in the battle against AIDS.During the past few years,there has been a drop in new AIDS cases in wealthy nations.In addition,promising drugs hold out hope of better health and longer life.
61.By telling the story of Karen,the author intends to .
A.were people against high risk behaviors
B.stress the importance of medical tests
C.express sympathy for AIDS victims
D.show the consequences of AIDS
62.The underlined part in Paragraph 1 most probably means “”.
A.were lucky in havingB.were asked to adopt
C.regretted havingD.gave birth to
63.Bill was suspected of being infected with HIV after .
A.he got married to Karen
B.the family members were tested
C.Karen persuaded him to see the doctor
D.he found something wrong with his tongue
64.It can be concluded from the passage that .
A.promising drugs will soon stop AIDS
B.the spread of AIDS could be controlled
C.it is hopeless to win the battle against AIDS
D.the death rate of AIDS patients has been reduced
B
How can a creature weighing over 5 tons and normally taking 150 kilograms of food and 120 liters of water per day survive in a desert environment?
In the southwest African country of Namibia, and the Sahara lands of Mali further north, the desert elephant does just that.
Although not regarded as a separate species from the African elephant, the desert cousin differs in many ways. Their bodies are smaller, to absorb less heat, and their feet are larger for easier walking across sandy surfaces. They are taller, to reach higher branches. They have shorter tusks (象牙), and most importantly, longer trunks to dig for water in riverbeds.
Desert clephants can travel over 70 kilometers in search for feeding grounds and waterholes, and have a larger group of families, They drink only every 3-4 days, and can store water in a “bag” at the back of their throat, which is only used when badly needed. Desert elephants are careful feeders-they seldom root up trees and break fewer branches, and thus maintain what little food sources are available. Yong elephants may even eat the dung (粪便)of the female leader of a group when facing food shortage.
During drought they are unlikely to give birth to their young but with good rains the birthrate will increase greatly. Desert elephants have sand baths, sometimes adding their own urine (尿液) to make them muddy!
As we continue to overheat our weak planet, it can only be hoped that other animal species will adapt as extraordinarily well to change as the desert elephant.
65. The underlined part in Paragraph 2 means“ ”.
A.remains in the African countries
B.drinks 120 liters of water a day
C.manages to live in desert areas
D.eats 150 kilograms of food daily
66.Desert elephants are called careful feeders because they .
A.tarely ruin trees
B.drink only every 3-4 days
C.search for food in large groups
D.protect food sources for their young
67.The author answers the question raised in the first paragraph with .
A.stories and explanation
B.facts and descriptions
C.examples and conclusion
D.evidence and argument
68.What can be inferred from the last sentence in the passage?
A.Overheating the earth can be stopped.
B.Not all animal species are so adaptable.
C.The planet will become hotter and hotter.
D.Not all animals are as smart as desert elephants.
C
Almost every child is scared of something, from monsters in the cupboard to dogs in the park. But the fact that such fears are common and normal doesn’t mean they can be taken lightly. Kids experience fears and phobias (恐惧症) much more strongly than adults. And the influence of the fear can be physical as well as psychological (心理的). It can build up so they almost seem scared of everything-a kind of childhood anxiety. Dr. Creswell says:“Your child may always seem to expect the worst to happen and lack confidence in his or her ability to deal with any challenge.”So don’t make the same old mistake of treating them as if they’re silly for boing a“scaredy cat”. Handing the fears is essential.
Children can be born nervous and., if you have such a baby, you’ll tend to prevent them from getting worried. So if they fear dogs, You’ll keep them away from dogs, but in fact that can just confirm to the child that dogs are scary. What is worse, keeping your child away from what they fear can turn that feeling into a phobia. Instead, you should encourage them to get in touch with the thing they fear, in a safe and supportive environment. Dr. Andy Field, a researcher of childhood fears, says:“You should’t force, for example, a dog anxious child to go up to a dog. But you can approach it yourself, show them there is nothing to be afraid of, stroke (抚摸) it, and talk about the dog being friendly. Once your child dares to stroke a dog-one that’s good with children, of course-then you should encourage them to carry on until they feel calmer, and reward them for‘being brave’.”
69.Children’s fears are usually taken lightly because .
A.they will not develop into phobias
B.their influence is psychological
C.they exist widely in the world
D.they will disappear gradually
70.If we fail to help children to overcome fears, they will .
A.make the same old mistake
B.overcome them by themselves
C.experience the worst of things
D.grow up lacking self-confidence
71.According to Dr. Andy Field, if a child is afraid of a cat, parents should .
A.tell the child not to be afraid of it
B.show the child how to approach it
C.keep the child away from it
D.ask the child to stroke it
D
Over the last 70 years, researchers have been studying happy and unhappy peopke and finally found out ten factors that make a difference. Our feelings of well-being at any moment are determined to a certain degree by genes. However, of all the factors, wealth and age are the top two.
Money can buy a degree of happiness. But once you can afford to feed, clothe and house yourself, each extra dollar makes less and less difference.
Researchers find that, on average, wealthier people are happier. But the link between money and happiness is complex. In the past half-century, average income has sharply inereased in developed countries, yet happiness levels have remained almost the same. Once your basic needs are met, money only seems to increase happiness if you have more than your friends, neighbors and colleagues.
“Dollars buy status, and status makes people feel better,” conclude some experts, which helps explain why people who can seek status in other ways-scientists or actors, for example-may happily accept relatively poorly-paid jobs.
In a research, Professor Alex Michalos found that the people whose desires-not just for money, but for friends, family, job, health-rose furthest beyond what they already had, tended to be less happy than those who felt a smaller gap (差距)。Indeed, the size of the gap predicted happiness about five times better than income alone. “The gap measures just blow away the only measures of income.”says Michalos.
Another factor that has to do with happiness is age. Old age may not be so bad“Given all the problems of aging, how could the elderly be more satisfied?”asks Protessor Laura Carstensen.
In one survey, Carstensen in tervicwed 184 people between the ages of 18 and 94, and asked them to fill out an emotions questionnaire. She found that old people reported positive emotions just as often as young people, Some scientists suggest older people may expect life to be harder and learn to live with it, or they’re more realistic abour their time running out, older people have learned to focus on things that make them happy and let go of those that don’t.
“People realize not only what they have, but also that what they have cannot last forever,” she says. “A goodbye kiss to a husband or wife at the age of 85, for example, may bring far more complex emotional responses than a similar kiss to a boy or girl friend at the age of 20.”
72.According to the passage, the feeling of happiness .
A.is determined partly by genes
B.increases gradually with age
C.has little to do with wealth
D.is measured by desires
72.Some actors would like to accept poorly-paid jobs because the jobs .
A.make them feel much better
B.provide chances to make friends
C.improse their social position
D.satisfy their professional interests
74.Aged people are more likely to feel happy because they are more .
A.optimisticB.successfulC.practicalD.emortional
75.Professor Alex Michalos found that people feel less happy if .
A.the gap between reality and desire is bigger
B.they have a stronger desire for friendship
C.their income is below their expectation
D.the hope for good health is greater
E
lmagine you’re at a party full of strangers. You’re nervous. Who are these people? How do you start a conversation? Fortunately, you’ve got a thing that sends out energy at tiny chips in everyone’s name tag (标签). The chips send back name, job, hobbies, and the time available for meeting-whatever. Making new friends becomes simple.
This hasn’t quite happened in real life. But the world is already experiencing a revolution using RFID technology.
An RFID tag with a tiny ehip can be fixed in a produet, under your pet’s skin, even under your own skin. Passive RFID tags have no energy source-batteries because they do not need it. The energy comes from the reader, a scanning device(装置),that sends out energy (for example,radio waves)that starts up the tag immediately.
Such a tag carries information speeific to that object,and the data can be updated.Already, RFID technology is used for recognizing each car or truck on the road and it might appear in your passport. Doctors can put a tiny chip under the skin that will help locate and obtain a patient’s medical records. At a nightclub in Paris or in New York the same chip gets you into the VIP (very important person)section and pays for the bill with the wave of an arm.
Take a step back:10 or 12 years ago,you would have heard about the coming age of computing. One example always seemed to surfact: Your refrigerator would know when you needed to buy more milk. The comcept was that computer chips could be put everywhere and send information in a smart network that would make ordinary life simpler.
RFID tags are a small part of this phenomenon. “The world is going to be a loosely coupled set of individual small devices, connected wirelessly,”predicts Dr.J.Reich. Human right supporters are nervous about the possibilities of such technology. It goes too far tracking school kids through RFID tags, they say. We imagine a world in which a beer company could find out not only when you bought a beer but also when you drank it. And how many beers. Accompanied by how many biscuits.
When Marconi invented radio, he thought it would be used for ship-to-shore communication. Not for pop music. Who knows how RFID and related technologres will be used in the future. Here’s a wild guess:Not for buying milk.
76.The artiele is intended to .
A.warn people of the possible risks in adopting RFID technology
B.explain the benefits brought about by RFID technology
C.convince people of the uses of RFID technology
D.predict the applications of RFID technology
77.We know from the passage that with the help of RFID tags,people .
A.will have no trouble getting data about others
B.will have more energy for conversation
C.will have more time to make friends
D.won’t feel shy at parties any longer
78.Passive RFID tags chiefly consist of .
A.scanning devicesB.radio waves
C.batteriesD.chips
79.Why are some people worried about RFID technology?
A.Because children will be tracked by strangers.
B.Because market competition will become more fierce.
C.Because their private lives will be greatly affected.
D.Because customers will be forced to buy more products.
80.The last paragraph implies that RFID technology .
A.will not be used for such matters as buying milk
B.will be windely used, including for buying milk
C.will be limited to communication uses
D.will probably be used for pop music
第四部分:书面表达(满分25分)
假设你是某中学学生会主席李华,你校与本地一所国际学样经常举办联谊活动.你计划在重阳节组织学生到养老院去慰问老人,拟邀请国际学校的学生参加,请你根据以下内容要点给国际学样的学生会主席Tony写一封信.
要点:向老人赠送礼物(鲜花、自制贺卡……);
为老人提供服务(做清洁、陪老人聊天……);
为老人表演节目(唱歌、跳舞……).
注意:1、词数为100左右;
2、信的开关和结尾已为你写好(不计入你所写词数);
3、已给出的信的开关和结尾不得抄入答题卡.
(以下所给内容不得抄入答题卡)
Dear Tony,
Chongyang,the traditional Chinese festival for the elderly,is coming around.We are planning to visit the Nursing Home to celebrate the speeial day, and we would like to invite students from your school to join us.

Looking forward to your early reply.
Yours,
Li Hua
2007 年普通高等学校招生全国统一考试(湖北卷)
英语试题参考答案
第一部分:听力
1.A 2.B 3.A 4. B 5.B 6. C 7.A 8.C 9.A 10.B 11.C 12.C 13.A 14.B 15.B 16.C 17.B 18.A 19.A 20.C
第二部分:英语语言知识运用
第一节:多项选择
21.B 22.D 23.C 24.A 25.C 26.D 27.B 28.D 29.A 30.D
第二节:完成句子
31.the door shut
32.(of) what he’s talking about
33.that he has failed (for) several times
34.had arrived for him
35.should (ought to/must) be allowed to learn (study)
36.memory for names
37.Whether he has been abroad or not
38.about three times as great as that of last year
39.will help be given to
40.may (might/could) have had a hand
评分原则:
1、结构正确、用给定的英语词完整表达所给的汉语信息,给满分;
2、未用给定词不给分;
3、结构不正确不给分;
4、结构正确,信息完整,但一个或一个以上拼写错误扣0.5分;
5、结构正确,信息有遗漏扣0.5分;
6、结构正确,添加无关信息扣0.5分.
第三节:完形填空
41.D 42.C 43.D 44.B 45.D 46.B 47.D 48.A 49.C 50.A 51.D 52.A 53.C 54.D 55.B 56.A 57.B 58.D 59.C 60.D
第三部分:阅读理解
61.D 62.A 63.D 64.B 65.C 66.A 67.B 68.B 69.C 70.D 71.B 72.A 73.C 74.C 75.A 76.D 77.A 78.D 79.C 80.B
第四部分:书面表达
One Possible Version:
We have planned several activities. When we get there,we will visit the elderly in their rooms in groups,presenitng them with flowers and self-made cards to show our respect and love. Then we will do some cleaning and washing for them with the help of the nurses.
As some old people feel lonely. We may chat with them about their old days, changes of our city, or anything they are interested in. We may also give them some performanees: singing, daneing, and so on.
I am sure we will both gain a better understanding of the elderly in China. If you have any suggestions, please let us know.
2007年全国高等学校统考 (湖南卷)
英语试题
第一部分 听力(共三节,满分30分)
第一节(共5小题,满分7.5分)
听下面5段对话,每段对话后有一个小题,从题中所给的A、B、C三个选项中选出最佳选项。听完每段对话后,你都有10秒钟的时间来回答有关小题和阅读下一小题。每段对话仅读一遍。
1. When will the two speakers leave if they get cheaper tickets?’
A. On Tuesday. B. On Thursday C. On Friday
2. What is the probably relationship between the two speakers?
A. Husband and wife B. Passenger and driver C. Salesgirl and customer
3. What does the man imply?
A. He got help from the woman. B. He needed more time to study
C. He worked hard for his composition
4. What will the woman do in the morning?
A. See Lisa off B. Go to the zoo C. Deal with an e-mail
5. Where does this conversation most probably take place?
A . In a bus B. In an office C. In a phone box
第二节 (共12小题,满分18分)
听下面4段对话。每段对话后有几个小题,从题中所给的A、B、C三个选项中,选出最佳选项,听每段对话前,你将有时间阅读各个小题,每小题5秒钟;听完后,各小题将给出5秒钟的作答时间。每段对话读两遍。
听下面一段对话,回答第6和第7两个小题。
6. Where will the man have his dinner?
A. At Golden Pond Restaurant B. At Black Forest Restaurant
C. At Moon River Restaurant
7. What do we know about the man?
A. He wants to eat beef B. He likes to have dinner late
C. prefers to wear a jacket and tie
听下面一段对话,回答第8至第10三个小题。
8. How does the man feel about the delay of his flight?
A. Calm B. Glad C. Angry
9. Which is the right boarding gate for the man?
A. Gate 5 B. Gate 20 C. Gate 38
10. When will the plane probably take off?
A. At 3:30 p.m. B. At 3:40 p.m. C. At 3:50 p.m.
听下面一段对话,回答第11至第13三个小题。
11. What are the two speakers mainly talking about?
A. Eco-tours B. Popular sports C. Famous countries
12. Which activity in the USA in introduced in the dialogue?
A. Mountain-climbing B. River-rafting C. Bird-watching
13. How many countries are mentioned in the dialogue?
A. Three B. Four C. Five
听下面一段对话,回答第14至第17四个小题。
14. Why does the man ask the woman for help?
A. He has to read a report B. He has to write a report
C. He has to wait for a call
15. What does the man ask the woman to do?
A. Buy some food B. Make tomato soup C. Bring back the café
16. Which of the following is suggested by the woman?
A. A pie B. A salad C. A sandwich
17. What does the woman think of the man?
A. Lazy B. Greedy C. Curious
第三节(共3小题,满分4.5分)
听下面一段材料,将第18至第20三个小题的信息补充完整,每个小题不超过三个单词。 听材料前,你将有时间阅读各个小题,每个小题5秒钟; 听完后,各小题将给出15秒钟的作答时间。本材料读两遍。
第二部分 英语知识运用
第一节 单项填空 (共15小题, 满分15分)
从A、B、C、D四个选项中,选出可以填入空白处的最佳选项,并在答题卡上将该项涂黑。
21. Reality is not the way you wish things to be, nor the way they appear to be, _________ the way they actually are.
A. as B. or C. but C. and
22. __________ the silence for the pauses, we could hear each other’s breathing and could almost bear our own heartbeats.
A. In B. For C. Under D. Between
23. Polar bears live mostly on ________ sea ice, which they use as _______ platform for hunting seals.
A. a; a B. a; the C. 不填; a D. the; 不填
24. The biggest problem for most plants, which ________ just get up and run away when threatened, is that animals like to eat them.
A. shan’t B. can’t C. needn’t D. mustn’t
25. As the years passed, many occasions—birthdays, awards, graduations—________ with Dad’s flowers.
A. are marked B. were marked C. have marked D. had marked
26. It’s hard for him playing against me. I’ve got nothing to play for, but for him, he needs to win so ________.
A. far B. well C. little D. badly
27. To save class time, our teacher has ________ students do half of the exercise in class and complete the other half for homework.
A. us B. we C. our D. ours
28. Having checked the doors were closed , and _________ all the lights were off, the boy opened the door to his bedroom.
A. why B. that C. when D. where
29. As the light turned green, I stood for a moment, not _________, and asked myself what I was going to do.
A. moved B moving C. to move D. being moved
30. We live day by day, but in the great things, the time of days and weeks _________ so small that a day is unimportant.
A. is B. are C. has been D. have been
31. Cathy is taking notes of the grammatical rules in class at Sunshine School, where she ________ English for a year.
A. studies B. studied C. is studying D. has been studying
32. By serving others, a person focuses on someone other than himself or herself, ___________ can be very eye-opening and rewarding.
A. who B. which C. what D. that
33. Most birds find it safe to sleep in the trees, but ________ they have eggs or young chicks, they don’t use a nest.
A. why B. how C. unless D. where
34. “Things _________ never come again!” I couldn’t help talking to myself.
A. lost B. losing C. to lose D. have lost
35. There is an old proverb, “Love me, love my dog.” But there is _________ wisdom in this: “Love me, love my book.”
A. some B. much C. more D. most
第二节 完形填空 (共20小题, 满分30分)
阅读下面短文,掌握其大意,然后从第36至第55小题所给的A、B、C、D四个选项中,选出最佳选项。
When I was young, my parents ran a snack bar in our small town.
One evening in early April, my mother told me to fill in at the snack bar 36 a worker who had the flu. I told her I would mess it up, 37 I had never worked at the bar before. I 38 that instead of making money, I would end up owing it.
“You can do it,” said my mother, “ 39 , you won’t get much business until lunch.”
“But I’ll never remember the orders, and I’m no good 40 money. Please, Mom, don’t 41 me.
“Then I’ll help you,” she said.
I shrugged my shoulders. I thought my mother’s 42 was a bad one, but I 43 .
When I got to the bar the next day, I found my mother was 44 . Because the weather that day was rainy and cold, people wanted hot snacks and drinks. 45 , I was really slow at taking the orders and making change. The line of people grew, and everybody seemed 46 , I was so nervous that my hands shook, and I 47 a cup into pieces. What a mess! Then my mother came to 48 me, and she also showed me how to make 49 . If someone gave me $ 5 for something that cost $ 3.25, I handed over
50 quarters and a dollar and said, “75 cents makes four dollars, plus one dollar makes five.” Things went more 51 after that.
By the end of the day, I could remember orders, 52 the bill, and make change quickly with a smile. I was even a little 53 when the sun came out and dried up business. My mother said she was proud of me, and when she 54 that I work at the snack bar again next year, I did not even shrug. I was too busy
55 the restaurant I would open one day.
36.
A. to
B. for
C. after
D. over
37.
A. because
B. though
C. until
D. while
38.
A. promised
B. noticed
C. worried
D. hoped
39.
A. Therefore
B. However
C. Besides
D. Yet
40.
A. of
B. on
C. about
D. with
41.
A. blame
B. fool
C. frighten
D. make
42.
A. idea
B. bar
C. day
D. answer
43.
A. guessed
B. obeyed
C. begged
D. admitted
44.
A. angry
B. sad
C. worry
D. ashamed
45.
A. At least
B. At last
C. At most
D. At first
46.
A. surprised
B. impolite
C. pleased
D. impatient
47.
A. damaged
B. destroyed
C. broke
D. ruined
48.
A. scold
B. help
C. beat
D. save
49.
A. money
B. lunch
C. coffee
D. change
50.
A. two
B. three
C. four
D. five
51.
A. smoothly
B. fairly
C. simply
D. conveniently
52.
A. turn in
B. count out
C. take over
D. add up
53.
A. discouraged
B. disturbed
C. disappointed
D. distrusted
54.
A. thought
B. stated
C. announced
D. suggested
55.
A. imagining
B. preparing
C. examining
D. describing
第三部分 阅读理解(共两节,满分40分)
第一节 选择题(共17小题, 满分34分)
A
In June, 2007, a group of students from eight high schools in Winnipeg, the capital of Canada’s Manitoba province, will begin test-launching (试发射) a satellite the size of a Rubik’s cube.
The one-kilogram Win-Cub satellite, named for its home city and its shape, will be put into low orbit. Once in space, it can perform for a few months or up to several years, communicating information that could help find the signs of earthquakes.
There are 80 similar satellite projects worldwide, but this is the first high-school based program of its kind in Canada. 30 Manitoba high school students are having a hand in designing and building the satellite, in cooperation with aerospace (航空航天的) experts and 10 students from the University of Manitoba, and with support from two other organizations.
The Win-Cube project is not something that goes on a piece of paper; it is real-world engineering, allowing high school students to have an opportunity to learn more about the exciting world of engineering through their participation in this challenging program. It is also taken as a wonderful example of the unique partnerships within Manitoba. Designing, building and launching a satellite with high-school participation will bring this world-class educational project into reality and Manitoba closer to space.
“These Manitoba high school students deserve congratulations for their enthusiasm, innovation (创新), and a strong love for discovery,” said Education, Citizenship and Youth Minister Peter Bjomson. “We want to make science more relevant (相关的), interesting and attractive to high school students by showing them how classroom studies can relate to practical experience in the workplace or, in this case, in space,” Bjomson added.
The Win-Cube program is mainly named at inspiring a strong desire for discovery on the part of the students. It also shows Manitoba’s devotion to research and innovation and the development of a skilled workforce—all important drivers of knowledge-based economic growth.
56. According to the passage, the Win-Cube satellite is _________.
A. named after Manitoba and its shape B. intended for international communication
C. designed like a Rubik’s cube both in shape and size
D. challenged by university students around the world
57. According to Mr. Bjomson, ___________.
A. those Manitoba high school students are worth praising
B. the study of space can be practically made in classrooms
C. Manitoba high schools are famous for the study of space
D. scientific research is too far away from high school students
58. The primary purpose of the project is to _________.
A. find the early signs of earthquakes B. relate studies to practical
C. help high school students study real-world engineering
D. inspire a strong desire for discovery among the students
59. The best title for this passage may be ________.
A. Manitoba School B. Win-Cube Program C. Space Co-operation
D. Satellite Launching
B

Welcome to my Message Board!
Subject: Slimming down classics?
Mr.
Handsome
2007-5-12
6: 34 AM
Orion Books, which decides there is a market in creating cut-down classics (经典著作), is slimming down some novels by such great writers as L. Tolstoy, M. Mitchell and C. Bronte. Now, each of them has been whittled down to about 400 pages by cutting 30 to 40 pages per cent of original, with words, sentences, paragraphs and, in a few cases, chapters removed. The first six shortened editions, all priced at £6.99 and advertised as great reads “in half the time”, will go on sale next month, with plans for 50 to 100 more to follow. The publishing house believes that modern readers will welcome the shorter versions.
Mr.
Edwards
2007-5-12
9: 40 AM
Well, I’m publisher of Orion Group. Thanks for your attention, Mr. Handsome.
I must say, the idea developed from a game of “shame” in my office. Each of us was required to confess (承认) to the most embarrassing blanks in his or her reading. I admitted that I had never read Anna Karenina and tried but failed to get through Gone with the Wind several times. One of my colleagues acknowledged skipping (跳读) Jane Eyre. We realized that life is too short to read all the books you want to and we never were going to read these ones.
As a leading publishing house, we are trying to make classics convenient for readers but it’s not as if we’re withdrawing the original versions. They are still there if you want to read them.
Ms. Weir
2007-5-12
11:35 AM
I’m director of the online bookclub www.lovereading.co.uk
Mr. Edwards, I think your shortened editions is a breath of fresh air. I’m guilty of never having read Anna Karenina, because it’s just so long. I’d much rather read two 300-page books than one 600-page book. I am looking forward to more shortened classics!
Mr.
Crockatt
2007-5-12
4:38 PM
I’m from the London independent bookshop Corckatt & Powell.
In my opinion, the practice is completely ridiculous. How can you edit the classics? I’m afraid reading some of these book is hard work, and that is why you have to develop as a reader. If people don’t have time to read Anna Karenina, then fine. But don’t read a shortened version and kid yourself it’s the real thing.
60. According to the message board, Orion Books ___________.
A. opposes the reading of original classics B. is embarrassed for cutting down classics
C. thinks cut-down classics have a bright future D. is cautions in its decision to cut down classics
61. In Mr. Edwards’ opinion, Orion Group is shortening classics to _________.
A. make them easier to read B. meet a large demand in the market
C. increase the sales of literary books D. compete with their original versions
62. By describing the shortened classics as “a breath of fresh air”, Ms. Weir ________.
A. speaks highly of the cut-down classics B. shows her love for original classics
C. feels guilty of not reading the classics D. disapprove of shortening the classics
63. Mr. Crockatt seems to imply that _________.
A. reading the classic works is a confusing attempt
B. shortening the classics does harm to the original
C. publishing the cut-down classics is a difficult job
D. editing the classic works satisfies children’s needs
C
Photos that you might have found down the back of your sofa are now big business!
In 2005, the American artist Richard Prince’s photograph of a photograph, Untitled (Cowboy), was sold for $ 1, 248, 000.
Prince is certainly not the only contemporary artist to have worked with so-called “found photographs”—a loose term given to everything from discarded(丢弃的) prints discovered in a junk shop to old advertisem*nts or amateur photographs from a stranger’s family album. The German artist Joachim Schmid, who believes “basically everything is worth looking at”, has gathered discarded photographs, postcards and newspaper images since 1982. In his on-going project, Archiv, he groups photographs of family life according to themes: people with dogs; teams; new cars; dinner with the family; and so on.
Like Schmid, the editors of several self-published art magazines also champion (捍卫) found photographs. One of them, called simply Found, was born one snowy night in Chicago, when Davy Rothbard returned to his car to find under his wiper(雨刷) an angry note intended for some else: “Why’s your car HERE at HER place?” The note became the starting point for Rothbard’s addictive publication, which features found photographs sent in by readers, such a poster discovered in our drawer.
The whole found-photograph phenomenon has raised some questions. Perhaps one of the most difficult is: can these images really be considered as art? And if so, whose art? Yet found photographs produced by artists, such Richard Prince, may riding his horse hurriedly to meet someone? Or how did Prince create this photograph? It’s anyone’s guess. In addition, as we imagine the back-story to the people in the found photographs artists, like Schmid, have collated (整理), we also turn toward our own photographic albums. Why is memory so important to us? Why do we all seek to freeze in time the faces of our children, our parents, our lovers, and ourselves? Will they mean anything to anyone after we’ve gone?
64. The first paragraph of the passage is used to _________.
A. remind readers of found photographs B. advise reader to start a new kind of business
C. ask readers to find photographs behind sofa D. show readers the value of found photographs
65. According to the passage, Joachim Schmid _________.
A. is fond of collecting family life photographs B. found a complaining not under his car wiper
C. is working for several self-published magazines
D. wondered at the artistic nature of found photographs
66. The underlined word “them” in Para 4 refers to __________.
A. the readers B. the editors C. the found photographs
D. the self-published magazines
67. By asking a series of questions in Para 5, the author mainly intends to indicate that ________.
A. memory of the past is very important to people
B. found photographs allow people to think freely
C. the back-story of found photographs is puzzling
D. the real value of found photographs is questionable
68. The author’s attitude towards found photographs can be described as _________.
A. critical B. doubtful C. optimistic D. satisfied
D
One of our biggest fears nowadays is that our kids might some day get lost in a “sea of technology” rather than experiencing the natural world. Fear-producing TV and computer games are leading to a serious disconnect between kids and the great outdoors, which will changes the wild places of the world, its creatures and human health for the worse, unless adults get working on child’s play.
Each of us has a place in nature we go sometimes, even if it was torn down. We cannot be the last generation to have that place. At this rate, kids who miss the sense of wonder outdoors will not grow up to be protectors of natural landscapes. “If the decline in parks use continues across North America, who will defend parks against encroachment (蚕食)?” asks Richard Louv, author of Last Child in the Woods.
Without having a nature experience, kids, can turn out just fine, but they are missing out a huge enrichment of their lives. That applies to everything from their physical health and mental health, to stress levels, creativity and cognitive (认知的) skills. Experts predict modern kids will have poorer health than their parents—and they say a lack of outside play is surely part of it; research suggests that kids do better academically in schools with a nature component and that play in nature fosters (培养) leadership by the smartest, not by the toughest. Even a tiny outdoor experience can create wonder in a child. The three-year-old turning over his first rock realizes he is not alone in the world. A clump of trees on the roadside can be the whole universe in his eyes. We really need to value that more.
Kids are not to blame. They are over-protected and frightened. It is dangerous out there from time to time, but repetitive stress from computers is replacing breaking an arm as a childhood rite(仪式)of passage.
Everyone, from developers, to schools and outdoorsy citizens, should help regain for our kids some of the freedom and joy of exploring, taking friendship in fields and woods that cement (增强) love, respect and need for landscape. As parents, we should devote some of our energies to taking our kids into nature. This could yet be our greatest cause.
69. The main idea of Paragraph 2 is that __________.
A. kids missing the sense of wonder outdoors
B. parks are in danger of being gradually encroached
C. Richard Louv is the author of Last Child in the Woods
D. children are expected to develop into protectors of nature
70. According to the passage, children without experiencing nature will _________.
A. keep a high sense of wonder B. be over-protected by their parents
C. be less healthy both physically and mentally
D. change wild places and creatures for the better
71. According to the author, children’s breaking an arm is ___________.
A. the fault on the part of their parents B. the natural experience in their growing up
C. the result of their own carelessness in play
D. the effect of their repetitive stress from computers
72. In writing this passage, the author mainly intends to ________.
A. blame children for getting lost in computer games
B. encourage children to protect parks from encroachment
C. show his concern about children’s lack of experience in nature
D. inspire children to keep the sense of wonder about things around
第二节 简答题(共3小题, 满分6分)
阅读下面短文,根据第73至第75小题的具体要求,简要回答问题。
The north magnetic pole (磁极) is no longer a resident of Canada. It has drifted across the Canadian Arctic and is now angling toward Siberia.
Not to be confused with the geographic North Pole—the northern extremity of the Earth’s axis(地轴),the north magnetic pole (where the Earth’s magnetic field is directed downward) is moving due to natural changes in the Earth’s magnetic field, a process that originates about 3, 000 kilometers below our feet, in the outer core(核心)of the planet. But scientists are now noticing that the magnetic pole has picked up its pace.
Over the past century, the pole has been migrating at an average speed of about 10 kilometers per year, says Larry Newitt, head of Geomagnetic Laboratory at Natural Resources Canada in Ottawa. Since the 1970s, this speed has increased to about 50 kilometers per year.
Scientists are wondering why the speed is increasing, says Newitt. One Hypothesis (假说) relates it to “magnetic jerks”, sudden shifts (变化) in the rate of change of the magnetic field. There have been three jerks in the past four decades, each one correlating to an increase in speed.
If the north magnetic pole continues at its current rate, it could reach Siberia by 2056.
“For most of recorded history, the pole has been a part of Canada, and now it’s not,” says Newitt. “It may be a blow to our collective psyche, but other than that, it doesn’t have a major effect on most people’s existence.
73. Find in the passage a word closest in meaning to the underlined word “angling”.
74. What could happen to the north magnetic pole by the middle of the 21 century? (回答词数不超过8个)

75. What is the main idea of the passage? (回答词数不超过8个)

第四部分 书面表达(共两节,满分35分)
第一节 填空(共10小题,满分10分)
阅读下面短文,根据所读内容在文后第76至第85小题的空格里填上适当的单词或短语。注意:每空不超过3 个单词。
Host family accommodation, or living with host families, remains popular among language travel students for its advantages. These days, host families are trying to offer more in terms of quality, for they still have something to worry about.
Host family accommodation is often seen as the number-one choice for its advantages in language study, cultural communication and cost of living. Staying alongside host families enables students to get enough practice for the short time of their studying so that their language acquisition is likely to become faster. Living in host families also has the advantage for students of being able to spend a lot of time communicating with their “host parents”, and get to know the local way of life, people and culture. Another advantage is that host family accommodation can sometimes be the least expensive. It attracts students for it ensures them a family type of living at a low cost.
The advantages, however, have not prevented host families form worrying. On one hand, some host families are losing their unique selling point. The problem is that the majority of hosts in big cites, now generally single and young, have less times available for students but the selling point for host family accommodation is communication practice. One the other hand, students’ expectations have risen. They are becoming more demanding and asking for more than ever from their accommodation, as they come mainly from high socio-economic groups in their own countries.
To get out of the difficult situation, host families are now making efforts to improve the quality of service. They are trying to making living conditions better, including broadband Internet service, private bathroom, and access to plenty of hot water for long showers. They are also providing students with structured family activities.
It is believed that host family accommodation will keep the popularity vote with language travel students.
Title: 76. ______________________
Theme
Host families are improving the accommodation quality
77. _____________
1)Language study
78. ________________
2)Cultural communication
(1) 79. _________________
(2) Knowing the local way of life, people and culture
3)80. _________________
Being least expensive sometimes
81. _____________
1)Loss of selling point
Hosts’ having less time for students
2)82. _________________
Students’ becoming more demanding
83. _____________
Service quality improvement
(1) 84. _________________
(2) Organizing structured family activities
85. _____________
Host family accommodation will keep the popularity vote with students.
第二节 写作(满分25)
假设你是王平,李华是你的同窗好友,请根据下列要点,用英文给李华写一篇毕业留言。
要点:1. 简要表述李华在你心目中的印象;
     2. 用一到两个相关的事例进行具体描述;
     3. 对他李华表示美好的的祝愿。
  注意:1. 内容积极向上,语意连贯,结构完整。词数120左右。
2. 不能写成诗歌形式;不能使用真实姓名和学校名称。
参考答案
第一部分 听力
1—17: BACCB, CABAB, CABCA, AB;
18. Music Camp
19. 3 to 7
20. Have wonderful concerts
第二部分 英语知识运用
21—35: CACBB,DABBA,DBCAC
36—55: ABCCD, DABCD, DCBDB, ADCDA
第三部分 阅读理解:
56—59 CADB,60—63 CAAB,64—68 DADBC,69—72 DCBC
73. Moving/ Migrating
74. It could reach Siberia
75. The north magnetic pole is moving faster.
第四部分 写作
填空:
76. Host Family Accommodation
77. Advantages
78. Possibly becoming faster
79. Communicating with hosts
80. Cost of living
81. Worries/ Problems
82. Rise in expectation
83. Efforts/ Measures
84. Bettering living conditions
85. Conclusion
Dear
书面表达:
Li Hua,
It’s time to say goodbye. It’s time to recall those beautiful days we spend together. You are the very person who gives me a hand when necessary. You are the most considerate person who knows exactly what I need.
Still remember that night two years ago? When I suddenly fell ill, it was you who carried me on your back to a nearby hospital. You took good care of me while I was having an injection. Later, you managed to help me with my lessons. Without your help, I would have failed in the examination.
I still have hundreds of stories to share with you. I still have thousands of blessings for you. May all your dreams come true! May our friendship last till the end of the universe.!
Yours,
Wang Ping
英语
第Ⅰ卷(选择题共115分)
第一部分听力(共两节,满分30分)
做题时,先将答案标在试卷上。录音内容结束后,你将有两分钟的时间将试卷上的答案转涂到答题卡上。
第一节(共5小题;每小题1.5分,满分7.5分)
听下面5段对话。每段对话后有一个小题,从题中所给的A、B、C三个选项中选出最人佳选项,并标在试卷的相应位置。听完每段对话后,你都有10秒钟的时间来回答有关小题和阅读下一小题。每段对话仅读一遍。
例:How much is the shirt?
A.£19.15.B.£9.15C.£9.18
答案是B。
1.Who is coming for tea?
A.John.B.Mark.C.Tracy.
2.What will the man do next?
A.Leave right away.B.Stay for dinner.C.Catch a train
3.What does the man come for?
A.A lecture.B.A meeting.C.A party.
4.What size does the man want?
A.9.B.35.C.39.
5.What are the speakers talking about?
A.Life in Southeast AsiaB.Weather conditions.C.A holiday tour.
第二节(共15小题;每小题1.5分,满分22.5分)
听下面5段对话或独白。每段对话或独白后有几个小题,从题中所给的A、B、C三个选项中选出最佳选项,并标在试卷的相应位置。听每段对话或独白前,你将有时间阅读各个小题,每小题5秒钟;听完后,各小题将给出5秒钟的作答时间。每段对话或独白读两遍。
听第6段材料,回答第6、7题。
6.What is the man doing?
A.Giving a speech.B.Chairing a meeting. C.Introducing a person.
7.Why soes the woman sing so well?
A.She has a great teacher.B.She teaches singing.C.She is young.
听第7段材料,回答第8、9题。
8.What is the second gift for Jimmy?
A.A car.B.A watch.C.A computer.
9.Why does Jimmy feel happy?
A.He lives with his parents.
B.He’s got what he dreamt of.
C.He’s received lots of presents.
听第8段材料,回答第10至12题。
10.What is the relationship between the speakers?
A.They are friends.
B.They are strangers to each other.
C.They are husband and wife
11.Why does the woman come to talk with the man?
A.To get a job.B.To take a test.C.To see the secretary.
12.What does the man mean by saying sorry?
A.He can’t hear the woman clearly.
B.He doesn’t need a designer.
C.He can’t help the woman.
听第9段材料,回答第13至16题。
13.What do we know about the woman?
A.She lives close to the office.
B.She is new to the company.
C.She likes the big kitchen.
14.How does the man go to work?
A.On foot.B.By bus.C.By car
15.Why was Susan late for work?
A.She missed the bus.
B.Her train was late.
C.Her car broke down.
16.What will the man do the next day?
A.Go to work by train.B.Visit Lily in her flat.C.Leave home earier.
听第10段材料,回答第17至20题。
17.Where can you most probably hear this talk?
A.In a class of the English language.
B.In a class of the Greek language.
C.In a class of the French language.
18.How long does the class last?
A.11 weeks.B.13 weeks.C.15 weeks.
19.What is “the short-cut” to learning words according to the speaker?
A.Taking more courses.
B.Reading basic words aloud.
C.Learning how words are formed.
20.Why is the class popular?
A.It is not offered each term.
B.It’s taught by Professor Morris.
C.It helps to master some useful rules.
第二部分英语知识运用(共两节,满分45分)
第一节单项填空(共15小题;每小题1分,满分15分)
从A、B、C、D四个选项中,选出可以填入空白处的最佳选项,并在答题卡上将该项涂黑。
例:It is generally considered unwise to give a child he or she wants.
A.howeverB.whateverC.whicheverD.whenever
答案是B。
21.—I saw Jane and her boyfriend in the park at eight yesterday evening.
—Impossible. She TV with me in my home then.
A.watchedB.had watchedC.would watchD.was watching
22.—How about Christmas evening party?
—I should say it was success.
A.a; aB.The; aC.a;不填D.the; 不填
23.The book is of great value. can be enjoyed unless you digest it.
A.NothingB.SomethinC.EverythingD.Anything
24.Jenny hopes that Mr. Smith will suggest a good way to have her written English in a short period.
A.improvedB.improvingC.to improveD.improve
25.My MP4 player isn’t in my bag. Where I have put it?
A.canB.mustC.shouldD.would
26.—Come on, please give me some ideas about the project.
—Sorry. With so much work my mind, I almost break down.
A.filledB.fillingC.to fillD.being filled
27.The village has developed a lot we learned farming two years ago.
A.whenB.whichC.thatD.where
28.—Excuse me, do you have the time?

A.Yes, I doB.Of course, I have
C.Aquarter to tenD.No problem
29.You will be successful in the interview you have confidence.
A.beforeB.onceC.untilD.though
30.Danny hard for long to realize his dream and now he is popular.
A.worksB.is working
C.has workedD.worked
31.—You are always full of . Can you tell me the secret?
—Taking plenty of exercise every day.
A.powerB.strengthC.forceD.energy
32.Although my opinion, the old professor didn’t come up with his own.
A.againstB.onC.forD.in
33.The news of the mayor’s coming to our school for a visit was on the radio yesterday.
A.turned outB.found outC.given outD.carried out
34.—Do you need any help, Lucy?
—Yes, The job is I could do myself.
A.less thanB.more than
C.no more thanD.not more than
35.It is none of your business other people think about you. Believe yourself.
A.howB.whatC.whichD.when
第二节完形填空(共20小题;每小题1.5分,满分30分)
阅读下面短文,从短文后所给各题的四个选项(A、B、C和D)中,选出可以填入空白处的最佳选项,并在答题卡上将该项涂黑。
I grew up in a tiny Baltimore row house in a faraway mountain area. My parents 36 the necessities of life 37 they couldn’t give much more. If I asked my father 38 a pair of jeans, he would say, “If you want them, make the money and buy them 39 .” He wasn’t being mean; he just couldn’t 40 them. From age 12 on, I did part-time jobs after school.
When I 41 from high school, I joined the navy. Soon I was in a boot camp(新兵训练营) at Parris Island, S. C., where I learned that life in the navy centered around completing daily
42 . These could be anything from cleaning the camp to conducting mock(模拟的) battles. Completing these tasks successfully 43 discipline, team-work and responsibility. It didn’t
44 whether you were black, white or Asian; everyone worked together for the 45 of the company.
I went 46 to graduate from the U.S. Naval Academy and later became an officer in the navy. The part of my job I 47 most was the comseling(咨询) meeting I 48 with the family members of the men and women in my 49 , trying to help them deal with the long periods of 50 . These proved popular and word of them spread. 51 I was being asked to give encouraging 52 to business groups, educators and keds across the country.
But I consider the boot camp my first real 53 , and my life is still guided by the
54 lessons I hearned there. It taught me discipline, friendship and the pride related to setting a task every day and working hard to 55 it.
36.A.providedB.gotC.madeD.bought
37.A.whileB.butC.soD.or
38.A.aboutB.withC.forD.of
39.A.themselvesB.ourselvesC.yourselfD.myself
40.A.payB.findC.produceD.afford
41.A.cameB.returnedC.escapedD.graduated
42.A.drillsB.tasksC.exercisesD.reports
43.A.includedB.askedC.requiredD.met
44.A.matterB.meanC.existD.work
45.A.goodB.bossC.restD.right
46.A.outB.onC.awayD.off
47.A.tookB.hatedC.enjoyedD.did
48.A.endedB.beganC.continuedD.held
49.A.chargeB.situationC.positionD.choice
50.A.lessonsB.meetingC.trainingD.separation
51.A.Long beforeB.Before longC.As usualD.Once again
52.A.performancesB.descriptionsC.speechesD.gifts
53.A.vacationB.placeC.jobD.travel
54.A.importantB.bitterC.normalD.difficult
55.A.gainB.achieveC.showD.match
第三部分阅读理解(共20小题;每小题2分,满分40分)
阅读下列短文,从每题所给的四个选项(A、B、C和D)中,选出最佳选项,并在答题卡上将该项涂黑。
A
When I met him, I had a lot of anger inside of me. I’ve lived my whole life in Spanish Harlem, but in my neighborhood, there are shoot-ups all the time. I know kids who have been shot or beaten up. I have friends who ended up in prison. I could have ended up that way, too, but
Mr. Clark wouldn’t let that happen.
Mr. Clark worked long hours, making sure I did my work. My grades rose. In fact, the scoresof our whole class rose. One day, he took our class to see The Phantom of the Opera, and it was the first time some kids had ever been out of Harlem. Before the show, he treated us to dinner at a restaurant and taught us not to talk with our mouths full. We did not want to let him down.
Mr. Clark was selected as Disney’s 2000 Teacher of the Year. He said he would draw three names out of a hat; those students would go with him to Los Angeles to get the award. But when the time came to draw names, Mr. Clark said, “You’re all going.”
On graduation day, there were a lot of tears. We didn’t want his class to end. In 2001, hemoved to Atlanta, but he always kept in touch. He started giving lectures about education, and wrote a bestselling book based on his classroom rules, The Essential 55. In 2003,
Mr. Clark took some of us on a trip to South Africa to deliver school supplies and visit orphanages(孤儿院). It was the most amazing experience of my life. It’s now my dream to one day start a group of women’s clubs, helping people from all backgrounds.
56.Without Mr. Clark, the writer .
A.might have been put into prison
B.might not have won the prize
C.might have joined a women’s club
D.might not have moved to Atlanta
57.The Essential 55 is .
A.a showB.a speech
C.a classroom ruleD.a book
58.How many students’names were finally drawn out of a hat by Mr. Clark?
A.NoneB.ThreeC.Fifty-five.D.All.
59.In the passage, the writer intends to tell us that .
A.Mr. Clark went to South Africa because he liked travelling
B.Mr. Clark helped to set up a group of women’s clubs
C.a good teacher can help raise his or her students’ scores
D.a good teacher has a good influence on his or her students
B
A small town in southwest Britain is banning(禁止) plastic bags in an attempt to help the environment and cut waste-a step that environmentalists believe is a first for Europe.
Shopkeepers in Modbury population 1,500, agreed to stop handing out disposable plastic bags to customers on Saturday. They said paper sacks and cloth carrier bags would be offered instead.
Last month, San Franciso became the first U.S. city to ban plastic grocery bags. Internationally, laws to discourage the use of plastic bags have been passed in parts of South Africa and Ireland, where govermments either tax shoppers who use them or fine companies that hand them out. Bangladesh already bans them, and so do at least 30 remote Alaskan villages.
Modbury, about 225 miles southwest of London, has also declared a bag ammesty(宽限期), allowing local people to hand in plastic bags that have piled up at home. They will be sent for a recveling.
The Modbury ban was the idea of Rebecca Hosking, who saw the effect of bags on manne life while working in the Pacific as a wildlife camerawoman. She said response(反应) in the town so far had been “really positive”.
“Modbury is quite an old-fashioned town and a lot of people have wicker baskets to go out shopping anyway,” Hosking told Sky News Television.
The Worldwatch Institute, an environmental research agency, states that 100 billion plastic bags are thrown away each year in the United States alone. More than 500 billion are used yearly around the world.
60.What was Rebecca Hosking?
A.A lawyer.B.An environmentalist.
C.A sailor.D.A photographer.
61.The underlined word “disposable” in the passage probably means .
A.acceptableB.valuableC.throw-awayD.long-lasting
62.It can be inferred from the passage that .
A.most of the people in Modbury cntinue to use plastic bags
B.fewer and fewer plastic bags will be used in the world.
C.San Francisco is the first city to ban plastic bags in the world
D.most countries in the world have passed laws to ban plastic bags
63.Which of the following would be the best title of the passage?
A.Environmental Protection
B.Big Cities Banning Plastic Bags
C.Effect of Plastic fa*gs on Sea Animals
D.British Town Banning Plastic Bags
C
Scientists have found what look like caves on Mars(火星), and say they could be protecting life from the planet’s terrible environment.
The first caves discovered beyond the Earth appear as seven anysterious black dots on the pictures sent back by NASA’s Mars Odyssey orbiter. Each as large as a football field, they may be openings into natural caves below the Martian suface.
“If there is life on Mars, there is a good chance you’d find it in caves,” said Jut Wynne,one of the reseachers who noticed the features while working on a US Geological Survey Mars Cave Detection Program.
Jonathan Clarke, a geologist with the Mars Society of Australia, yesterday described the discovery as exciting.
One photo taken at night by an infrared imager(红外线成像器) showed one hole to be unusualy warm, suggesting hot air trapped during the day is flowing out.
“I said:‘Wow, that’s a cave’” Dr. Clarke said excitedly. “People have been looking for these for a long time; now we have found them.”
He agreed such caves would be perfect places to hunt for life escaping from the bitterly cold,radiatiion-soaked(充满辐射的), dry surface.
“Tiny drops of water could collect inside,” he said. “If there are gases coming out, they could provide energy for a whole range of bacteria. A cave is also a protection from radiation; the surface of Mars is exposed to high levels of space radiation.”
The caves probably formed when tube-shaped lave flows(管状岩浆流) spread across the planet long ago. The outside of the tubes cooled, forming solid walls, while something hotter inside allowed the remaining have to flow out, forming caves.
64.What does the passage mainly talk about?
A.How the caves were formed on Mars.
B.How scientists found these caves on Mars.
C.Caves on Mars may be full of hot air or a sign of life.
D.Scientiste have completely recognized the surface of Mars.
65.We can learn from the passage that .
A.water has already been found on Mars
B.the scientists found all the caves at night
C.it is certain that there is life in these caves
D.the surface of Mars is bitterly cold, radiation-soaked and dry
66.According to the passage, Dr. Clarke was so excited because .
A.such caves could provide energy for life
B.they had finally found the caves on Mars
C.such caves would be perfect places to hunt for life
D.scientists had long been looking for these caves
67.Necessary conditions for life on Mars mentioned in the passage may include .
A.lava and energy
B.water and radiation from space
C.gases and lava
D.water and protection from radiation
D
68.The best title of the 3nl advertisem*nt would be
A.Training at HomeB.Learning at Home
C.Recording WorkD.Working at Home
69.According to the information above, if your child has hearing problems, you can turn to .
A.At-Home Profession Corp.B.Globe Insurance Company
C.Bradford PubbcationsD.American Inventors Corp.
70.From the advertisem*nts above, we can learn that .
A.Globe Insurance Company has a history of 51 years
B.as a medical transcriptionist, you may earn $ 25,000 a month
C.Bradford Pulications offers a treatment to hearing impaired parents
D.you can visit www. aic. com when you invent a new model of hearing aid
E
Dogs wag(摇摆) their tails in different directions depending on whether they are excited and wanting to move forward or threatened and thinking of moving back, a study has found.
Researchers in Italy examined the tail wagging behaviour of 30 dogs, catching their responses to a range of stimuli(刺激物) with video camems. To conduct the study they chose 15 male dogs and 15 female ones aged between one and six years. The dogs were all family pets whose owners had allowed them to take part in the experiment at Bari Umversity. The dogs were placed in a large wooden box with an opening at the front to allow for them to view various stimuli. They were tested one at a time.
The researchers led by Professor Giorgio Vallortigara of the University of Trieste found that when the dogs were shown their owners—a positive expenence—their tails wagged energetically to the right side. When they were shown an unfamiliar human they wagged to the right, but with somewhat less enthusiasm. The appears of a cat again caused a right-hand side wag, although with less intensity again. The appearance of a large unfamiliar dog, similar to a German shepherd, changed the direction of tail wagging to the left. Research ers supposed the dog was thinking of moving back. When the dogs were not shown any stimuli they tended to wag their tails to the left, suggesting they preferred company. While the changes in the tail wagging were not easily noticed without the aid of video, it was thought that the findings could help people judge the mood (心情) of dogs. Computer and video systems, for example, could be used by professional dog trainers to determine the mood of dogs that they were required to approach.
71.The video cameras were used to catch the dogs’ responses because .
A.it was easier to catch the dogs’response changes in the tail wagging
B.the dogs were put in the wooden boxes and tested one at a time.
C.they enabled the dogs’owners to know about their dogs’habit
D.the dogs wagged their tails in different directions when they were in different moods
72.The underlined word“intensity” in the passage means .
A.surpriseB.worryC.excitementD.interest
73.When there are no stimuli, a dog will .
A.wag to the leftB.wag to the right
C.not wag at allD.wag to the left and then to the right
74.The underlined word “they” refers to .
A.the dogsB.the trainersC.the systemsD.the rescarchers
75.The purpose of doing the experiment is .
A.to train dogs for their ownersB.to help people judge the mood of dogs
C.to help dogs find companyD.to help people choose their pet dogs
第Ⅱ卷(非选择题共35分)
第四部分写作(共两节,满分35分)
第一节短文改错(共10小题;每小题1分,满分10分)
此题要求改正所给短文中的错误。对标有题号的每一行作出判断:如无错误,在该行右边横线上画一个勾(√);如有错误(每行只有一个错误),则按下列情况改正:
此行多一词:把多余的词用斜线()划掉,在该行右边横线上写出该词,并也用斜线()划掉。
此行缺一个词:在缺词处加一个漏字符号(∧),在该行右边横线上写出该加的词。
此行错一个词:在错的词下划一横线,在该行右边横线上写出改正后的词。
注意:原行没有错的不要改。
短文改错
May 10,2007 Fine
Tady I was having a PE lesson while I fell down76.
and hurt my foot. I was in greatly pain at that moment,77.
but I tried to act as if nothing has happened until the78.
class was over. Though I had difficulty walk back to79.
my classroom, I still didn’t tell anyone but even refused80.
the offer of help of my classmates. As result, the hurt81.
in my foot became worse. Now I know I’m wrong. We82.
can tell others our need for help and accept his help.83.
Some day we can not help others in return. In this way,84.
we can get along to each other happily and peacefully.85.
第二节书面表达(满分25分)
假设你是林华,福建省某中学高三学生,请你根据以下学生会倡议书的内容给某英文报社写一封信,叙述拟开展活动的内容并发表自己的看法。
注意:1.信的开头语已经给出,不计入总词数;
2.可根据内容要点适当发挥,但不要逐条翻译;
3.词数100左右:
4.参考词汇:增进promote;鼓励节约encourage econonmy
Dear editor,
I am a student of Senior Tree in a middle school in Fujian Province.
Yours sincerely,
Lin Hua
英语试题参考答案
第一部分
1.B2.A3.B4.A5.C6.C7.A8.C9.B10.B
11.A12.C13.A14.B15.B16.C17.A18.B19.C20.C
第二部分
21.D22.B23.A24.A25.A26.B27.D28.C29.B30.C
31.D32.A33.C34.B35.B36.A37.B38.C39.C40.D
41.D42.B43.C44.A45.A46.B47.C48.D49.A50.D
51.B52.C53.C54.A55.B
第三部分
56.A57.D58.A59.D60.D61.C62.B63.D64.C65.D
66.B67.D68.D69.C70.D71.A72.C73.A74.B75.D
第四部分
第一节:
May 10,2007 Fine
Tady I was having a PE lesson while I fell down76. when
and hurt my foot. I was in greatly pain at that moment,77. great
but I tried to act as if nothing has happened until the78. had
class was over. Though I had difficulty walk back to79. walking
my classroom, I still didn’t tell anyone but even refused80. and
the offer of help of my classmates. As result, the hurt81. a
in my foot became worse. Now I know I’m wrong. We82. √
can tell others our need for help and accept his help.83. their
Some day we can not help others in return. In this way,84. not
we can get along to each other happily and peacefully.85. with
第二节:
One possible version
Dear editor,
I am a student of Senior Tree in a middle school in Fujian Province.
In order that we can make the best of learning materials, the Students’Union of our school is arranging an activity. We students of Senfor Three are called on to give away our used books, newspapers or magazines to the students in the lower grades. The idea, which is intendent to promote friendsship and encourage economy, is highly praised and supported by the teachers and students alike. The activity will last about ten days, from June 10 to 20.
As a student, I am strongly for the activity because it is very meaningful and helpful. I hope that this kind of activity will continue in the future.
Yours sincerely,
Lin Hua
2007年普通高等学校招生全国统一考试(辽宁卷)
英 语
本试卷分第I卷(选择题)和第II卷(非选择题)两部分。第I卷1至12页。第II卷13至14页。考试结束,将本试卷和答题卡一并交回。
第I卷
注意事项:
1.答题前,考生在答题卡上务必用直经0.5毫米黑色墨水签字笔将自己的姓名,准考证号填写清楚,并贴好条形码。请认真核准条形码上的准考证号、姓名和科目。
2.每小题选出答案后,用2B铅笔把答题卡上对应题目的答案标号涂黑。如需改动,用橡皮擦干净后,再选涂其他答案标号,在试题卷上作答无效。
第一部分 听力(共两节,满分30分)
做题时,先将答案标在试卷上。录音内容结束后,你将有两分钟的时间将试卷上答案转涂到答题卡上。
第一节 (共5 小题;每小题 1.5 分,满分 7.5 分)听下面5段对话。每段对话后有一个小题,从题中所给的A、B、C三个选项中选出最佳选项,并标在试卷的相应位置。听完每段对话后,你都有10秒钟的时间来回答有关小题和阅读下一小题。每段对话仅读一遍。
例:How much is the shirt?
A.£19.15.
B.£9.15.
C.£9.18.
答案是B。
1.Who is coming for tea?
A.John.
B.Mark.
C.Tracy.
2.What will the man do next?
A.Leave right away.
B.Stay for dinner.
C.Catch a train.
3.What does the man come for?
A.A lecture.
B.A meeting.
C.A party.
4.What size does the man want?
A.9.
B.35.
C.39.
5.What are the speakers talking about?
A.Life in Southeast Asia.
B.Weather conditions.
C.A holiday tour.
第二节 (共15小题;每小题 1.5 分,满分 22.5 分)
听下面5 段对话或独白。每段对话或独白后有几个小题,从题中所给的A、B、C三个选项中选出最佳选项,并标在试卷的相应位置。听每段对话或独白前,你将有时间阅览室读各个小题,每小题5秒钟;听完后,各小题将给出5秒钟的作答时间。每段对话或独白读两遍。
听第6段材料,回答第6、7题。
6.What is the man doing?
A.Giving a speech.
B.Chairing a meeting.
C.Introducing a person.
7.Why does the woman sing so well?
A.She has a great teacher.
B.She teaches singing.
C.She is young.
听第7段材料,回答第8、9题。
8.What is the second gift for Jimmy?
A.A car.
B.A watch.
C.A computer.
9.Why does Jimmy feel happy?
A.He lives with his parents.
B.He’s got what he dreamt of.
C.He’s received lots of presents.
听第8段材料,回答第10至12题。
10.What is the relationship between the speakers?
A.They are friends.
B.They are strangers to each other.
C.They are husband and wife.
11.Why does the woman come to talk with the man?
A.To get a job.
B.To take a test.
C.To see the secretary.
12.What does the man mean by saying sorry?
A.He can’t hear the woman clearly.
B.He doesn’t need a designer.
C.He can’t help the woman.
听第9段材料,回答第13至16题。
13.What do we know about the woman?
A.She lives close to the office.
B.She is new to the company.
C.She likes the big kitchen.
14.How does the man go to work?
A.On foot.
B.By bus.
C.By car.
15.Why was Susan late for work?
A.She missed the bus.
B.Her train was late.
C.Her car broke down.
16.What will the man do the next day?
A.Go to work by train.
B.Visit Lily in her flat.
C.Leave home earlier.
听第10段材料,回答第17至 20 题。
17.Where can you most probably hear this talk?
A.In a class of the English language.
B.In a class of the Greek language.
C.In a class of the French language.
18.How long does the class last?
A.11 weeks.
B.13 weeks.
C.15 weeks.
19.What is“the short-cut” to learning words according to the speaker?
A.Taking more courses.
B.Reading basic words aloud.
C.Learning how words are formed.
20.Why is the class popular?
A.It is not offered each term.
B.It’s taught by Professor Morris.
C.It helps to master some useful rules.
第二部分 英语知识运用(共两节,满分45分)
第一节 单项填空(共15小题;每小题1分,满分15分)
从A、B、C、D四个选项中,选出可以填入空白处的最佳选项,并在答题卡上将该项涂黑。
例:It is generally considered unwise to give children ________ they want.
A.however
B.whatever
C.whichever
D.whenever
答案是B。
21.Christmas is ____________special holiday when________whole family are supposed to get together.
A.the;the
B.a;a
C.the;a
D.a;the
22.—Have you handed in your schoolwork yet?
—Yes, I have, I guess it_________ now.
A.has graded
B.is graded
C.is being graded
D.is grading
23.I have been living in the United States for twenty years, but seldom _______ so lonely as now.
A.have I felt
B.I had felt
C.after it
D.after this
24.Eric received training in computer for one year, ________ he found a job in a big company.
A.after that
B.after which
C.after it
D.after this
25.Health problems are closely connected with bad eating habits and a_______ of exercise.
A.limit
B.lack
C.need
D.demand
26.—Excuse me, could you tell me the way to the British Museum?
—Sorry, I ‘m a stranger here.
— __________
A.Thanks, anyway
B.It doesn’t matter
C.Never mind
D.No problem
27.The information on the Internet gets around much more rapidly than ______ in the newspaper.
A.it
B.those
C.one
D.that
28.Help others whenever you can______ you will make the world a nicer place to live in.
A.and
B.or
C.unless
D.but
29.The crowd cheered wildly at the sight of Liu Xiang, who was reported_______ the world record in the 110-meter hurdle race.
A.breaking
B.having broken
C.to have broken
D.to break
30.— Turn off the TV, Jack. _____ your homework now?
— Mum, just ten more minutes, please.
A.Should you be doing
B.Shouldn’t you be doing
C.Couldn’t you be doing
D.Will you be doing
31.Don’ t be ________ by products promising to make you lose weight quickly.
A.taken off
B.taken out
C.taken away
D.taken in
32.We had to wait half an hour ______ we had already booked a table.
A.since
B.although
C.until
D.before
33.— Has your father returned from Africa yet?
— Yes, but he_____ here for only three days before his company sent him to Australia.
A.was
B.has been
C.will be
D.would be
34.You’ll imagine what difficulty we had_____ home in the snowstorm.
A.walked
B.walk
C.to walk
D.walking
35.— Tony said he could fix my bicycle, but I really doubt it.
—_______He’s very good at this sort of thing.
A.Don’t worry
B.I couldn’t agree more
C.Of course
D.A piece of cake
第二节 完型填空(共20小题;每小题 1.5 分,满分 30 分)
阅读下面短文.掌握其大意,然后从 36-55各题所给的四个选项(A、B、C 和 D)中,选出最佳选项,并在答题卡上将该项涂黑。
Lang Lang is a world-class young pianist who grew up in Shenyang. He went to a piano school in Beijing when he was just eight. “You need 36 ,” his father said. “But if you don’t work hard, no fortune will come.”
What made him sad was 37 his piano teacher in Beijing didn’t like him. “You have no talent. You will never be a pianist. 38 a nine-year-old boy Lang Lang was badly 39 . He decided that he didn’t want to be a 40 any more. For the next two weeks he didn’t touch the piano 41 his father didn’t push, but waited.
Luckily, the day came when his teacher asked him to 42 some holiday songs. He didn’t to, hut as he placed his fingers on the piano key., he 43 that he could show others that he had talent__44__.That day he told his father 45 he had been waiting to hear--- that he wanted is study with a new teacher. 46 that point on, everything turned around.
He started 47 competitions(比赛). In the 1994 International Young Pianists Competition, when it was 48 that Lang Lang had won, he was too 49 to hold back his tears. Soon 50 as clear that he couldn’t stay in China forever—he had to play on the world’s big 51 in 1997 Lang Lang 52 again, this time to Philadelphia, U. S. There he spent Two years practicing. And by 1999 he had worked hard enough for fortune to take over. After his 53 performance at Chicago’s Ravinia Festival, gigs(特邀演出)in Lincoln Center and Carnegie hail started 54 Lang Lang finally worked to reach the place where fortune spots(发现) him, and lets him 55
36.A.exercise B.fortune C.knowledge D.wealth
37.A.whether B.why C.when D.that
38.A.Like B.With C.To D.As
39.A.hurt B.weakened C.ruined D.frightened
40.A.singer B.pianist C.Conductor D.player
41.A.Hopefully B.Patiently C.Wisely D.Painfully
42.A.play B. sing C.write D.study
43.A.seemed B.admitted C.noticed D.realized
44.A.in all B.above all C.after all D.at all
45.A.that B.what C.which D.when
46.A.From B.At C.Since D.After
47.A.receiving B.accepting C.winning D.beating
48.A.told B.mentioned C.announced D.recognized
49.A.excited B.encouraged C.shocked D.satisfied
50.A.this B.it C.that D.what
51.A. conceits B.tours C.competitions D.stages
52.A.started B.left C.moved D.performed
53.A.successful B.cheerful C.respectful D.meaningful
54.A.pulling B.breaking C.falling D.pouring
55.A.brighten B.shine C.admire D.develop
第三部分 阅读理解 (共 20 小题;每小题 2 分,满分 40 分)
阅读下列短文,从每题所给的四个选项(A、B、C 和 D)中,选出最佳选项.并在答题卡上将该项涂黑。
A
What will people die of 100 years from now? If you think that is a simple question, you have not been paying attention to the revolution that is taking place in bio-technology(生物技术). With the help of new medicine, the human body will last a very long time. Death will come mainly from accidents, murder and war. Today’s leading killers, such as heart disease, cancer, and aging itself, will become distant memories.
In discussion of technological changes, the Internet gets most of the attention these days. But the change in medicine can be the real technological event of our times. How long can humans live? Human brains were known to decide the final death. Cells(细胞) are the basic units of all living things, and until recently, scientists were sure that the life of cells could not go much beyond l20 years because the basic materials of cells, such as those of brain cells, would not last forever. But the upper limits will be broken by new medicine. Sometime between 2050 and 2100, medicine will have advanced to the point at which every 10 years or so, people will be able to take medicine to repair their organs ( The medicine, made up of the basic building materials of life, will build new brain cells, heart cells, and so on—in much the same way our bodies make new skin cells to take the place of old ones.
It is exciting to imaging that the advance in technology may be changing the most basic condition of human existence, but many technical problems still must be cleared up on the way to this wonderful future.
56.According to the passage, human death IS now mainly caused by____.
A.diseases and aging
B.accidents and war
C.accidents and aging
D.heart disease and war
57.In the author’ s opinion, today’s most important advance in technology lies in____.
A.medicine B. the Internet
C.brain cells D. human organ
58.Humans may live longer in the future because_____.
A.heart disease will be far away from us
B.human brains can decide the final death
C.the basic materials of cells will last forever
D.human organs can be repaired by new medicine
59.We can learn from the passage that ______.
A.human life will not last more than 120 years in the future
B.humans have to take medicine to build new skin cells now
C.much needs to be done before humans can have a longer life
D.we have already solved the technical problems in building new cells
B
Danielle Steel, America’s sweetheart, is one of the hardest working woman in the book business. Unlike other productive authors who write one book at a time, she can work on up to five. Her research some before writing takes at least three years. Once she has fully studied her subjects, ready to divided into a book, she can spend twenty hours nonstop at her desk..
Danielle Steel comes from New York and was sent to France for her education. After graduation, he worked in the public relations and advertising, industries. Later she started a job as a writer which she was best fit for. Her achievements are unbelievable: 390 million copies of books in print, nearly fifty New York Times best-selling novels, and a series of “Max and Martha” picture books for children to help them. Deal with the real-life problem of death, new babies and new schools. Her l998 book about the death of her was shot to the top of the New York Times best-selling list as soon as it came out. Twenty-eight of her books have been made into film. She is listed in the Guinness Books of World Records for one of her hooks being the Times best-seller for 381 weeks straight.
Not content with a big house, a loving family, and a view of the Golden Gate Bridge, Danielle Steel considers her readers to be the moat important resource(资源)and has kept in touch with them by e-mail. While she is often compared to the heroines(女主人公)of her own invention. Her life is undoubtedly much quieter. But if she does have anything in common with them, it is her strength of will and her inimitable(独特的)style. There is only one Danielle Steel.
60.Danielle Steel is different from other writers in that ______
A.she can write several books at the same time
B.she often does some research before writing a book
C.she is one of the most popular American women writers
D.she can keep writing for quite a long time without a break
61.Children who have read “Max and Martha” picture books may know ______
A.how to deal with affairs at school
B.what to do if Max and Martha die
C.what to do when new babies are born into their families
D.how to solve the difficult problems in their writing classes
62.One of Danielle Steel’ a achievements is that______
A.some TV plays were based on her books
B.her picture books attracted a lot of young men
C.one of her books became a best-seller in 1998
D.she wrote the Guinness Book of World Records
63.We can learn from the passage that Danielle Steel ______
A.lives an exciting life
B.values her readers a lot
C.writes about quiet women
D.is pleased with her achievements
C
Most rain forests lie close to the equator(赤道), where the climate is often mild and there are long hours of sunshine. The warmth of the land heats the air above, causing it to rise and tiny drops of water to fall as rain. The rainfall can reach at least 98 inches a year. This wet, warm world with plenty of sunlight is perfect for plants to grow so the trees grow fast with green leaves all the year round, The trees themselves also have an effect on the climate. They gather water from the soil and pass it out into the air through their leaves. The wet air then forms clouds, which hang over the treetops like smoke. These clouds protect the forest from the daytime heat and nighttime cold of nearby deserts, keeping temperatures fit for plant growth.
Rain forests slightly farther away from the equator remain just as warm, but they have a dry season of three months or morn when little rain falls. Tree leaves fall during this dry season and new leaves grow when the wet season or monsoon(雨季)begins. Thus these areas are known as the “monsoon forest”.
Another type of rain forest grows on tropical mountains. It is often called the “cloud forest” because clouds often hang over the trees like fog.
The rain forest is the ideal place for the growth of many different trees. Most of them depend on animals to eat their fruits and spread their seeds. When the fruits are eaten, the seeds inside them go undamaged through animals’ stomachs and arc passed out in their droppings. The seeds lying on the forest floor then grow into new trees.
64.The climate of the rain forests near the equator is______
A.mild, wet and windy
B.hot, rainy and foggy
C.hot, wet and cloudy
D.warm, wet and sunny
65.We can learn from the passage that_______
A.tree leaves are green all the lime in the monsoon forest
B.there is a dry season in the cloud forest on tropical mountains
C.clouds help the plants in the rain forest near the deserts to grow
D.the formation of climate in the rain forest has little to do with the trees
66.According to the passage,______ play with the most important role in the spreading seeds.
A.animals.
B.droppings
C.fruits
D.winds
67.This passage is most likely to be found in ______
A.a travel guide
B.a story book
C.a technical report
D.a geography book
D
All her life, my mother wanted busy children. It was very important that her house should remain at all times clean and tidy.
You could turn your hack for a moment in my mother’ s house, leave a half-written letter on the dining room table, a magazine open on the chair, and turn around to find that my mother had put it back where it belonged,” as she explained.
My wife, on one of her first visits to my mother’ s house, placed a packet of biscuits on an end table and went to the kitchen to fetch a drink. When she returned, she found the packet had been removed. Confused(疑惑的), she set down her drink and went back to the kitchen for more biscuits, only to return to find that her drink had disappeared. Up to then she had guessed that everyone in my family held onto their drinks, so as not to make water rings on the end tables. Now she knows better.
These disappearances had a confusing effect on our family. We were all inclined to(有……的倾向)forgetfulness. And it was common for one of us, upon returning from the bathroom, to find that every sigh of his work in progress had disappeared suddenly. “Do you remember what I was doing was a question frequently asked, but rarely answered.
Now my sister has developed a second-hand love of clean windows, and my brother does the cleaning in his house, perhaps to avoid having to be the one to hit his feet. I try not to think about it too much, but I have at this later time started to dust the furniture once a week.
We have all become busy persons.
68.Which of the following is TRUE about my mother?
A.She enjoyed removing others drinks.
B.She became more and more forgetful.
C.She preferred to do everything by herself.
D.She wanted to keep her house in good order.
69.Sly wife could not find her biscuits and drink in my mother’ a house because______.
A.she had already finished them
B.my mother had taken them away
C.she forgot where she had left them
D.someone in my family was holding them
70.The underlined part in the fifth paragraph suggests that my sister _______
A.is happy to clean windows
B.loves to dean used windows
C.is fond of clean used windows
D.likes clean windows as my mother did
71.This passage mainly tells us that _______
A.my mother often made us confused
B.my family members had a poor memory
C.my mother helped us to form a good habit
D.my wife was surprised when she visited my mother.
E
Ii may help you to know that there is no such thing as a perfect speech. At some point in every speech, every speaker says something that is not understood exactly as he has planned. Fortunately, the moments are usually not obvious(明显的)to the listeners. Why? Because the listeners do not know what the speaker plans to say. They hear only what the speaker does say. If you Lose your place for a moment, wrongly change the order of a couple of sentences, or forget to pause at a certain point, no one will be any the wiser. When such moments occur, don’t worry about them. Just continue as if nothing happened.
Even if you do make an obvious mistake during a speech, that don’ t really matter. If you have ever listen to Martin Luther King’ s famous speech—“I Have a dream ”, you may notice that he stumbles(结巴)his words twice during the speech, Most likely. however. you don’t remember. Why? Because you were fixing your attention on his message rather than on his way of speech-making. People care a lot about makings mistake in a speech because they regard speechmaking as a kind of performance rather than as an act of communication(交流). They feel the listeners are like judges in an ice-skating competition. But, in fact, the listeners are not looking for a period performance. They are looking for a well-thought-out speech that expresses the speaker’s ideas clearly and directly. Sometimes a mistake or he can actually increase a speaker’s attractiveness by making him more human.
As you work on your speech, don’t worry about being perfect. Once you free your mind of this, you will find it much easier to give your speech freely.
72.The underlined part an the first paragraph means that no one will ______
A.be smarter than you
B.notice your mistakes
C.do better than you
D.know what you arc talking about
73.You don’t remember obvious mistakes in a speech because_____
A.your attention is on the content
B.you don’t fully understand the speech
C.you don’t know what the speaker plans to say
D.you find the way of speech-making more important
74.It can be inferred from the passage that_____
A.giving a speech is like giving a performance
B.one or two mistakes in a speech may not be bad
C.the listeners should pay more attention to how a speech is made
D.the more mistakes a speaker makes, the more attractive he will be
75.What would be the best title for the passage?
A.How to Be a Perfect Speaker
B.How to Make a Perfect Speech
C.Don’ I Expect a Perfect Speech
D.Don’t Expect Mistakes in a Speech
第二卷
第四部分 写作(共两节,满分35分)
第一节 短文改错(共10小题;每小题1分,满分10分)
此题要求改正所给短文中的错误。对标有题号的每一行作出判断:如无错误,在该行右边横线上画一个勾(√);如有错误(每行只有一个错误),则按下列情况改正:
此行多一个词:把多余的词用斜线(\)划掉,在该行右边横线上写出该词,并也用斜线划掉。
此行缺一个词:在缺词处加一个漏字符号(∧),在该行右边横线上写出该加的词。
此行错一个词:在错的词下划一横线,在该行右边横线上写出改正后的词。
注意:原行没有错的不要改。
(试题内容见答题卡)
第二节 书面表达(满分25分)
大学生活即将开始,你将面对新的学习和生活环境,请根据示写一篇英语短文,谈谈你打算如何安排你的大学生活。内容要点应包括:
确定新的学习目标
改进学习方法
学会独立生活
参加各种课外活动
处理好与同学的关系
注意:①短言语的内容要连贯、完整;
②短文单词数:100左右(开头已给出的单词不计入单词总数)。
2007年普通高等学校招生全国统一考试(辽宁卷)
英语试卷
参考答案
第I卷
1.B  2.A  3.B  4.A  5.C  6.C  7.A  8.C
9.B  10.B  11.A  12.C  13.A  14.B  15.B  16.C
17.A  18.B  19.C  20.C  21.D  22.C  23.A  24.B
25.B  26.A  27.D  28.A  29.C  30.B  31.D  32.B
33.A  34.D  35.A  36.B  37.D  38.D  39.A  40.B
41.C  42.A  43.D  44.C  45.B  46.A  47.C  48.C
49.A  50.B  51.D  52.C  53.A  54.D  55.B  56.A
57.A  58.D  59.C  60.A  61.C  62.C  63.B  64.D
65.C  66.A  67.D  68.D  69.B  70.D  71.C  72.B
73.A  74.B  75.C
第II卷
76.am-was
77.the-a
78.plate-plates
79.pointing-pointed
80.正确
81.去掉one
82.while-when
83.They-It
84.late-later
85.添加at
One possible version
I will go to college in the near future. After I enter college, I plan to set new goals in my study and improve my way of learning. What’s more, as I am away from my parents, it is necessary for me to learn to live on my own, such as doing some washing and cleaning by myself. In my spare time, I will take part in different kinds of school activities, for example, I will often go to the English Corner to practice my spoken English. In addition, I need to get along well with my classmates and teaches at college. I think I will have a wonderful college life. (102 words)
2007年普通高等学校招生全国统一考试(重庆卷)
英 语
本试卷分第Ⅰ卷(选择题)和第Ⅱ卷(非选择题)两部分。第Ⅰ卷1至14页,第Ⅱ卷15至16页,共16页,共150分。考试用时120分钟。
第Ⅰ卷(三部分,共115分)
注意事项:
答题前,考生务必将自己的姓名、准考证号填写在试题卷和答题卡上,并将准考证号条形码粘贴在答题卡上的指定位置。
每小题选出答案后,用2B铅笔把答题卡上对应题目的答案标号涂黑。如需改动,用橡皮擦干净后,再选涂其他答案标号。答在试题卷上无效。
考试结束,监考人员将本试题卷和答题卡一并收回。
一、听力(共两节,满分30分)
第一节(共5小题,每小题1. 5分,满分7. 5分)
请听下面5段对话。每段对话后有一个小题,从题中所给的A、B、C三个选项中选出最佳选项,并标在试卷的相应位置。听完每段对话后,你都有10秒钟的时间来回答有关小题和阅读下一小题。每段对话仅读一遍。
例:How much is the shirt?
A. £19. 15 B. £ 9. 15 C. £9. 18
答案是B.
1. What does the woman want?
A. Tea. B. Water. C. Coffee.
2. Where are the two speakers?
A. In a bank. B. In a restaurant. C. In a post office.
3. Which book has the woman bought?
A. The math book. B. The history book. C. The English book.
4. What will the woman do this evening?
A. Meet her manager. B. Have a meeting. C. Go to the party.
5. What does the woman mean?
A. She hasn’t met the new director yet.
B. The new director went to London.
C. She doesn’t like the new director.
第二节(共15小题,每小题1. 5分,满分18分)
请听下面4段对话或独白。每段对话或独白后有几个小题,从题中所给的A、B、C三个选项中选出最佳选项,并标在试卷的相应位置。在听每段对话或独白前,你将有时间阅读各个小题,每小题5秒钟;听完后,每个小题将给出5秒钟的作答时间。每段对话或独白读两遍。
请听第6段材料,回答第6至8题。
6. Why did Susan call?
A. To say hello to her sick brother.
B. To discuss her business plan.
C. To ask about her family.
7. Where is Susan’s mother now?
A. At home. B. In a hospital. C. At a shopping center.
8. When will Susan probably come back home?
A. In one week. B. In two weeksC. In three weeks.
请听第7段材料,回答第9至11题。
9. What is the main purpose of the talk?
A. To tell the students how to use the library.
B. To tell the students what to do when ill.
C. To tell the students where to go next.
10. Who is the speaker?
A. A nurse. B. A teacher. C. A librarian.
11. What will the students probably do next?
A. See a doctor. B. Visit the librarian. C. Take some exercise.
请听第8段材料,回答第12至14题。
12. What is the relationship between the two speakers?
A. Teacher and student. B. Brother and sister. C. Classmates.
13. How will the speakers go to the beach?
A. By car. B. By bus. C. By train.
14. When are the two speakers leaving?
A. At 3:30. B. At 4:00. C. At 4:30.
请听第9段材料,回答第15至17题。
15. What did Mary want to be when she was small?
A. A painter. B. A swimmer. C. A singer.
16. At what age did Mary become famous?
A. 15. B. 16. C. 17.
17. Why does Mary stop swimming?
A. She is too old to win.
B. She wants to be a painter.
C. She needs more school education.
第三节(共3小题,每小题1. 5分,满分405分)
请听下面一段独白。用所听到的独白中的词或数填空,每空限填一个词或数。在听每段对话或独白前,你将有时间阅读各个小题,每小题5秒钟;听完后,每个小题将给出5秒钟的作答时间。每段对话或独白读两遍。
听第10段材料,回答第18至20题。
Switzerland
Land
About ____18_____% of its area is mountainous.
People
About 5% of its population was born ____19____.
Language
French is the ____20____ most widely-spoken language.
第二部分:英语知识运用(共两节,满分45分)
第一节 单项填空(共15小题;每小题1分,满分15分)
从A,B,C,D四个选项中,选出可以填入空白处的最佳选项。
21. Jim sold most of his things. He has hardly _______ left in the house.
A. anything B. everything C. nothing D. something
22. My parents live in a small village. They always keep candles in the house _____ there is a power out.
A. ifB. unlessC. in caseD. so that
23. —Did Alan enjoy seeing his old friends yesterday?
—Yes, he did. He ________ his old friends for a long time.
A. didn’t see B. wouldn’t see C. hasn’t seenD. hadn’t seen
24. Gorge couldn’t remember when he first met Mr. Anderson, but he was sure it was _____ Sunday because everybody was at _______ church.
A. /; the B. the; /C. a; /D. /; a
25. The children went home from the grammar school, their lessons _______ for the day.
A. finishing B. finishedC. had finished D. were finished
26. —Now let’s move on to another topic. Do you follow me?
—_______, Professor.
A. No, I am notB. Yes, I willC. No, I haven’tD. Yes, perfectly
27. It is not who is right but what is right ______ is of importance.
A. whichB. it C. that D. this
28. Leonardo da Vinci (1452 - 1519) ________ birds kept in cages in order to have the pleasure of setting them free.
A. is said to be buying B. is said to have bought
C. . had said to buy D. has said to have bought
29. —What do you think we can do for our aged parents?
—You ________ do anything except to be with them and be yourself.
A. don’t have to B. oughtn’t toC. mustn’t D. can’t
30. Human facial expressions differ from those of animals in the degree ______ they can be controlled on purpose.
A. with which B. to whichC. of whichD. for which
31. She’s having a lot of trouble with the new computer, but she doesn’t know whom to _______.
A. turn to B. look forC. deal withD. talk about
32. —Do you have any idea what Paul does all day?
—As I know, he spends at least as much time playing as he _______.
A. writesB. does writingC. is writingD. does write
33. —When I called you this morning, nobody answered the phone. Where______.
A. did you go B. have you goneC. were you D. had you been
34. _______, his idea was accepted by all the people at the meeting.
A. Strange as might it soundB. As it might sound strange
C. As strange it might sound D. Strange as it might sound
35. —I’ve studied growing plants as one of my interests. Could I make some suggestions?
— ______.
A. You will make itB. Go right ahead. C. Don’t mention it D. Take it easy
第二节:完形填空(共20小题;每小题1. 5分,满分30分)
  阅读下面短文,从短文后所给各题四个选项(A、B、C和D)中,选出可以填入空白处的最佳选项,并在答题卡上将该项涂黑。
In the clinic, I asked if Michael could be retested, so the specialist tested him again. To my __36__, it was the same score.
Later that evening, I _37_ told Frank what I had learned that day. After talking it over, we agree that we knew our _38_ much better than an IQ(智商) test. We _39_ that Michael’s score must have been a __40___ and we should treat him ___41___ as usual.
We moved to Indiana in 1962, and Michael studied at Concordia High School in the same year. He got _42 _ grades in the school, especially _43_ biology and chemistry, which was a great comfort.
Michael _44_ Indiana University in 1965 as a pre-medical student, soon afterwards, his teachers permitted him to take more courses than _45_. In 1968, he was accepted by the School of Medicine, Yale University.
On graduation day in 1972, Frank and I _46_ the ceremony (典礼) at Yale. After the ceremony, we told Michael about the _47_ IQ score he got when he was six. Since that day, Michael sometimes would look at us and say _48_, “My dear mom and dad never told me that I couldn’t be a doctor, not until after I graduated from medical school!” It is his special way of thanking us for the _49_ we had in him.
Interestingly, Michael then _50_ another IQ test. We went to the same clinic where he had _51_ the test eighteen years before. This time Michael scored 126, an increase of 36 points. A result like that was supposed to be _52_.
Children often do as _53_ as what adults, particularly parents and teachers, _54_ of them. That is, tell a child he is “ _55_”, and he may play the role of a foolish child.
36. A. joyB. surpriseC. dislikeD. disappointment
37. A. tearfullyB. fearfullyC. cheerfullyD. hopefully
38. A. studentB. sonC. friendD. doctor
39. A. arguedB. realizedC. decidedD. understood
40. A. jokeB. mistakeC. warningD wonder.
41. A. speciallyB. strictlyC. naturallyD. carefully
42. A. poorB. goodC. averageD. standard
43. A. in B. aboutC. ofD. for
44. A. visitedB. choseC. passedD. entered
45. A. allowedB. describedC. requiredD. offered
46. A. missedB. heldC. delayedD. attended
47. A. highB. sameC. lowD. different
48. A. curiouslyB. eagerlyC. calmlyD. jokingly
49. A. faithB. interestC. prideD. delight
50. A. looked forB. asked forC. waited forD. prepared for
51. A. receivedB. acceptedC. organizedD. discussed
52. A. imperfectB. impossibleC. uncertainD. unsatisfactory
53. A. honestlyB. muchC. well D. bravely
54. A. hear B. learnC. expectD. speak
55. A. wiseB. rudeC. shyD. stupid第三部分:阅读理解(共20小题;每小题2分,满分40分)
阅读下列短文,从每篇短文后所给各题的A,B,C,D四个选项中,选出最佳选项。
A
The very wealthy English Baron Fitzgerald had only one child, a son, who understandably was the apple of his eye. His wife died when the child was in his early teens. So Fitzgerald devoted himself to fathering the kid. Unfortunately the son died in his late teens.
Meanwhile, Fitzgerald's wealth greatly increased. He spent a lot on art works of the masters. Later Fitzgerald himself became seriously ill. Before his death, he had carefully prepared his will as to how his wealth would be settled-to sell his entire collection at an auction(拍卖).
Because of the large quantity and high quality of his collection, a huge crowd of possible buyers gathered for the auction. Many of them were museum directors and private collectors eager to bid(出价). Before the auction, the art works were shown, among which was a painting of Fitzgerald's son by an unknown artist. Because of its poor quality, it received little attention.
When it was time for the auction, the auctioneer gaveled(敲槌)the crowd to attention. First the lawyer read from Fitzgerald's will that the first art work to be auctioned was the painting of his son.
The poor-quality painting didn't receive any bidders. . . except one-the old servant who had served the son and loved him, and who for emotional reasons offered the only bid.
As soon as the servant bought the painting for less than one English pound, the auctioneer stopped the bidding and asked the lawyer to read again from the will. The crowd became quiet, and the lawyer read from the will: “Whoever buys the painting of my son gets all my collection. ”Then the auction was over.
56. The English Baron Fitzgerald was __________.
A. a museum director B. a master of art
C. an art collector D. an art dealer
57. Why did the old servant bid for the painting of Fitzgerald's son?
A. He was devoted to the family. B. He saw that no one bid for it.
C. He knew the content of the will. D. He found it cheap for him to buy.
58. Fitzgerald's will showed __________.
A. his desire to fool the bidders
B. his invaluable love for his son
C. his sadness at the death of his son
D. his regret of having no children to take over his wealth
B
Camp memories last forever! We make sure they are unforgettable!
Shadow Ridge Summer Camps offer so many exciting things for campers to do. Unlike other camp programs that include horses as a small part of their program, at Shadow Ridge horses ARE the program! We are 100%horse from stable(马厩)management, nature walks, and track rides to bedroom furnishings.
Horses help us achieve many of our aims. Girls can learn to develop responsibility, self-confidence and personal connections in their lives while having fun. Using horses as a wonderful tool for education, our camps offer an interesting place for growth and learning.
Imagine each girl having her very own horse to spend time with and a best friend to love and take care of. Each camper is responsible for a horse for the week. Our riding program provides a lot of riding and lesson time. Campers will learn how to take care of the horse and the tack(马具), as well as how to ride. Days are filled with horse-related activities to strengthen the connection between each girl and horse, as the girls learn to work safely around the horses.
At Shadow Ridge we try to create a loving, caring family atmosphere for our campers. We have “The Bunkhouse”(4 girls), the“ Wranglers Roost”(4 girls), and “The Hideout”(2 girls)in our comfortable 177-year-old farm house. All meals are home cooked, offering delicious and healthy food for the hungry rider.
Our excellent activities create personalized memories of your child's vacation, Each child will receive a camp T-shirt and a photo album(usually 300-500 pictures)of their stay at camp.
Our camps are offered during June, July and August 2007, for small groups of girls aged 13-16 years, not only from Canada but also other parts of the world.
We will send you full program descriptions at your request.
59. What is the main purpose of the passage?
A. To attract people to the camps. B. To talk about camping experiences.
C. To describe the programs of the camps. D. To explain the aims of the camps.
60. What do we know about the camp programs at Shadow Ridge?
A. Campers are required to wear camp T-shirts.
B. Horses play a central role in the activities.
C. Campers learn to cook food for themselves.
D. Horse lessons are offered all the year round.
61. The programs at Shadow Ridge mainly aim to help people .
A. understand horses better B. enjoy a family atmosphere
C. have fun above other things D. achieve an educational purpose
62. The passage is written mainly for .
A. horse riders B. teenage girls C. Canadian parents D. international travelers
C
Dear all,
Please read Professor Hume's email about his next lecture on Rosa Parks.
Susan Miller
Secretary
***************************************
Dear Susan,
Please forward this message to students of my history class.
Besides the life story of Rosa Parks in the textbook, the students are also required to read the passage below and some related stories that can be borrowed from the school library.
Ted Hume
The early experiences of Rosa Parks(1913-2005), long known as the “mother of the civil rights movement, ”were not different from those of many African-Americans at that time. The black woman, however, turned the course of American history in December 1955 when she refused to give up her seat on a bus to a white man. “By sitting down, ”remarked John Lewis, “she was standing up for all Americans. ”
Among the numerous awards Parks received in her life were the Presidential Medal of Freedom(1996)and the Congressional Gold Medal(1999).
Parks died on Oct. 24, 2005. At St. Paul A. M. E. Church in Montgomery, a large crowd including Secretary of State Condoleezza Rice celebrated her life. Rice said she and others, who grew up when the political activities of Parks held public attention, might not have realized her impact(影响)on their lives, “but I can honestly say that without Mrs. Parks, I probably would not be standing here as Secretary of State. ”
After her casket(灵柩)was placed at the Capitol, U. S. President Bush, members of Congress and ordinary Americans paid their respects. In American history Parks is the first woman to lie in state at the Capitol, a very high regard usually reserved for Presidents of the United States.
63. What is the main purpose of Susan’s email?
A. To make arrangements for Professor Hume's class.
B. To introduce to the students Rosa Parks.
C. To help the students organize a lecture.
D. To answer Professor Hume's last email.
64. What does the underlined word “forward” mean?
A. Explain. B. Send. C. Take. D. Read.
65. The political impact of Rosa Parks lies in the fact that she .
A. helped Condoleezza Rice achieve political success
B. joined the civil rights movement at a young age
C. made racial equality a common value in American society
D. set a good example in her early life for other black Americans
66. How was Rosa Parks treated after her death?
A. She was named “mother of the civil rights movement. ”
B. She was received by President Bush at the Capitol.
C. She was given the Presidential Medal of Freedom.
D. She was honored to lie in state at the Capitol.
D
The African elephant, the largest land animal remaining on earth , is of great importance to African ecosystem(生态系统). Unlike other animals, the African elephant is to a great extent the builder of its environment. As a big plant-eater, it largely shapes the forest-and-savanna(大草原)surroundings in which it lives, therefore setting the terms of existence for millions of other animals that live in its habitat(栖息地).
It is the elephant's great desire for food that makes it a disturber of the environment and an important builder of its habitat. In its continuous search for the 300 pounds of plants it must have every day, it kills small trees and underbushes, and pulls branches off big trees. This results in numerous open spaces in both deep tropical forests and in the woodlands that cover part of
the African savannas. In these open spaces are numerous plants in various stages of growth that attract a variety of other plant-eaters.
Take the rain forests for example. In their natural state, the spreading branches overhead shut out sunlight and prevent the growth of plants on the forest floor. By pulling down trees and eating plants, elephants make open spaces, allowing new plants to grow on the forest floor. In such situations, the forests become suitable for large hoofed plant-eaters to move around and for small plant-eaters to get their food as well.
What worries scientists now is that the African elephant has become an endangered species. If the elephant disappears, scientists say, many other animals will also disappear from vast areas of forest and savanna, greatly changing and worsening the whole ecosystem.
67. What is the passage mainly about?
A. Disappearance of African elephants.
B. Forests and savannas as habitats for African elephants.
C. The effect of African elephants' search for food.
D. The eating habit of African elephants.
68. What does the underlined phrase “setting the terms” most probably mean?
A. Fixing the time. B. Worsening the state.
C. Improving the quality. D. Deciding the conditions.
69. What do we know about the open spaces in the passage?
A. They result from the destruction of rain forests.
B. They provide food mainly for African elephants.
C. They are home to many endangered animals.
D. They are attractive to plant-eating animals of different kinds.
70. The passage is developed mainly by.
A. showing the effect and then explaining the causes
B. pointing out similarities and differences
C. describing the changes in space order
D. giving examples
E
Throughout the history of the arts, the nature of creativity has remained constant to artists. No matter what objects they select, artists are to bring forth new forces and forms that cause change-to find poetry where no one has ever seen or experienced it before.
Landscape(风景) is another unchanging element of art. It can be found from ancient times through the 17th-century Dutch painters to the 19th-century romanticists and impressionists. In the 1970s Alfred Leslie, one of the new American realists, continued this practice. Leslie sought out the same place where Thomas Cole, a romanticist, had produced paintings of the same scene a century and a half before. Unlike Cole who insists on a feeling of loneliness and the idea of finding peace in nature, Leslie paints what he actually sees. In his paintings, there is no particular change in emotion, and he includes ordinary things like the highway in the background. He also takes advantage of the latest developments of color photography(摄影术) to help both the eye and the memory when he improves his painting back in his workroom.
Besides, all art begs the age-old question: What is real? Each generation of artists has shown their understanding of reality in one form or another. The impressionists saw reality in brief emotional effects, the realists in everyday subjects and in forest scenes, and the Cro-Magnon cave people in their naturalistic drawings of the animals in the ancient forests. To sum up, understanding reality is a necessary struggle for artists of all periods.
Over thousands of years the fun_ction of the arts has remained relatively constant. Past or present, Eastern or Western, the arts are a basic part of our immediate experience. Many and different are the faces of art, and together they express the basic need and hope of human beings.
71. The underlined word “poetry” most probably means __________.
A. an object for artistic creation B. a collection of poems
C. an unusual quality D. a natural scene
72. Leslie's paintings are extraordinary because .
A. they are close in style to works in ancient times
B. they look like works by 19th-century painters
C. they draw attention to common things in life
D. they depend heavily on color photography
73. What is the author's opinion of artistic reality?
A. It will not be found in future works of art.
B. It does not have a long-lasting standard.
C. It is expressed in a fixed artistic form.
D. It is lacking in modern works of art.
74. What does the author suggest about the arts in the last paragraph?
A. They express people's curiosity about the past.
B. They make people interested in everyday experience.
C. They are considered important for variety in form.
D. They are regarded as a mirror of the human situation.
75. Which of the following is the main topic of the passage?
A. History of the arts. B. Basic questions of the arts.
C. New developments in the arts. D. Use of modern technology in the arts.
第II卷(共35分)
第四部分:写作(共两节,满分35分)
第一节 短文改错(共10小题;每小题1分,满分10分)
此题要求改正所给短文中的错误。对标有题号的每一行作出判断:每行只有一个错误,请按下列情况改正:
该行多一个词:把多余的词用斜线()划掉,在该行右边横线上写出该词,并也用斜线划掉。
该行缺一个词:在缺词处加一个漏字符号(^),在该行右边横线上写出该加的词。
该行错一个词:在错的词下划一横线,在该行右边横线上写出改正后的词。
注意:请在答题卡上作答。
Last weekend we went mountain-climbing. Even the
Heavy rain in the morning could not prevent us go. Setting 76. ______
Off very early, we went along an extreme narrow road, all 77. ______
in high spirits. On every side of the road were green fields 78. ______
and some farm house. We could hear the sound of the 79. ______
rain and our footsteps mixing with our laughter. At noon80. ______
we reached the top of the mountain. That surprised us 81. ______
most there was the beauty of scenes. After having a short82. ______
rest there and sharing with the food we had brought, we83. ______
started going down. It had rained even harder. We were84. ______
wet to the skin, and we still sang and laughed happily. 85. ______
第二节 书面表达(满分25分)
据报道,7岁的美国女孩Amy Bruce得知自己身患肺癌(lung cancer)后处于极度的痛苦之中,美国抗癌协会(ACS)决定,每当Amy收到一封慰问信,就给她增加3美分的治疗款。
假如你是新华中学的学生李华,请你用英文给Amy写一封慰问信,主要内容包括:
自我介绍
得知的情况
表示鼓励
打算为她做什么
祝她早日康复
注意:
信的格式已为你写好
信中不得出现你的真实情况。
词数100左右。
请在答题卡上作答。
参考答案
听力
1-5 BCCAA6-10 CCBBA11-15 BCACA16-17 BA 18. 70 19. foreign 20. second / 2nd
单项选择
21-25 ACDCB26-30 DCBAB31-35 ABCDB
完形填空
36-40 DABCB41-45 CBADC46-50 DCDAB51-55 ABCCD
阅读理解
56-60 CA BAB61-65 DCABC66-70 DCDDA71-75 CCBDB
改错
76. go→going 77. extreme→extremely 78. every→each/ either 79. house→houses 80. mixing→mixed 81. That→What 82. scenes前加the 83. 去掉with 84. 去掉had 85. and →but
书面表达
Dear Amy,
I am a student from Xinhua Middle School. I am very sorry to have heard of your present situation that you are so bitter after having a lung cancer.
At present, ACS decides that it will give you three cents when you receive a letter. That will offer you so much courage. I hope you will make efforts to overcome all kinds of difficulty, no matter how hard it is. You are sure to make it.
I will try my best to ask all my friends, schoolmates and relatives to write letters to you, which may help you. It will give more courage to live on.
May you recover soon.
Yours,
Li Hua
联系人 :黄刚
联系地址:重庆垫江一中
邮政编码: 408300
E-mail:hgang168@163. com
英 语
一、英语知识运用(共三大题,满分50分,)(一)语音知识(共5小题,每小题1分,满分5分)
从每小题的A、B、C、D四个选项中,找出其划线部分与所给单词的划线部分读音相同的选项,并在答题卡上将该选项涂黑。
1.success (D)
A.officialB.excuseC.correctD.exciting
2.society(C)
A.achieveB.friendC.scienceD.believe
3.theirs (B)
A.threadB.smoothC.thirstyD.bealth
4.raise(A)
A.remainB.certainC.moantainD.bargain
5.compare(B)
A.companyB.continueC.commonD.concert
(二)语法和词汇知识(共15小题,每小题1分,满分15分)
从每小题的A、B、C、D四个选项中,选出可以填入空白处的最佳选项,并在答案卡上将该选项涂黑。
6.- Could I use your computer for a few moments,please?
- . I’m not using it myself.
A.Come onB.It dependsC.Go aheadD.That’s great
7.Though he started late. Mr. Guo played the piano as well as,if , (A)
A.not better thanB.not betterC.no better thanD.better
8. parents say and do has a life-long effect on their children. (C)
A.ThatB.WhichC.WhatD.As
9.As a result of the serious flood, two-thirds of the buildings in the area
.(A)
A.need rcpairingB.needs to repairC.needs repairingD.need to repair
10. that Maric was able to set up new branches clsewhere.(B)
A.So successful her business wasB.So successful was her business
C.So her business was successfulD.So was her successful business
11.We had wanted to finish our task by noon, but it didn’t quite .(D)
A.find outB.give outC.hand outD.work out
12.He and his wife are of the same ;they both want their son to go to college.(D)
A.soulB.spiritC.hand outD.work out
13. – I think he is taking an active part in social work. I agree with you
.(A)
A.in a wayB.on the wayC.by the wayD.in the way
14.I told your friend how to get to the hotel, but perhaps I have driven her ther.(D)
A.couldB.mustC.nightD.should
15. that she didn’t do a good job, I don’t think I am abler than her.(B)
A.To have saidB.Having saidC.To sayD.Saying
16.- There is still a copy of the book in the library. Wall you go and bottow
?
- No, I’d rather buy in the bookstore.(A)
A.it;oneB.one;oneC.one;itD.it;it
17.In film Cast Away.Tom Hanks plays man named Chuck
Noland.(B)
A.a;theB.the;aC.the;theD.a;a
18.- when has the country been open to international trade?
- 1978, I suppose.(A)
A.SinceB.InC.FromD.After
19.I there little more than a week when I set to work with the scientist.(C)
A.would beB.have beenC.had beenD.will be
20.Today, we’ll discuss a number of cases beginners of English fail to use the language properly.(D)
A.whichB.asC.whyD.where
(三)完形填空(共20小题,每小题1.5分,满分30分)
阅读下面短文,从短文后各题的A、B、C、D四个选项中,选出适合填入对应空白处的最佳选项,并在答题卡上将该选项涂黑。
I was having my dioner at McDonald’s one evening when so old couple siowly walked in. They 21 their meal. Took a table near the window and started 22 food out of the plate. There was one hamburger, one order of French fries(炸薯条) and one drink. The man 23 the food into two halves and carefully placed 24 before his wife.
He took a sip(一小口)of the drink. His wife also took one and then 25 the cup down between them. “That 26 old couplel All they can 27 is one meal ofr the two of them,”thought L 28 the man began to eat his French fries. I 29 to my feet, went over and said that I was 30 to buy another meal for them. Bet he
31 refused me and said that they made it a 32 to share everything.
33 ,the lady didn’t take a bite, She sat there 34 her husband eat, and taking turns (轮流) sipping the drink, Again I 35 to buy them something but was refused,When the man finished eating and was 36 his face with a napkin (纸巾).
I 37 no longer stand it. I made an offer to them a third time. 38 being politely refused, I asked the lady 39 ,“Madam, why aren’t you eating? You said that you share everthing, 40 is it that you are waiting for?”“The teeth,”she auswercd.
21.A.servedB.requestedC.collectedD.ordered(D)
22.A.carryingB.rakingC.fetchingD.bringing(B)
23.A.dividedB.cutC.changedD.formed(A)
24.A.itB.thisC.thatD.one(D)
25.A.gotB.settledC.setD.torned(C)
26.A.funnyB.crazyC.strangeD.poor(D)
27.A.affordB.payC.demandD.choose(A)
28.A.WhileB.SinceC.AsD.Until(C)
29.A.cameB.struggledC.rushedD.rose(D)
30.A.anxiousB.willingC.satisfiedD.quick(B)
31.A.warmlyB.proudlyC.kindlyD.seriously(C)
32.A.wayB.habitC.caseD.model(B)
33.A.SurprisinglyB.SadlyC.ShockinglyD.Bitterly(A)
34.A.seeingB.noticingC.watchingD.finding(C)
35.A.wantedB.askedC.plannedD.attempted(B)
36.A.wipingB.touchingC.bathingD.washing(A)
37.A.shouldB.couldC.mightD.would(B)
38.A.InB.UponC.AfterD.With(C)
39.A.curiouslyB.carefullyC.naturallyD.plainly(A)
40.A.HowB.WhoC.WhyD.What(D)
二、阅读理解(共25小题,阅读部分每小题2分,补全对话每小题1分,满分45分)
(一)阅读下列五篇短文,从每小题后所给的A、B、C、D四个选项中,选出最佳选项,并在答题卡上将该选项涂黑。
A
Louis Armstrong had two famour nicknames (绰号). Some people called him Bagamo. They said his mouth looked like a large bag, Musicians often called him Pops,as a sign of respect for his influence (影响) on the world of music.
Born in 1901 in New Orleans, be grew up poor, but lived among great musicians. Jazz was invented in the city a few years before his birth. Armstrong often said,“Jazz and I grew up together.”
Armstrong showed a great talent (天赋) for music when he was taught to play the cornet (短号) at a boy’s home. In his late teens, Armstrong began to live the life of a musician. He played in parades, clubs, and on the steamboats that traveled on the Mississippi River. At that time, New Orleans was famous for the new music of jazz and was home to many great musicians. Armstrong learned from the older musicians and soon became respected as their cqual.
In 1922 he went to Chicngo. There, the tale of Louis Armstrong begins. From then until the end of his life, Armstrong was celebrated and loved wherevet be went Armstrong had no equal when it came to playing the American popular song.
His cornet playing had a deep humanity (仁爱) and warmth that caused many listeners to say, “Listening to Pops just makes you feel good all over.”He was the father of the jazz style(风格) and also one of the best-known and most admired people in the world. His death, on July 6,1971, was headline news around the world.
41.Armstrong was called Pops because he .(B)
A.looked like a musieianB.was a musician of much influence
C.showed an interest in musicD.traveled to play modern music
42.The third paragraph is developed .(C)
A.by space B.by examplesC.by timeD.by comparison
43.Which statement about Armstrong is true?(C)
A.His tale begins in New Orleans.
B.He was born before jazz was invented.
C.His music was popular with his listeners.
D.He learned popular music at a boy’s home.
44.Which would be the best title for the text?(B)
A.The Invention of the Jazz Music
B.The Father of the Jazz Style
C.The Making of a Musician
D.The Spread of Popular Music
B
When people want to know about the wcather, they usually go to there radios, TVs, pewspapers, or to the Internet. However, you can also find many weather signs among wildlife, because of their highly developed senses. Drops in air pressure(压力) produce an effect on small animals in many ways. Mice and deer are good weather indicators. People who spend a lot of time outdoors have observed that, before a storm, field mice come out of their holes and run around, Deer leave high ground and come down from the mountains.
Birds are especially good weather indicators because they also show the effect of a prcssure drop in many ways. For example, some birds become irritable(急躁的)and qusrrelsome and will fight over a picce of bread. Other birds chirp(叽叽喳喳)and sing just before a storm. It seems they know they won’t get snother chance for an bour on two. Birds also seek safe placcs before a storm, You will sometimes see birds settling in trees or gathering together on a wire close to a building. Pre-storm low pressure makes the are so thin that birds have difficulty flying.
It is unusual to see many birds flying overhead in the summertime, rather than during the periods in the spring or autumn. Watch for other weather signs if you see this. If they fly in the wrong direction,they may be flying abead of a storm.
By paying closer attention to some important signs in nature, we can become better prepared for any kind of weather.
45.The word“indicators”in paragraph I probably means .(C)
A.mapsB.servicesC.signsD.stations
46.There will be a storm if birds .(A)
A.make more noise than usualB.fly in different directions
C.come down from tall treesD.share a piece of bread
47.How can birds sense the coming of a storm?(C)
A.By feeling a drop in air temperature.
B.By noticing the change of wind directions.
C.By feeling a drop in air pressure.
D.By notieing the movements of other animals.
48.The best title for the text would be .(D)
A.Signs of a StormB.Drops in Air Pressure
C.Animals’Sharp SensesD.Nature’s Weather Signs
C
When former American President Bill Clinton traveled to South Korea to visit President Kim Young Sam, be rcpeatedly referred to the Korean president’s wife as Mrs. Kim. By mistake, Prcsident Chnton’s advisers thought that Koreans have the same naming customs as the Japanese. Clinton had not been told that, in Korea, wives keep their family names. President Kim Young Sam’s wife was named Sohm Myong Suk.Thercfore, she should be addressed (称谓)as Mrs. Sohn.
President c*ntot arrived in Korea directly after leaving Japan and had not changed his culture gears. His failare to follow Korean customs gave the imprassion that Korea was not as important to him as Japan.
In addition to Koreans. Some Asian husbands and wives do not share the same family names. This practice often puzzles(使困惑) English-speaking teachers when talking with a pupil’s parents. They become puzzled about the student’s correct last name. Placing the family name first is common among a number of Asian cultures.
Mexican naming customs are different as well. When a woman marries,she keeps her family name and adds her husband’s name after the word de (of). This affects (影响)how they fill in forms in the United States. When requested to fill in a middle name, they generally write the father’s family name. But Mexicans are addressed by the family name of the mother. This often causes puzziement.
Here are a few ways to deal with such difficult situations:don’t always think that a married woman uses her husband’s last name. Remember that in may Asian cultures,the order of first and last names is reversed (颠倒),Ask which name a person would prefer to use. If the name is difficult to pronounce,admit it, and ask the person to help you say it conectiy.
49.The story of Bill Clinton is used to .(B)
A.improve US Kotean relations
B.introduce the topic of the text
C.describe his visit to Korea
D.tell us how to address a person
50.The word “gears”in paragraph 2 is closest in meaning to .(D)
A.action plansB.naming customsC.travel mapsD.thinking patterns
51.When a woman marries in Kores, she .(A)
A.continues to use her family name
B.uses her husband’s given name
C.shares her busband’s family name
D.adds her husband’s given name to hers
52.To address a married woman properly, you’d better .(C)
A.are her middle nameB.use her husband’s first name
C.ask her which name she likesD.change the order of her names
D
Teens don’t understand the big fass (小题大做).As the first generation to grow up in a wired world,they hardly know a time when computers weren’t around, and they cagerly eatch the chance to spend hours online, chatting with friends,So what?
But researchers nationwide are increasingly worried that teens are becoming isolated(孤寂),less skillful at person-to-person relationships, and perhaps numb (麻木) to the chcatings that are so much a part of the e-mail world.“And a tenn’s sense of self and values may be changed in a world where personal connections can be limitless,”said shetty Turklt.
Another resarcher, Pobert Kraut, said he’s worried about the“opportunity costs”(机会成本) of so much online time for youths. He found that teens who used computers,even just a few hours a week, showed incrcased signs of loneliness and socol isolation.“Chatting onine may be better than watching television, but it’s worse than hanging out (闲逛)with real friends,”he said.
Today’s teens, however,don’t see anything strange in the fact that the computer takes up a central place in their social lives,“School is busy and full of pressurs. There’s almost no time to just hang out.”said Parker Rice, 17. “Talking online is just catch up time.”
Teens say they feel good about what they say online or taking the time to think about a reply. Some teens admit that asking someone for a date, or breaking up, can be easier in message form,though they don’t want to do so. But they insist there’s no harm.
53.The researchers argue that .(A)
A.teens may develop a different sense of values
B.nothing is wrong with teens’chatting online
C.teens can manage their social connections
D.spending hours online does much good to teens
54.The text mainly deals with .(D)
A.use computers properlyB.improve their school work
C.develop an interest in soeial skillsD.reduce their mental pressures
55.The text mainly deals with .(D)
A.tcens’ pleasant online experience
B.teens’ computer skills and school work
C.the effects of the computer world on teens
D.different opinions on teens’ chatting online
56.The purpose of the text is to .(B)
A.describe computer research results
B.draw attention to teens’ computer habits
C.suggest ways to deal with problem teens
D.discuss problems teens have at school
E
Lest year more than one million Filipinos worked abroad as servants, nurses,sailors and in other difficult but low paid jobs. Southeast Asians leave their poorer countries for their richer neighbours.
Many of these wage earners return in the end. In the meantime, they send home huge amounts of money-in the Philippines’ case, over 10% of its GDP. Between January and November, the amount was up 18% on the same period of 2005, Poverty and unemployment are still bigh in the Philippines and other labour exporring(劳务输出)countries. They would be far worse but for this outflow of bodies and inflow of doflars As for those Asian countries that inport (输入)labour,as in Europe, falling birth rates mean they are going to need more foreign workers.
On Jamuary 13th leaders of the Association of Southeast Asian Nations(ASEAN)signed an agreement to help migrant(流动的)workers-with the realization that the flow of labour between their countries is a growing problem that they cannot blame on outsiders. A 2005 study showed that 8.4 million Southeast Asians worked outside their home countries, but this did not include the huge numbers of Indonesians doing so without papers. So the true total is probably rather higher.
Of the ten ASEAN countries, the Philippines,Indonesia, Myanmar,Carnbodia,Victram and Lacs export labour, Singapore and Brunci import it, and Thailand and Malaysia do both. Sriracaki of the UN’s Internstional Labour Organiztion points out that,in the next ten years,the total labour foree of the worker expotting countries should grow by about a third. It makes semse to work on the problem before it gets out of control.
People in the receiving countries seem to be worried about competition for their jobs.Most Thais said theit government should admit (准入) no more foreign workers, and a few thought otherwise. Even in Singapore,just over ball of people are against adunitting more foreign workers. Malaysians think that the incrcase in foreign workers has worsened crime rates(犯罪率).
57.It can be inferred from the case of the Philippines that .(C)
A.the country is Asla’s main source of migrant workets
B.labour exparts lend to a 10% growth of its GDP
C.the outflow of lend to helps solve its social probiems
D.the country Both exports and imports labour force
58.The flow of labour is a growing problem because .(A)
A.there is a greater flow of labour than reported
B.more Indonesians work abroad without papers
C.some countries suffer from low birth rates
D.the ASEAN is against admitting foreign workers
59.For the labour importing countries, the flow of labour may lead to .(D)
A.higher birth ratesB.lower crime rates
C.greater money inflowsD.stronger job competition
60.The writer of the text seems to .(B)
A.support the flow of labour between countries
B.report fairly on the question of labour flow
C.express his worries over the ASEAN’s decision
D.regard the outflow of labour as a serious problem
(二)根据对话情景和内容,从对话后所给的选项中选出能填入每一空白处的最佳选项,并在答题卡上将该选项涂黑。选项中有两个为多余选项。
Li Hua,who is pleuning to travel with his parents, is now calling Hyan Travel
Ageney in Xi’an. (A:AssistantB:Li Hua)
A:Hello,this is Hyatt Travel Agency. May I help you?
L:Yes. This is Li Hua. I’ll have a short holiday, and I’m thinking about traveling abrose with my parents. 61 (D)
A:Of course. May I ask how long your holiday will be? L:About a week.
A:OK. 62 (C) L:I’d love to go to Europe.
A: 63 We have a special offer for European tours. (B) L:Great! 64 (F)
A:Sure. For family tours, we have five European cities on the list, and we’ll see the Big Ben in London,the Eiffel Tower in Paris and other places of interest.
L: 65 I’ll talk about this with my parents and call you cack. Thank you.(E)
A:You are welcome.
A.What would you like to know?B.You are very lucky.
C.What do you have in mind?D.Can you pive me some advice?
E.That’s just what I am thinking of.F.Can I have some extra information?
G.I hope you’ll have a good journey there.
三、写作(共三大题,满分55分)(一)单词拼写(共10小题,每小题1分,满分10分)
根据下列各句句意和空白之后的汉语提示词,在答题卡指定区域的横线上写出对应单词的正确形式,每空只写一词。
66.Spring has come, and the trees are thick with green leaves (叶子).
67.Andorra is a small mountain republic (共和国)between France and Spain.
68.I consider it impossible (不可能) to believe a single word you say.
69.He turned on the television sct hanging from the ceiling (天花板).
70.We were talking on the phone when, suddenly (突然), the line went dead.
71.One of my favourite sayings is “Understand all and fargive (宽怒)all.”
72.A man I recognized/trcognized (认出)as his father sat with a newspaper on his knees.
73.Techaical progress would put out firm in posseasion (拥有) of the home market.
74. Winning (赢得) the support of the majority requires time,energy and devotion.
75.Xi’an International Exhibition (展览) Center has attracted many business people.
(二)短文改错(共10小题,每小题1.5分,满分15分)
据题要求在答题卡上改正所给短文中的错误。请根据上下文对标有题号的每一行作出判断;如无错误,在该行右边横线上划一个勾(√);如有错误(每行只有一个错误),刚按下列情况改正;
此行多一个词:把多余的词用斜线()划掉,在该行右边横线上写出该词,并用斜线划掉。
此行缺一个词:在缺词处加一个漏字符号(∧),在该行右边横线上写出应加的单词。
此行错一个词:在错词下划一道线,在该行右边横线上写出改正后的单词。
注意:原行没有错误的不要改。
Are the years you spent at school best years of you life?76. the
Personally, I found most lesson rather uninteresting. We had to77. lessons
sit at our desks in silence and paid attention to what the teachers78. pay
were saying. They were used to write on the blackboard and ask79. were
us difficult questions. We also had to do plenty of homework,and80. √
hand it on time. We had to wear school uniforms and obey lots of81. in
rules. I left school as soon as I can and started work. I read books 82. could
at the publie library, and late I decided to attend college. Now83. later
I really enjoy study because I’m growing old and know what I84. studying
Want it. When I was at school. I was just the wrong age!85. it
(三)书面表达(满分30分) 请你根据所给材料、提纲以及要求写一篇英文稿件。
背景材料:
English Horizons 第25卷 总228(5/2007)期
编者按:本期刊出Lora的来信,欢迎读者就信中谈到的问题逆 互动空间
行分析并提出建议。
Dear editor.
I’m a senior high school student,and I want to work part-time this summer. I think it is good to do so,but my parents don’t agree with me I just can’t understand them I need your advice. Lara
要求:
1.短文须写在答题卡的指定区域。
2.短文词数不少于80(不含已写好的部分)。
3.内容充实,结构完整,语意连贯。
4.书写须清晰、工整。
提纲:
第1段:Lora利用暑假打工的理由
第2段:Lora父母不同意的理由
第3段:你给Lora的具体建议
I think that Lora wants to work part-time for these reasons. First,she can make some pocket money to meet pet daily needs and thus develop a sense of independenca. Second. she can learn how to deal with various problems in the workplace. Third,she can build up new friendships and improve bet social skills.
But her parents may regard learning as an increasingly important task for Lora. This holiday should be a catch-up time for independent study. Besides,personal safety may also be one of their maior considerations.
My advice for Lora is this:make a detailed plan for her study and work;ask her friends to work together with her, and tell her parents that she ean manage things properly. I wish Lora a pleasant summer.
B卷选择题答案 1.B2.D3.C4.B5.A6.D7.C
8.D9.C10.A11.C12.A13.D14.B15.A16.C
17.A18.B19.D20.B21.C22.A23.B24.A25.D
26.B27.C28.A29.B30.D31.B32.D33.D34.A
35.C36.D37.C38.A39.B40.C41.A42.B43.B
44.D45.D46.C47.B48.A49.A50.C51.C52.D
53.D54.C55.B56.D57.A58.C59.B60.A61.C
62.B63.A64.E65.F

展开更多......

07年英语高考卷_21世纪教育网-二一教育 (2024)
Top Articles
Latest Posts
Article information

Author: Fredrick Kertzmann

Last Updated:

Views: 5721

Rating: 4.6 / 5 (46 voted)

Reviews: 85% of readers found this page helpful

Author information

Name: Fredrick Kertzmann

Birthday: 2000-04-29

Address: Apt. 203 613 Huels Gateway, Ralphtown, LA 40204

Phone: +2135150832870

Job: Regional Design Producer

Hobby: Nordic skating, Lacemaking, Mountain biking, Rowing, Gardening, Water sports, role-playing games

Introduction: My name is Fredrick Kertzmann, I am a gleaming, encouraging, inexpensive, thankful, tender, quaint, precious person who loves writing and wants to share my knowledge and understanding with you.